You are on page 1of 224

2/5

SMLE
KSAU-HS
Question Bank
1.10 Edition
Surgery Questions

This is an accumulative effort from King Saud bin Abdulaziz University for Health Sciences (2016-17/
Batch 9) interns to organize and answer what have been collected previously from SMLE Q Bank
2015-16

We would like to acknowledge:


- King Saud bin Abdulaziz University for Health Sciences (2016-17/Batch 9) interns for their huge efforts in
accomplishing this project
- SMLE Q Bank Group
‫ مدونة طالب طب سعودي‬-

‫جهد بشري قابل للخطأ والصواب‬


For any comments, kindly contact us at
SMLE2016.17@gmail.com

 Highlighted in yellow are queried questions


 Highlighted in red are some repeated questions

2
Surgery

Table of Contents
General Surgery ........................................................................................................................................... 4
Plastic Surgery ........................................................................................................................................... 69
Pediatrics Surgery ...................................................................................................................................... 76
Urology ..................................................................................................................................................... 79
ENT ........................................................................................................................................................... 99
Ophthalmology.........................................................................................................................................125
Orthopaedics ............................................................................................................................................160
Anesthesia ...............................................................................................................................................215
Extra information .....................................................................................................................................221

3
General Surgery

4
1. Internal iliac injured which is affected

A. Anterior thigh muscle


B. Ovary
C. Bladder
D. Anterior abdomen
Answer: C
The arterial blood supply of the bladder arrives primarily via the internal iliac (hypogastric) arteries. These branch into the umbilical
artery, which supplies several superior vesicle branches, and inferior vesical arteries, which come as direct internal iliac branches in
males or from the vaginal arteries in females.
(http://emedicine.medscape.com/article/1949017-overview#a2)

2. Case Necrotizing fasciitis treatment?

A. Imipenem & metronidazole


B. Ampicillin & gentamicin
C. Pipracillin & tazobactam
D. Penicillin and smth

Answer: C
Necrotizing fasciitis is a rapidly progressive inflammatory infection of the fascia, with secondary necrosis of the subcutaneous
tissues.
For treatment refer to:
(http://emedicine.medscape.com/article/2012058-overview)

3. MVA when to do diagnostic peritoneal lavage:


A. Hypotensive patients.
B. All MVA patients.
C. Unconscious patients with severe head injury.
D. Conscious patients with abdomen pain.
Answer: A
DPL can be used to evaluate both blunt and penetrating abdominal trauma in patients who are hemodynamically unstable or who
require urgent surgical intervention for associated extra-abdominal injuries. DPL can rapidly confirm or exclude the presence of
intraperitoneal hemorrhage. Thus, the patient with a closed head injury, the unstable patient who has been in a motor vehicle
accident, or the patient with a pelvic fracture and potential retroperitoneal hemorrhage can be appropriately triaged to emergency
laparotomy.
Http://emedicine.medscape.com/article/82888-overview

4. Patient with BPH best investigation:


A. Cystoscopy
B. Annual renal function
C. Annual prostate antigen

5
D. Beta-blocker
Answer : C
Source: http://emedicine.medscape.com/article/437359-workup
And Toronto notes 2014 FM17

5. A patient undergone for a major surgery, a days later, his surgical wound was infected, what is the most likely source of
infection:
A. The patient visitor at visiting time.
B. The staff hands during examination and dressing.
C. The pressure dressing gauze.
D. The dressing tools and devices
Answer: B

6. A child falls down the stair and his head hit the floor. On examination, he was alert and oriented, not having any neurological
deficits, he had only bleeding from his right ear, on further ear exam, the eardrum was obviously rupatient ured. What is the
most likely the source of the bleeding?
A. Subdural Hemorrhage.
B. Skull base Fracture.
C. Subarachnoid hemorrhage.
D. Other choices I can not remember.
Answer: B

7. A child suddenly hit a hard object, he sustained a forehead wound, where will this wound be drained?
A. Pre auricle lymph nodes.
B. Retro auricle lymph nodes.
C. Occipital lymph nodes.
D. Sub-mental lymph nodes.
Answer: A
Forehead drain in pre auricle ( parotid)

8. During nissen fundoplication the surgeon injure posterior vagal trunk. Which of the following structure will be affected
A. Esophagus
B. Jejunum
C. Descending colon
D. Urinary bladder

6
Answer: A
9. Which of the following anaphylactic shock appears initially then disappear?
A. Patchy
B. Papule
C. Macule or nodule (I can not remember )
D. Whale* ?? (not clear choice)

Answer:

10. Sickle cell patient with recurrent RUQ pain with elevated pancreatic exam ,During U/S there was multiple gall stone (I think 7
gall stones), The largest one was2.5 cm when you did ERCP there was no stone in billiary tree , what is your management ?
Antibiotic
Answer: Cholecytectomy

11. What is the most common presentation of lower intestinal obstruction ?


A. Alternating diarrhea and constipation
B. Absolute constipation
C. Vomiting
D. Abdominal distention
Answer: D the answer should be colicky abdominal pain
Http://bestpractice.bmj.com/best-practice/monograph/877/diagnosis/history-and-examination.html

12. Newborn baby complains of (many signs that indicate distress likes tachypnea). Patient looks unwell on examination.
Auscultation of the left side of chest shows no breath sound and the heart sound heard in right side. What is the most likely
diagnosis?
A. Situs inversus
B. Pneumothorax
C. Bowl hernia
D. Another not related answer
Answer: C.
Infants with congenital diaphragmatic hernia (CDH) most often present with respiratory distress in the first few hours or days of life.
Physical findings include a barrel-shaped chest, a scaphoid-appearing abdomen (because of loss of the abdominal contents into the
chest), and absence of breath sounds on the ipsilateral side. In patients with a left-sided CDH, the heartbeat is displaced to the right
because of a shift in the mediastinum.
Http://cursoenarm.net/UPTODATE/contents/mobipreview.htm?31/57/32657

13. Routine breast self-exam advice:


A. Daily
B. Weekly
C. Monthly
D. Yearly
Answer: C, New Evidence: Not Advised
Http://www.uspreventiveservicestaskforce.org/Page/Document/updatesummaryfinal/breast-cancer-screening

7
14. Elderly bedridden has an ulcer 4*5 reaching fascia and muscle, what is the stage?
A- Stage 1
B- Stage 2
C- Stage 3
D- Stage 4
Answer: D

15. What is the recommended management for a patient with carotid bruit with occlusion 60% of the left carotid artery?
A. Aspirin daily
B. Angiography
C. Carotid endarterectomy
D. None of the above
Answer: A
We recommend intensive medical therapy using all available risk reduction strategies for patients with asymptomatic carotid
atherosclerosis. Currently viable strategies include statin therapy, antiplatelet therapy, blood pressure control, and lifestyle
modification consisting of smoking cessation, limited alcohol consumption, weight control, regular aerobic physical activity, and a
Mediterranean diet.
We suggest CEA (see 'Carotid endarterectomy' below) plus intensive medical therapy, rather than intensive medical treatment
alone, for medically stable patients who have a life expectancy of at least five years and a high grade (≥80 percent) asymptomatic
carotid atherosclerotic stenosis at baseline or progression to ≥80 percent stenosis despite intensive medical therapy while under
observation, provided the combined perioperative risk of stroke and death is less than 3 percent for the surgeon and center.

16. 5-year-old with blunt abdominal trauma, Upper GI series showed coil spin in 2nd 3rd duodenal and high amylase, How to
manage:
A) Laparotomy and hematoma evacuation
B) Gastroenterostomy
C) Bowel rest
D) CT guided needle for hematoma extraction
Answer: C
17. 18 years old boy play basketball he came with abdominal pain without any injury in match physical exam was tenderness in
Periumbilical what you next:
A. Chest x-ray
B. Abdominal CT
C. 24 recheck
D. Kidney US

ANSWER: B
- In Acute abdomen: symptoms that suggest surgical or emergent conditions include fever, protracted vomiting, syncope or
presyncope, and evidence of gastrointestinal tract blood loss.
- CT scan should be used after other investigations (CBC- Electrolytes- Amylase- Lipase- LFT- ECG)
- FIRST AID: SURGERY Clerkship

18. Presentation of DiGeorge syndrome


A. Cervical cyst
B. Ectopic thymus
C. Ectopic parathyroid
D. Fistula
Answer: B
- Typically results from a deletion in chromosome 22, which disrupts the development of the pharyngeal arches and pouches,
and may also cause neurological, immunological, endocrinological, or cognitive deficits.

8
- The classical presentation is a triad of cardiac anomalies, hypoplastic thymus, and hypocalcaemia (resulting from
parathyroid hypoplasia).
- An absent thymus or one in an aberrant location may be noted on chest radiographs and CT scans.
- Http://bestpractice.bmj.com/best-practice/monograph/947/highlights/summary.html

19. A man who is post-cholecystectomy, now complaining of unilateral parotid swelling. He has history of mumps. Facial nerve
intact, no decrease in salivation. Lab shows cloudy saliva, and saliva culture in negative.
A. Sarcoidosis Granuloma
B. Sialadenitis
C. Parotid cancer
D. Mumps
E. Sjogren syndrome

Answer is B
 Sialadenitis (inflammation of the salivary glands), or in this case bacterial parotitis occurs in postoperative patients or other
severely ill patients who become dehydrated.
 Mumps (viral parotitis) happen in unvaccinated children. Sjogren syndrome affects post-menopausal women; it causes
chronic parotitis, dry eyes, and dry mouth.
Source: http://emedicine.medscape.com/article/882461-overview

20. HIV patient presented with (symptoms of intestinal obstruction); did intestinal resection. The tumor found was white in
color, nearly encircling the wall. What is the tumor?
A. Hodgkin
B. Non-Hodgkin
C. Adenocarcinoma
D. Plasmacytoma

Answer is B
 The most common malignancies in HIV patients are Kaposi sarcoma followed by Non-Hodgkin’s lymphoma. Non-Hodgkin’s
in HIV patients is almost always extra-nodal, and one-third occurs in the GI tract.
Source: http://hivinsite.ucsf.edu/insite?Page=kb-04-01-11#S9X
Http://www.medscape.com/viewarticle/734593

21. Child after trauma in perineum region showed urine extravasation


A. Penile urethra
B. Prostate urethra
C. Ureter
D. Bladder

9
22. Women traveled 18 hrs after landing She couldn't put her feet back on the shoes The best Diagnosis test ?

A. MRI pelvis
B. CT pelvis &abdomen
C. CT abdomen
D. Compression CT

Answer: D

23. Patient after 2 weeks post MI c/o unilatral leg swelling, pale, loss of hair other leg is normal. Diagnosis?
A- acute arterial emboli
B- acute arterial thrombus
C- DVT
D- disecting artrey

Answer: A
Explanation: It is often difficult to distinguish an embolus from a thrombosis, but embolic occlusions should be suspected in patients
with the following features: 1) acute onset, where the patient is often able to accurately time the moment of the event; 2) a history
of embolism; 3) a known embolic source, such as cardiac arrhythmias; 4) no prior history of intermittent claudication; and 5) normal
pulse and Doppler examination in the unaffected limb.
Reference: http://www.medscape.com/viewarticle/431272_2

24. Dense echo, acoustic shadow:


A-Tumor
B-Uric acid stone
C-Blood clot
D-Sloughed papilla
Answer: B
Reference: https://quizlet.com/60348762/nephro-flash-cards/ question number 7
Http://radiopaedia.org/articles/urolithiasis http://emedicine.medscape.com/article/983759-workup#c5

25. Patient with swelling in the thyroid, She is euthyroid what will you do ?
A. Thyroid lobectomy
B. FNA
C. Biopsy
D. Excisional biopsy

Answer: B

If the serum TSH concentration is normal or elevated, and the nodule meets criteria for sampling, then FNA biopsy is indicated.

Http://www.uptodate.com/contents/diagnostic-approach-to-and-treatment-of-thyroid-
nodules?Source=outline_link&view=text&anchor=H11#H11

10
26. Patient was skating on the stair, then he was falling with a saddle trauma, he developed penile, scrotal & lower abdominal
swelling with hematuria, which organ was affected?
A. Bladder
B. Urethra
C. Testes
D. Prostate

Answer: B
In this injury a male patient falls forcefully with the legs apart on something hard. Some examples of this are falling upon the
crossbar of a bicycle, a railing, or being bucked onto the horn of a saddle. In this injury, the urethra is squeezed against the underside
of the pubic bone and forcefully divided in two

Http://healthcare.utah.edu/urology/conditions/traumatic-urologic-injuries.php

27. Patient presented with cervical mass not in the midline and move with swallowing, what is diagnosis:
A. Thyroid nodule
B. Thyroglossal cyst
C. Brachial plexus
D. Cervical lymph nodes

Answer : A

Assessment of the mass with swallowing is important as movement from swallowing suggests a lesion in the thyroid gland or a
thyroglossal cyst . The latter also elevates with tongue protrusion .

Http://www.ajol.info/index.php/cme/article/download/43974/27491

28. Patient presented with cervical swelling associated with weight loss, night sweating and fever but no respiratory or
neurological symptoms, he was diagnosed as non hodgkin's lymphoma. On CT scan, there are paraaortic and inguinal lymph
nodes enlargement
what is the stage:
A. IB
B. IIB
C. IIIBS
D. IIIIBD
Answer :C

http://www.uptodate.com/contents/image?Imagekey=HEME%2F97479&topickey=HEME%2F4696&source=see_link
Http://www.cancer.org/cancer/non-hodgkinlymphoma/detailedguide/non-hodgkin-lymphoma-staging

29. Patient came with symptoms of increase IC pressure (confusion , nausea , vomiting), before doing Head CT what is the cranial
nerve examination will support the DX.?
A. Optic
B. Oculomotor
C. Facial
D. Vestibulococlar
Answer: A

papilledema is often used broadly to denote a swollen optic nerve head, the term papilledema should be reserved for optic disc
swelling that is due to raised intracranial pressure.

11
Http://www.uptodate.com/contents/overview-and-differential-diagnosis-of-
papilledema?Source=outline_link&view=text&anchor=H6#H6

30. What’s the medication that could increase pain in acute cholecystitis?
A. Acetaminophen
B. Propoxyphene
C. Meperidine
D. Morphine (increases the pressure at sphincter of Oddi)
Answer: D
Http://www.meb.uni-bonn.de/dtc/primsurg/docbook/html/x3982.html
31. Lesion that is volcano-like on a 70 year old farmer’s hand?
A. Basal cell carcinoma
B. Squamous cell carcinoma
C. Melanoma
D. Leishmania
Answer: B
Keratoacanthoma is a Squamous cell carcinoma associated with sun exposure and usually present on the hand
Http://www.skinsight.com/adult/keratoacanthoma.htm

32. Theoretically if there could be a vaccine that could prevent an oncogenic disease, it would be?
A. AML
B. ALL
C. Adult t-cell lymphoma/leukemia
D. Mycosis fungoides
Answer: C

33. Thoracentesis in mid axillary line


A. 4th intercostal space
B. 5th intercostal space
C. Intercostal space 8
D. Between 7
Answer: D
According to Medscape it’s between 7-9 ribs, so I believe D is the right answer but it wasn’t written completely.
Http://emedicine.medscape.com/article/80640-overview#a3

34. X of ulcerative colitis when will do colonoscopy?


A. 1 y
B. 6 m
C. 8y after dx
D. Barem enema 1 y
Answer: C -First Aid
35. Patient has high Alkaline Phosphatase, in order to confirm the hepatic origin of this elevation the doctor ordered one more
test. What is it?

A- Lactate dehydrogenase.
B- Creatinine kinase
C- Gamma-glutamyl transpeptidase
D-Lipase

12
Answer: C
The gamma-glutamyl transferase (GGT) test may be used to determine the cause of elevated alkaline
Phosphatase (ALP).
Both ALP and GGT are elevated in disease of the bile ducts and in some liver diseases, but only ALP
Will be elevated in bone disease. Therefore, if the GGT level is normal in a person with a high ALP, the
Cause of the elevated ALP is most likely bone disease.

Reference: https://labtestsonline.org/understanding/analytes/ggt/tab/test/

36. Old patient that presented with abdominal pain from time to time starts in the left mid abdomen radiates to the back,
whenever he have the attacks, he lies down on that side and bend his body position like a baby, no vomiting, diarrhea or wt
loss what is the diagnosis:
A- Duodenal ulcer
B- Gastric ulcer
C- Chronic pancreatitis
D- Mesenteric thrombosis

Answer: C
 Pancreatitis
- Clinically, the patient experiences intermittent attacks of severe pain, often in the midabdomen or left upper
abdomen and occasionally radiating in a bandlike fashion or localized to the midback.
- Other symptoms associated with chronic pancreatitis include diarrhea and weight loss.
- During an attack, patients may assume a characteristic position in an attempt to relieve their abdominal pain
(eg, lying on the left side, flexing the spine and drawing the knees up toward the chest).
Reference: Medscape: http://emedicine.medscape.com/article/181554-clinical#b1

37. A patient was not able to move her arm above shoulder after breast CA surgery that involved axillary dissection.
What is the nerve injured?
A. Long thoracic nerve
B. Rotator cuff
C. Supraspinatus
D. Brachial plexopathy

Answer: A
We have four nerves must the surgeon be aware of during an axillary dissection:
 Long thoracic nerve
 Thoracodorsal nerve
 Medial pectoral nerve
 Lateral pectoral nerve
th
Reference: Surgical Recall, 6 edition

38. A 32 years-old alcoholic male patient is brought to the emergency department with the history of vomiting large amount of
bright red blood. Physical examination revealed splenomegaly and ascites. Which of the following is the most likely source of
bleeding?
A. Duodenal ulcer.
B. Proton pump.
C. Esophageal varices.
D. Gastric cancer.

13
Answer: C
Reference: Toronto Notes

39. 23 years old woman with cyclic bilateral nodularity in her breast since 6 months. On examination there is tender 3 cm mobile
subareolar mass on her right breast. What will you do next?
A. FNA with cytology
B. Mammogram
C. Biopsy
D. Follow up for next cycle
E. Observation

Answer: C

Reference: https://books.google.com.sa/books?Id=rkt3hwnsbp4c&pg=PT1027&dq=#v=onepage&q&f=false

40. A patient with diaphragmatic hernia and you want to cut the phrenic nerve. Where can you do that?
A. Anterior to scalenus anterior
B. Anterior to scalenus medius
C. Posterior to scalenus anterior
D. Posterior to scalenus medius

Answer: A

14
41. Which of the following suggest ductal papilloma on breast examination?
A. Blood stain on nipple
B. Hemorrhagic discharge
C. Serous discharge
D. Pus from nipple

Answer: A

42. A patient came to your clinic with inflammation, redness and swelling around the nail. You gave 1 week course of augmentin
but of no use. What should be done? (Picture was also attached)
A. Give augmentin for one more week
B. Incision and drainage
C. Warm soaks
D. Splint

Answer: B If an abscess has developed, incision and drainage must be performed. Reference:
http://emedicine.medscape.com/article/1106062-overview

43. You need to put cannula in the great saphenous vein. Where can you find
its branch?
A. In front of medial malleolus
B. In front of lateral malleolus
C. Behind medial malleolus
D. Behind lateral malleolus

Answer: A
The great saphenous vein runs anteriorly to the medial malleolus. Reference:
http://www.umed.lodz.pl/anestezja/dokumenty/injections.pdf

44. A patient presented with hard mass on the outer upper area of the breast. Which lymph node you have to examine?
A. Posterior axillary
B. Anterior axillary

15
C. Lateral axillary
D. Medial axillary

Answer: B
Anterior (pectoral) group: from the lateral quadrants of the breast. Reference:
https://www.dartmouth.edu/~humananatomy/part_2/chapter_7.html#chpt_7_lymphatic

45. A patient presented with lymph node enlargement on the horizontal line of the inguinal ligament. What is the structure that
you must examine?
A. Anal canal
B. Gluteal folds
C. Anterior medial leg
D. Anterior medial thigh

Answer: A
Superficial inguinal lymph nodes: Anal canal (below pectinate line), skin below umbilicus (except popliteal territory), scrotum.
Reference: FA USMLE step1

46. A patient with a stabbed wound to the Gluteus. Examination: The patient tilt to the unaffected side while walking. Which
nerve is affected?
A. Femoral N
B. Obturator N
C. Superior Gluteal N
D. Inferior Gluteal N
E. Peroneal N

Answer: C
Superior gluteal nerve is a nerve that originates in the pelvis and supplies the gluteus medius, the gluteus minimus (abductor
muscles), and the tensor fasciae latae muscles. (Trendelenburg gait)

47. A patient had appendectomy. The artery that supplies the appendix is a branch from which of the following?
A. Superior epigastric Artery
B. Inferior epigastric Artery
C. Superior Mesenteric Artery
D. Inferior Mesenteric Artery

Answer: C
The appendicular artery (appendiceal artery) is a terminal branch of the ileocolic artery which is a branch of Superior mesenteric
th
artery. Reference: Surgical Recall, 6 edition

16
48. Patient have appendectomy what is the most common infection will be come after operation?
A. B. Fragilis
B. Staph aureus
C. Shigella
D. Pseudomonas

Answer: A
Enterococcus faecalis, Escherichia coli and B. Fragilis are common pathogens in Surgical Site Infections after clean contaminated
surgery.
Reference:
Https://books.google.com.sa/books?Id=n9y58-950vyc&pg=PA543&lpg=PA543&dq#v=onepage&q&f=false
Http://www.cdc.gov/hicpac/SSI/table7-8-9-10-SSI.html
49. During a cholecystectomy, there was an injury to the cystic artery. The surgeon applied pressure on the free margin of the
lesser omentum. What is the name of the artery compressed in this maneuver?
A. The right gastric artery
B. Coeliac
C. Hepatic
D. Splenic
E. Gastro-epiploic/duodenal

Answer: C

17
Reference: Farquharson's Textbook of Operative General Surgery, 10th Edition and Wikipedia.

50. 30 years pt with cyclic breast pain and tenderness ..pain is relieved after the cycle..
By examination there is periaerular nodular mass 3cm ..what is invx?
A. FNA
B. mammogram followed by US ( newly added )
C. u/s
D. follow her to next period CT scan ( newly added )
Answer: U/S

The first line of investigation is imaging not cytology nor tissue biopsy, So that excludes FNA.
Because the pt is young (30 yr old) U/S is more sensitive. Because of the tissue density.
Refereence: Baily & Love

51. Chronic alcoholic wake up at 3 am with abdominal pain after a little while the pain generalized all over the abdomen and is
severely exacerbated with movement:
A. Peptic ulcer perforation
B. Cholecystitis
C. Diverticulitis
D. Pancreatitis

Answer : D
-Alcohol do NOT cause ulcers. They delay the healing of ulcers
-Alcohol is a major cause of acute and chronic pancreatitis
Refrence: https://www.pancreasfoundation.org/patient-information/acute-pancreatitis/acute-pancreatitis-diagnosis-and-
treatment/

52. 18-year-old healthy male was playing baseball and suddenly he felt abdominal pain. On examination he has para-umbilical
mass. His vital signs are: BP 100/76, RR 30, HR 100, O2 sat is 95% with 100% oxygen mask. What is your next step in the
management?
A. Abdominal US.
B. CT scan.
C. Erect Chest x-ray.
D. Reassure and send home.

Answer : C ?
Intestinal obstruction due to hernia
Erect chest xray looking for air fluid level
Reference : Toronto notes

53. If there was inferior mesenteric artery thrombosis. Which artery will not be affected!
A. Descending colon *IMA
B. Sigmoid *IMA
C. Splenic *IMA
D. Cecum *SMA
Answer: D
Referenec: http://radiopaedia.org/articles/inferior-mesenteric-artery

18
54. Bilateral breast cancer is associated with :
A. Paget disease
B. Lobular carcinoma
C. Mucinus carcinoma
D. Ductal carcinoma
Answer: B
Invasive Lobular carcinoma is the second most common cause of breast cancer. Complication of ILC in addition to invading the
breast tissue is that it can spread to the lymph nodes, the other breast and possibly to other areas of the body.
Reference: http://www.breastcancer.org/symptoms/types/ilc

55. What is the best drug given to prevent postoperative thromboembolism?


A. LMW heparine
B. UF heparine
C. Warfarin
D. Enoxaparin
Answer: D

Reference: https://www.surgeons.org/media/19373/VTE_Poster.pdf
19
56. Patient having Fecal incontinence, which level of the spinal cord is affected:
A. Above C2
B. Below C2
C. Above T12
D. Below T12

Answer: D
Pudendal nerve (s2-4) is responsible for innervation to pelvic floor muscles and external anal sphincter
Reference: http://www.nafc.org/spinal-cord/

57. patient post lap choly develope sob cough respiratory distress What's dx
A. PE
B. Stroke
C. Pneumonia
D. Atelectasis.

Answer: D
It is a common pulmonary complication in patients following thoracic and upper abdominal procedures. General anesthesia and
surgical manipulation lead to atelectasis by causing diaphragmatic dysfunction and diminished surfactant activity.
PE is one of the correct options if the surgery is prolonged like in joint replacements (so unlikely after lap chole) and usually present
5-7 days after surgery, unlike atelectasis which develop one day after surgery.
Reference:
http://emedicine.medscape.com/article/296468-overview.

58. 17 years old boy with acute appendicitis, appendectomy was done. If you take it under microscope, which of the following
will be found?
A. Neutrophils in muscularis propria.
B. Mucus filled lumen.
C. Neoplastic tumor at the tip.

Answer: A
Microscopy demonstrates neutrophil infiltrate of the mucosal and muscularis layers extending into the lumen.
Reference: http://emedicine.medscape.com/article/773895-workup#c20
Neutrophils extend into and through the wall of the appendix in a case of acute appendicitis. Clinically, the patient often presents
with right lower quadrant abdominal pain. Rebound tenderness of the right lower quadrant is often noted on physical examination,

20
as well as positive obturator or psoas sign. An elevated WBC count is usually present.
Http://library.med.utah.edu/webpath/GIHTML/GI058.html

59. mass in neck in cervicle the best intial test?


A. Laryngioscope
B. CT
C. FNA
D. Biopsy

Answer: C
Reference: https://www.med.unc.edu/surgery/education/files/articles/Neck%20Mass.pdf

60. Case of female with Hx of gallstones in the gallbladder she had 2 Attack which relived her attack by analgesia:
In the CT report: Low density 0.3 cm calculi in the gallbladder. What you will prescribe:
A. Imitadiben (lipid lowering agent )
B. Simvastarin
C. Cholydoxcoic
D. Urodoxcholic

Answer: D
Explanation: For patients who decline surgery or who are at high surgical risk (eg, because of concomitant medical disorders or
advanced age), gallbladder stones can sometimes be dissolved by ingesting bile acids orally for many months. Ursodeoxycholic acid
dissolves 80% of tiny stones < 0.5 cm in diameter within 6 mo.
Reference: http://www.merckmanuals.com/professional/hepatic-and-biliary-disorders/gallbladder-and-bile-duct-
disorders/cholelithiasis

21
61. Self-breast examination decrease breast cancer by years:
A. One year.
B. Two years.
C. Three years.
D. Four years.
Answer: ?

62. Colon Cancer stage B2 means?


A. Mets to LN
B. No LN involvement
Answer: B

A- invasion of submucosa
B1- invasion of muscularis propria
B2- invasion through wall
C- positive lymph node
D- Distant metastasis

Surgical recall

63. Most specific predictor of local recurrence in breast cancer.


A. size.
B. ln mets. lymph node number
C. estrogen receptors
D. progest receptors
Answer: B
Several clinical and histopathologic factors, such as young age and presence of ductal carcinoma in situ, are known to be predictors
for local recurrence after breast conserving therapy (BCT). After mastectomy, lymph node status and tumor size are dominant risk
factors for local recurrence.
Link: https://www.researchgate.net/publication/221884330_Predictive_Factors_for_Local_Recurrence_in_Breast_Cancer

22
64. VMA pateint hemodynamic collapse what is warning sign can present when he collapse?
A. Hypotension
B. Rised jvp
C. Deviation of trachea
D. Resistance of ventlitor

65. Post RTA in ICU present with significant blood loss hypotension now in multi organ failure what is the most organ causing
other organ failure?
A. Heart
B. Lung
C. Kidney
D. Liver

66. Scenario of pt come with fractured rub what is your next action
A. Echo and cadilogy consltant
B. X – ray
C. Picardiocynteysis more info is needed

67. Best diagnostic method for diagnosing breast cyst is:


A. US
B. Mammogram
C. CT
Answer: A

68. Patient came to u her mother and her sister diagnosed with breast cancer, came for counseling, what to do now?
A. BRCA?
B. MRI
C. PET scan

Answer: US and mammogram if present + genetic counseling

69. Male after prostatectomy through venous plexus the metastasis will go to:
A. Skull
B. Vertebral body
C. Lung
Answer: B
Prostatic venous plexus drains into the internal iliac vein which connects with the vertebral venous plexus, this is thought to be the
route of bone metastasis of prostate cancer.

70. Psychiatric patient swallowed 2 safety pins, found on duodenum what to do?
A. Immediate laparotomy
B. Admit and observe
C. Charcoal

Answer: B

23
71. Female with right upper abdomen pain and fever no jaundice.. What is the management :
A. Emergent surgery.
B. IV fluid and antibiotics.
C. Discharge
Answer: A
Definitive treatment: Surgery within 3 days.
Initial management: IV fluids and Antibiotic.
Source: Toronto Notes 2014

72. Patient with abdominal trauma, after stabilization CT was done and there is splenic injury grade 1. How you will manage?
A. Conservative in surgical ward
B. Conservative in ICU AV.
C. Immediate laparotomy AW. Splenectomy

Answer: B
The diagnosis is confirmed with CT in stable patients and with bedside (point of care) ultrasonography or exploratory laparotomy in
unstable patients.
Patients who remain stable are transferred from the ICU after 12 to 48 h of observation, depending on the severity of their other
injuries, and are discharged after becoming mobile and tolerating diet. Http://www.msdmanuals.com/professional/injuries-
poisoning/abdominal-trauma/splenic-injury

73. A patient was stabbed in his abdomen, o/e he’s vitally stable, and some of the mesentery is out. What will u do
A. Exploratory laparotomy
B. Wound exploration
C. Observe
Answer: A

By definition, an exploratory laparotomy is a laparotomy performed with the objective of obtaining information that is not available
via clinical diagnostic methods. It is usually performed in patients with acute or unexplained abdominal pain, in patients who have
sustained abdominal trauma, and occasionally for staging in patients with a malignancy.
Http://emedicine.medscape.com/article/1829835-overview
74. First degree spleen injury?

o Diagnosis
Imaging (CT or ultrasonography)

24
The diagnosis is confirmed with CT in stable patients and with bedside (point of care) ultrasonography or
exploratory laparotomy in unstable patients.
o Treatment
 Observation
 Angioembolization
 Sometimes surgical repair or splenectomy
 Hemodynamically stable patients with low-grade (I to III) blunt or penetrating splenic injuries without any evidence for other
intra-abdominal injuries, active contrast extravasation, or a blush on CT, may be initially observed safely .

- Hemodynamically unstable – Based upon ATLS principles, the hemodynamically unstable trauma patient with a positive FAST
scan or DPA/DPL requires emergent abdominal exploration to determine the source of intraperitoneal hemorrhage .

Reference: http://www.uptodate.com/contents/management-of-splenic-injury-in-the-adult-trauma-
patient?Source=outline_link&view=text&anchor=H9#H9

75. You performed a pudendal nerve block on a woman in labor, which of the following structures will be fully sensitive and not
blocked by the anesthesia?
A. Perineal body
B. Urogenital diaphragm
C. Rectum
Answer: C perineum, vulva, scrotum/vagina those are blocked.
UpToDate

76. Surgeon take graft from rectus muscle which artery should be dissected?
Other version:
(In patient post mastectomy they do for her reconstruction from the rectus muscle what is the vessels may be injured or effected)
A. Superior epigastric
B. Inferior epigastric
C. Superficial epigastric
Answer B The deep superior epigastric vessels are not used as the pedicle for the free flap because it is of smaller caliber than the
inferior vessels and a greater amount of skin can be harvested with the inferior system. Medscape

77. Child with history of comminuted clavicular # due to MVA , treatment :


A. Arm sling
B. Close reduction
C. ORIF (open reduction internal fixation)
Answer: C.
Http://www.orthobullets.com/trauma/1011/clavicle-fractures

78. Case of trauma patient unconscious e abdomen distension what initial treatment:
A. CT
B. IV fluid
C. Fast
Answer: B
In hemorrhagic shock, in trauma patients, first start with IV fluids to stabilize the patient. Kaplan surgery trauma chapter

79. Nerve supply to tensor tympani and stapedius:


A. Trigeminal and facial

25
B. Facial and auditory
C. Trigeminal and ....
Answer: A
Tensor tympani is innervated by the mandibular division of the trigeminal nerve (V), through the trigeminal ganglion. Stapedius is
innervated by the tympanic branch of the 7th nerve. Http://www.dizziness-and-balance.com/anatomy/ear/ema.html

80. Headach , pain in Rt eye ,Halos around light :


A. Hyphema
B. Digoxin intoxication
C. Acute angle Glucoma
It is because of pain and halos. They do happen in digoxin toxicity but without pain and bilateral.
Http://emedicine.medscape.com/article/798811-clinical

81. Liver biopsy :


A. Midaxillary at 6th intercostal space
B. Midaxillary at 7th intercostal space
th
C. Midaxillary at 10
Answer: B
Http://emedicine.medscape.com/article/149684-technique

82. Low intestinal obstruction, 1st symptom?


A. Constipation
B. Flatus
C. Distention

Answer: A,C
Mont Reid:
• Distal Intestinal Obstruction: Obstipation\constipation and distention may occur before feculent emesis. A cramping pain
referred to lower abdomen. Obstipation is characteristic of complete obstruction.
• Proximal Intestinal Obstruction: bilious emesis early in course, minimal distention. May still have bowel movements, pass
gas while moving bowels distal to obstruction. A cramping pain referred to periumbilical region.

26
83. Patient suffered a Road Traffic Accident came to ER after 30 mins. On examination, in the chest there's sound bilateral +
distended neck. Diagnosis?
A. Hemothorax.
B. Pneumothorax.
C. Cardiac Tamponade.

Answer: A

Toronto:
 Becks triad: Hypotension, Distant Heart Sound, Distended Neck Veins.

84. 70 years old man with back pain & urinary symptom. Lab shows PSA= 40, ALP= high, LFT= normal
A. Benign prostatic hypertrophy (BPH)
B. Prostatic cancer with metastasis
C. Renal cell cancer

Answer: B
A normal PSA value ranges from 0 to 4 micrograms/L, in BPH the PSA is slightly elevated while in late stage malignancy it’s more
than 20 micrograms/L and signs of metastasis will appear above this threshold (in this case bone metastasis)
A positive prostate biopsy is the most sensitive and specific test to differentiate between prostate cancer and BPH.
Http://bestpractice.bmj.com/best-practice/monograph/254/diagnosis/tests.html
Http://bestpractice.bmj.com/best-practice/monograph/254/treatment/step-by-step.html

85. There was injury to the head exactly to the jagular foramen ,so which of the following muscle is intact ? ‫يعني كل العضالت مصابه‬
‫ ماعدا ؟ في األسئله حقتهم هنا جايبن االكسبت لكن بطريقه‬EXCEPT ‫ويقولون الهيئه مايجيبون الـ‬

A.Staylopharygeus
B. Styloglossus
C. Sternocleidomastoid
Answer: B

86. Loss of taste sensation of the anterior 2/3 of the one side of the tongue and decreased hearing on the ear on the same side ,
so where is the level of the injury of the facial nerve ?
A. After the nerve to stapidial muscle
B. Proximal to greater petrosal nerve
C. Distal to greater petrosal nerve

Answer: C

87. Post op prophylactic against DVT?


A. LMWH
B. Unfractionated heparin
C. Warfarin

Answer: “B” if Moderate risk patient, “A\B” if high risk patient.

Toronto:

27
88. MVA unconscious patient in ER, with difficulty in ventilation with ambubag-mask what you will you do?
A. Exaggerate jaw thrust
B. More head tilt
C. Go directly for intubation
Answer: Most likely C

89. Young female noticed a mass 1-week after her menstrual cycle. It persists for three cycles; the mass is rubbery and mobile.
Diagnosis?
A. Fibroadenoma
B. Fibrocystic changes
C. Duct ectasia
Answer: A
Http://emedicine.medscape.com/article/345779-overview

90. Patient had a RTA on x-ray there was multiple forehead fractures, and there is nasal discharge, what nerve is affected?
A) Optic
B) Olfactory
C) Ophthalmic

Not sure see below

The most common anatomic sites of spontaneous cerebrospinal fluid (CSF) leaks are the areas of congenital weakness of the
anterior cranial fossa and areas related to the type of surgery performed. The lateral lamella of the cribriform plate appears to be
involved in approximately 40% of the cases, whereas a defect in the region of the fontal sinus is detected 15% of the time. The sella
turcica and sphenoid sinus are involved in 15% of the cases as well.
Common sites of injury secondary to endoscopic sinus surgery include the lateral lamella of the cribriform plate and the posterior
ethmoid roof near the anterior and medial sphenoid wall.

Cerebrospinal fluid rhinorrhea in the acute phase after trauma has been reported in as many as 39% of the patients with skull base
[104]
fractures. Patients present with a variety of symptoms depending on the acuteness of the event. In the acute phase following the
traumatic event, patients may present with epistaxis, nasal discharge, periorbital ecchymosis, chemosis, oculomotor impairment,
anosmia, motor deficit, open-head injury with CSF leakage, loss of vision, cranial nerve deficits (most frequently, first–third and
[104,118]
fifth–seventh cranial nerve injuries), meningitis, and pneumocephalus. In the chronic phase, patients may present with
[59] [23]
recurrent meningitis, intermittent nasal discharge, headaches, salty or sweet taste in the retropharyngeal space, hyposmia, and
brain abscess.
91. Young boy presented with diarrhea sometimes bloody, Weight loss, arthritis, anemia the diagnosis is:
A. Crohns
B. UC
C. Celiac
28
Answer: b
Rectal bleeding is the hallmark feature of UC, however diarrhea may be present if more than the rectum is involved. Extra-intestinal
manifestation: Erythema nodosum, Peripheral arthritis Ankylosing spondyliti – Toronto notes 2015

92. Which of the following types has the highest risk for developing strangulated hernia?
A) Direct inguinal
B) Indirect inguinal
C) Femoral
Answer: C
The rates of strangulation were 22 and 45 percent at 3 and 21 months, respectively, for femoral hernias, compared with 2.8 and
4.5 percent for inguinal hernias. Http://www.uptodate.com/contents/overview-of-treatment-for-inguinal-and-femoral-hernia-in-adults

93. Pt. With high intestinal obstruction, what will be the symptom?
A) Absolute constipation
B) Diarrhea
C) Change of habit
Answer: should be vomiting, if it's not in options it maybe B
Nausea, vomiting - Associated more with proximal obstructions
Diarrhea - An early finding. Constipation - A late finding, as evidenced by the absence of flatus or bowel movements.
Http://emedicine.medscape.com/article/774140-clinical
Small gut obstruction produces effects which differ according to the level at which it occurs. The higher the obstruction the
earlier and the worse the patient's vomiting, and the greater the threat to his life from electrolyte imbalance but the less his
distension. Conversely, the lower the obstruction the greater his distension, the greater his pain, and the later he starts to
vomit. Http://www.meb.uni-bonn.de/dtc/primsurg/docbook/html/x3146.html

94. Pt. Comes with sign of intestinal obs and decrease pr may be absent of bowel sound Dx?
A) Pancreatitis
B) Cholecystitis
C) Myocardial infarction
Answer: A
Pancreatitis: Abdominal tenderness, muscular guarding (68%), and distention (65%) are observed in most patients; bowel
sounds are often diminished or absent because of gastric and transverse colonic ileus.

95. Patient present with hypotension his phosphate in normal level after one day her phosphate level decrease. What organ
damage:
A- liver
B- kidney
C- lung
Answer: B

96. During adrenalectomy you injured structure passing anterior to it:


A. Inferior vena cava
B. Duodenum
C. Rectum

ANSWER: A
- Anterior to right Supra renal Gland is: part of the right lobe of the liver + Inferior vena cava
- Anterior to the Left supra renal gland is: part of the stomach and Pancreas
- Part of diaphragm is posterior to both glands
- GRAY'S ANATOMY FOR STUDENTS

29
97. Women with mastitis:
A. Stop breast feeding
B. Clean nipple with alcohol
C. Surgical drainage

Answer: question and choices are not complete


- Mastitis is a cellulitis of the periglandular breast tissue
- Caused by nipple trauma from breastfeeding coupled with the introduction of bacteria, usually Staphylococcus aureus, from
the infant’s pharynx into the nipple ducts.
- Symptoms often begin 2–4 weeks postpartum
- Symptoms are usually unilateral and include the following: Breast tenderness. \ Palpable mass \ Erythema, edema, warmth,
and possible purulent nipple drainage.
- Treatment:
1- Continued breastfeeding to prevent the accumulation of infected material
2- Oral antibiotics effective against penicillin-resistant staphylococci (dicloxacillin, cephalexin, amoxicillin/clavulanate,
azithromycin, clindamycin).
3- If not clinical improvement within 48–72 hours, evaluate with breast ultrasonography to assess for abscess. If present, treat
with incision and drainage.

- First Aid Step 2CK


98. Abdominal solid mass (renal I guess but not sure) confirmed by:
A. CT
B. MRI
C. US

ANSWER: A
- Both US and CT usually demonstrate the organ from which a mass arises, But CT is more accurate than US
- MRI may be used to evaluate complex lesions not definitely characterized by US or CT. MRI excels in specifically
characterizing fat, protein, fluid, blood products, vascularized tissue, and metal.
- Https://acsearch.acr.org/docs/69473/Narrative/

99. Patient male 50s i think with chronic epigastric abd pain , last wk lost 7 kg +ve stool occult blood
A. Chronic pancreatitis
B. Chronic cholecystitis
C. Chronic gastritis
Answer: i think due to significant wt loss should think about malignencey .. But the case is not clear .

100.Male with right lower hypochondrium pain , in surgical exploration they found inflamed appendix , what is the congenital
malformation responsible for this :
A- reverse rotation
B- non rotation
C- behind liver ..

Answer: B
Explanation: Intestinal malrotation, also known as intestinal nonrotation or incomplete rotation, refers to any variation in this
rotation and fixation of the GI tract during development. Appendectomy is performed during operation for malrotation because the
normal anatomical placement of the appendix is disrupted.
Reference: http://emedicine.medscape.com/article/930313-overview

30
101.Elderly with acute urine retention:
A- transurethral prostatectomy
B- partial prostatectomy
C- foley catheter and urine culture
Answer: C
Explanation: The overriding therapeutic goal is reestablishment of urinary flow. Before specific therapy for obstruction is initiated,
the life-threatening complications of obstructive uropathy must be investigated and treatment started. Once urinary obstruction is
under consideration, a transurethral bladder catheter should be placed
Reference: http://emedicine.medscape.com/article/778456-overview

102.Old patient c/o increase urination at night and weak urine stream. Lab result show PSA 1 ? What is your diagnosis?
A- Prostitis
B- BPH
C- prostatic cancer

Answer: B

Explanation: usually fever, lower back or suprapubic pain, or tender rectal exam is more consistent with prostitis. In BPH, Symptoms
include frequency, urgency, and nocturia due to incomplete emptying, In addition to hesitancy and intermittency. If the PSA level is
> 4 ng/ml, then a transrectal biopsy is recommended to exclude prostate cancer.
Reference: http://www.merckmanuals.com/professional/genitourinary-disorders/benign-prostate-disease/benign-prostatic-
hyperplasia-bph
103.Male patient came to ER with stab wound and hypotension what is your next step?
A. Fresh frozen plasm
B. IV ringer lactate
C. Packed RBC

Answer: B
Explanation: According to ATLS guidelines, trauma patients are managed by following ABC (Airway, breathing & circulation) in
primary survey. In Circulation, apply direct pressure to any bleeding wound, establish 2 16 gauge IV line in each antecubital fossa
then administer isotonic fluid either normal saline or ringers lactate with 3:1 ratio (fluid to blood loss ratio) & start with 2L initially.
Only if the patient remains unstable packed RBC is considered, and the ratio is 1:1.
Reference: First Aid Step 2Ck.

104.Characteristic of perforated duodenal ulcer?


A. Mid-epigastric pain
B. Steatorrhea
C. Melena

Answer: A
• Symptoms include sudden, severe abdominal pain, a rapid heartbeat, and a low body temperature. Pain may radiate to one
or both shoulders, and the abdomen may become rigid. The abdominal pain is usually sudden, sometimes producing
collapse or syncope. Localization is usually epigastric, but it quickly becomes generalized.
Reference: Uptodate

105.If a breast lump felt by self-exam for how many years it's there?
A. 1
B. 2
C. 4

Answer: C
31
Source: No clear source. References generally indicate that it is between 2 and 5 years. Mammograms can detect breast CA 2 years
before being felt as a lump.
106.Which vitamin can prevent or reduce risk of colorectal cancer? 3 times
A. Folic acid
B. Vit.D
C. Vit.E
Answer: B
Reference: UpToDate
Http://www.uptodate.com/contents/colorectal-cancer-epidemiology-risk-factors-and-protective-
factors?Source=outline_link&view=text&anchor=H19#H19
107.decrease colorectal cancer
A. Vitamin
B. Vitamin E
C. Folic acid
Answer:
- Folic acid decrease the risk of colorectal cancer .
- Vit D act as inhibtor of colorectal cancer by influence the intiation and progrestion .
-also, "higher intake of vitamin B6 was associated with lower risk of colorectal cancer"

Reference:http://www.hopkinscoloncancercenter.org/CMS/CMS_Page.aspx?Currentudv=59&CMS_Page_ID=1293D614-71B1-4A5A-
8CFD-7BF8760295FA

Vit B6
Medscape
Http://www.medscape.org/viewarticle/506337
uptodate
http://www.uptodate.com/contents/colorectal-cancer-epidemiology-risk-factors-and-protective-
factors?Source=outline_link&view=text&anchor=H20#H20

108.A patient with achalasia. Repeated attempt of balloon expansion relapses again. What is the management?
A. Stent insertion
B. Myotomy
C. Nasogastric tube
Answer: B
Reference: Surgical Recall, 6th edition
109.24 years old girl came to you with a painless mobile breast mas that does not change with menstrual cycle. It started
increasing since 3 months. Now it’s 3 cm in size. What is the most likely cause?
A. Fibroadenoma
B. Fat tissue
C. Cyst

Answer: A
➢Fibrocystic: changes with menstrual cycles (hormones), lumps in both breasts
➢Fibroadenomas. These are the most common benign tumors. They are solid, round, rubbery lumps that move freely. They’re
usually painless.
➢Simple cysts: Simple cysts are fluid-filled sacs that usually happen in both breasts. There can be one or many. They can vary in
size. Tenderness and size often change with menstrual cycle.
➢Intraductal papillomas: These are small, wart-like growths in the lining of the mammary duct near the nipple. They usually affect
women who are 45 to 50. They can cause bleeding from the nipple.

32
110.45 years old female presented with neck swelling and anxiety. On examination the swelling is moving with swallowing and
lateral to midline. What is the most likely diagnosis?
A. Thyroglossal cyst
B. Branchial cyst
C. Thyroid nodule
Answer: C
th
Reference: Surgical Recall, 6 edition

111.A patient received clindamycin before surgery. 3rd day post-op patient developed watery diarrhea. What test will you order?
A. Stool ova and parasite
B. EIA for C.difficile
C. Viral

Answer: B The primary risk factor for C difficile colitis is previous exposure to antibiotics; the most commonly implicated agents
include the cephalosporins (especially second and third generation), the fluoroquinolones, ampicillin/amoxicillin, and clindamycin
Reference: http://emedicine.medscape.com/article/186458-overview#a5

112.(long scenario) lactating women 10 days after delivery complaining of fever and rigors. On examination: tender left breast
and nodules in upper outer area (investigations result included). What is the most likely diagnosis?
A. Postpartum sepsis.
B. Breast abscess.
C. Inflammatory breast cancer.
Answer: B
Postpartum fever is defined as a temperature greater than 38.0°C on any 2 of the first 10 days following delivery exclusive of the first
24 hours. (which is not clearly met here).
Reference: http://emedicine.medscape.com/article/796892-overview#showall

113.Patient have blunt in right 4 intercostal space. Which of the following is most likely affect affected?
A. Upper lobe of lung
B. Lower lobe of lung
C. Horizontal

Answer: C
The horizontal fissure arise from the right oblique fissure and follow the fourth intercostal space from the sternum until it meets the
oblique fissure as it crosses right 5th rib. Reference: http://radiopaedia.org/articles/horizontal-fissure

114.(long scenario) man with solid thyroid nodule, what is the most appropriate thing to do?
A. Incisional biopsy.
B. Excisional biopsy.
C. Fine needle aspiration.

Answer: C
Fine-needle aspiration biopsy is used for definitive diagnosis. FNAB has attracted much attention in the adult population in the
evaluation of thyroid nodules. Reference: http://emedicine.medscape.com/article/924550-workup#c5

115.indirect hernia related to spermatic cord ?


A. Anterio medial
B. Posterio medial
C. Something another something
Answer: A

33
http://fitsweb.uchc.edu/student/selectives/Luzietti/hernia_inguinal_indirect.htm

116.Patient came with history of upper abdominal pain, the investigation showed high amylase levels, your diagnosis will be:

A. Acute pancreatitis.
B. Gastric ulcer.
C. Acute cholecystitis.

Answer is: A
Amylase is an enzyme produced by your pancreas and salivary glands.
The pancreas can sometimes become damaged or inflamed, which causes it to produce too much or too little amylase. An abnormal
amount of amylase in your body may be a sign of a pancreatic disorder.
Reference: http://www.healthline.com/health/amylase-blood
http://bestpractice.bmj.com/best-practice/monograph/66/diagnosis/tests.html

117.which of the following has strong association for colorectal cancer:

A. Familial Adenomatous Polyposis (FAP)


B. Ulcerative colitis
C. Crohn’s disease

Answer is: A
More than 95% of people with FAP will have multiple colon polyps by age 35. If FAP is not recognized and treated, there is almost a
100% chance that a person will develop colorectal cancer.
Reference: http://www.cancer.net/cancer-types/familial-adenomatous-polyposis
118.Patient has deep ulcer with erythema on the sole of the foot, he is a known case of uncontrolled DM. What is the treatment?
A. Give oral antibiotics
B. Admit the patient for debridement and deep culture
C. Perform superficial culture

Answer: B
- Adequate debridement, proper local wound care (debridement and dressings), redistribution of pressure on the ulcer by
mechanical off-loading, and control of infection and ischemia (when present) are important components of treatment for all ulcers,
regardless of stage and depth.

- For most patients with diabetic foot ulcers, we suggest surgical (sharp) debridement rather than another method .
Reference: http://www.uptodate.com/contents/management-of-diabetic-foot-
ulcers?Source=outline_link&view=text&anchor=H28#H28

119.Patient did gastrectomy and need to take vitamin B12 for life ,which cells are responsible: (Repeated in Basic Science)
A. Goblet cells
B. Chief cells
C) Parietal cells
Answer: C
Parietal cells are responsible for the secretion of intrinsic factor. Under normal circumstances intrinsic factor binds to vitamin B12
and assists with the absorption of this vitamin in the lower portion of the small bowel. When vitamin B12 is poorly absorbed, anemia
and, in some cases, poor nerve function can occur.
Reference: http://my.clevelandclinic.org/health/diseases_conditions/post-gastrectomy-syndrome-overview

34
120.Patient underwent supradrelenectomy, you are worried not to injure which organ:
A. Inferior vena cava
B. Kidney
C. Appendix
Answer: B
On the left side, the spleen is the most commonly, The tail of the pancreas and the blood vessels supplying the kidney are also at
risk.
On the right side, the liver and duodenum (first part of the small intestine) are also at risk of injury.
Http://m.endocrinediseases.org/adrenal/surgery_complications.shtml

121.Case of RLQ pain and mass, what is Tx :


A. Conservative.
B. Surgery.
C. Antibiotics.
Answer: C , Inflamed Appendicular mass is either a phlegmon or an abscess , both are treated non surgically by antibiotics ( in some
abscess cases by US guided drainage ).

122.gastrectomy which one will be defecient


A. Lipase
B. Pepsin
C. Trypsin
Answer: B
Reference: http://rfwdata.net/USMLEQBANK/block17questions.html

123.painful axillary lump with tender and erythematous with black head papule and large pores?
A. Local antibiotic
B. Oral antibiotic
C. Surgery

Answer: C
It is most probably epidermoid cyst , if not infected it will resolve spontaneously but recurrence in high , if fluctuating I and D is
indicated . It's removed via simple excision or incision with removal of the cyst and cyst wall though the surgical defect.
Reference ; up to date, medscape
http://www.uptodate.com/contents/overview-of-benign-lesions-of-the-
skin?Source=outline_link&view=text&anchor=H1101420445#H1101420445
Http://emedicine.medscape.com/article/1061582-treatment#d8

124.Single thyroid nodule iodine sensitive. Best management?


A. Lobectomy
B. Radio iodine therapy
C. Anti thyroid drug
Answer: B
Hot nodules secrete thyroid and should be ablated using radioactive iodine or surgery

35
Reference :american family physician
125.Pt co of infected wound ulcer ,intact pulse ,no improve by AB & pt known uncontrolled DM2 •
A. Amputation
B. Antibiotics
C. Surgical debridement

Answer: B
A good clinical response for mild to moderate infections can be expected in 80%–90% of appropriately treated patients [10, 50] and,
for deeper or more extensive infections, in 50%–60% [64, 86]. When infection involves deep soft-tissue structures or bone, more
thorough debridement is usually needed. Bone resections or partial amputations are required in about two-thirds of this patient
group. Most of these amputations can be foot sparing, and long-term control of infection is achieved in >80% of cases. Infection
recurs in 20%–30% of patients, many of whom have underlying osteomyelitis. Factors that predict healing include the absence of
exposed bone, a palpable popliteal pulse, toe pressure of >45 mm Hg or an ankle pressure of >80 mm Hg, and a peripheral WBC
count of <12,000/mm3 [19]. The presence of edema or atherosclerotic cardiovascular disease increases the likelihood of
amputation. Amputation may be more often required for patients with combined soft-tissue and bone infection than for patients
with either type of infection alone [86]. Patients who have had one infection are at substantial risk of having another within a few
years; thus, educating them about prevention techniques and about prompt consultation when foot problems occur is critical.
Reference: http://cid.oxfordjournals.org/content/39/Supplement_2/S104.full

126.case head trauma on parietal lobe subdural hematoma which artery is injured ?
A. Superficial temporal .
B. Mid cerebral
C. Rt.cerebral .......

Answer: B

The cortical branches of the MCA supply the lateral surface of the hemisphere, except for the medial part of the frontal and the
parietal lobe (anterior cerebral artery), and the inferior part of the temporal lobe (posterior cerebral artery)
http://www.radiologyassistant.nl/en/p484b8328cb6b2/brain-ischemia-vascular-territories.html
36
127.pt noticed lump for three months the mass freely mobile no discharge not related to menstrual?
A. Cystic
B. Fibroadenoma
C. Ductal papilloma

Answer: B
Reference: http://www.mayoclinic.org/diseases-conditions/fibroadenoma/basics/definition/con-20032223

128.old patient presented with congested neck veins bilateral upper limb swelling tumor in the right upper lung which part it's
compressing?
A. Superior mediastinal
B. Anterior mediastinal
C. Posterior mediastinal
Answer: A
This patient has superior vena cava (SVC) syndrome due to lung mass compression
The SVC is located in the anterior right superior mediastinum
Reference: http://www.ncbi.nlm.nih.gov/pmc/articles/PMC4093359/

129.Best prognostic indicator for breast cancer?


A. Estrogen receptor
B. Stage and grade
C. Involvement of axillary LN
Answer: C
The most significant prognostic indicator for patients with early-stage breast cancer is the presence or absence of axillary lymph
node involvement.
http://theoncologist.alphamedpress.org/content/9/6/606.full

130.Child with history of severe pain in testis, no history of trauma what is the best next step?
A. Surgery consultation
B. Urine culture and analysis
C. Analgesics and ice

Answer: b (not sure)


Reference : http://www.aafp.org/afp/1999/0215/p817.html

37
131.Newborn with tender bilateral scrotal swelling what is the next investigation :
A. Illumination
B. Wait till patient cough or cry
C. Watch voiding

Answer: ?

132.Best imaging to visualize breast mass:


A) Mammogram
B) US
C) CT
Answer: A
I looked it up and got confused but it is either Mammogram or breast MRI. According to this article which reviewed the literature, it
is mammogram.
Reference: http://citeseerx.ist.psu.edu/viewdoc/download?Doi=10.1.1.409.2944&rep=rep1&type=pdf (last paragraph in the
conclusion part)

133.Infant periumbilical hernia, what you will do:


A. Put plastic in mid abdomen
B. Reassurance
C. Do hernia repair before start school
answer: b
Umbilical hernias: Most umbilical hernias do not require surgical repair because they close by their own. Surgery is indicated when it
doesn’t close by the age of 5 or when it’s painful.
Http://emedicine.medscape.com/article/932680-treatment
Http://www.mayoclinic.org/diseases-conditions/umbilical-hernia/basics/treatment/con-20025630

134.48 yrs women, without any family history of Breast CA had Normal mammogram when to repeat it?
A. 2 yrs
B. 3 yrs
C. 5 yrs
Answer : ?
Annual screening for some women annual screening for some women (eg, premenopausal)
http://www.uptodate.com/contents/screening-for-breast-cancer-strategies-and-
recommendations?Source=outline_link&view=text&anchor=H79986#H79986

Women older than 45 with average risk should get mammogram every year!!
Http://www.cancer.org/cancer/breastcancer/moreinformation/breastcancerearlydetection/breast-cancer-early-detection-acs-recs

135.Testicular cancer with high AFP and BHCG


A. Seminoma (high beta-hcg)
B. Embryonal (high AFP+ beta-hcg )
C. Teratoma
Answer: B
Elevations in the serum concentration of AFP or beta-hcg cannot be attributed to teratomatous elements.
AFP: Elevation of serum AFP is seen in 40% to 60% of men with nonseminomas. Seminomas do not produce AFP.

38
Beta-hcg: Elevation of the beta subunit of hcg is found in approximately 14% of the patients with stage I pure seminoma
Approximately 40% to 60% of men with nonseminomas have an elevated serum beta-hcg.
Http://www.cancer.gov/types/testicular/hp/testicular-treatment-pdq
Http://www.uptodate.com/contents/anatomy-and-pathology-of-testicular-
tumors?Source=outline_link&view=text&anchor=H11#H11

136.Baby bout by his mother with abdominal distention, bad smell vomiting, history of delayed passege of muconime “after 3
days after using enema’ rectal examination showed empty rectum (typical history of hirschsprung's disease with pic of
abdominal x ray showing multiple air-fluid levels). And the doctors start to decompress his stomach. What is the definitive
treatment?
A. Leveling Colostomy
B. Total colectomy
C. Antibiotic
Answer: abdominoperineal pull-through in two or three stages, in which patients initially underwent a diverting colostomy
http://www.uptodate.com/contents/congenital-aganglionic-megacolon-hirschsprung-
disease?Source=outline_link&view=text&anchor=H13#H13

137.A woman has lichen sclerosis which cancer she will get:
A. Squamous cell carcinoma
B. Adenocarcinoma carcinoma
C. Adenosquamous carcinoma

Answer : A
Lichen sclerosus of anogenital sites is associated with an increased risk of vulval, penile or anal cancer (squamous cell carcinoma,
SCC)
Http://www.dermnetnz.org/immune/lichen-sclerosus.html

138.Question about bladder cancer after they removed what you will do next
A. Intravesical medication
B. Observe and serial assessment
C. And 2 another choices regarding treatment
Answer: ? Depend on stage

139.Surgery is contraindicated for which lung CA?


A. SCC
B. SCL
C. Adenocarcinoma
Answer: ?
I think B refers to small cell lung cancer (SCLC) which is the right answer.

140.Para follicular cell neoplasm? Which tumor?


A. Papilary thyroid cancer
B. Follicular thyriod ca
C. Others
Answer: Medullary ca
Medullary thyroid cancer originates from the parafollicular cells (also called C cells) of the thyroid.
Http://www.endocrineweb.com/conditions/thyroid-cancer/thyroid-cancer-medullary-cancer

141.Resected end of small bowel and beginning of large colon >> deficiency in?
A. Fa

39
B. B12✔
C. Uc-..

Answer: B
Vitamin B12 is readily absorbed in the last part of the small intestine (ileum)
Http://www.merckmanuals.com/home/disorders-of-nutrition/vitamins/vitamin-b-12

142.Patient came with loose stools, history of loose stools before ,, Its watery like with mucous not containing blood ( forget the
other details ) what is you diagnosis:
A. IBS
B. Crohn’s disease
C. Ulcerative colitis
Answer: A – Kaplan

143.And half years old uncircumcised boy, toilet Trained , came with his parent with UTI
What is the best method to obtain urine culture?
A. Catheter
B. Clean catch
C. Suprapubic tap
Answer: B

A midstream, clean-catch specimen may be obtained from children who have urinary control (toilet trained). In the infant or child
unable to void on request, the specimen for culture should be obtained by suprapubic aspiration or urethral catheterization.

144.Old patient in 60s of age smoker presented with history of sore throat hoarseness of voices for 3 month with enlarged
cervical lymph nodes: on examination the was large mass in the throat ( the history was highly suggestive of larynx Ca ) what
is the most probably your action :
A. Saline water gargles
B. Analgesia and antibiotic
C. Surgery and radiotherapy
Answer: C

145.For LP you will insert the needle in:


A. L1- L2
B. L3 - L4
C. L5 - S1
Answer: B, L3-L4

146.Female with breast tender nodule 3*4 in the upper outer quadrant , what is the Next step?
A. FNA and cytology
B. Follow up
C. Reassurance
Answer: A - Surgical recall

147.Patient presented with dysphasia, he has hoarseness and excessive salivation and feels hbat there is a lump in his throat,
what is the dx?
A. Achalisia
B. Diffuse esophageal spasm
C. Cricopharengeal dysfunction
Answer:

40
Most probable esophagus ca
Http://emedicine.medscape.com/article/277930-clinical

148.Obstruction: abdominal pain and vomiting, HD stable, on x-ray dilated loops and air in rectum, what will you do?
A. Rectal decompression and IV antibiotics
B. Nasogastric tube and IV NS/Antibiotic
C. Nasogastric tube and …
D. (last option I think was observe)
- (when you have air/gas in rectum + dilated loops > think paralytic ileus)
Answer: c if (NGT and IV saline)
most cases of postoperative ileus resolve with watchful waiting and supportive treatment. Patients should receive intravenous
hydration. For patients with vomiting and distention, use of a nasogastric tube provides symptomatic relief; however, no studies in
the literature support the use of nasogastric tubes to facilitate resolution of ileus. Long intestinal tubes have no benefit over
nasogastric tubes
Http://emedicine.medscape.com/article/2242141-treatment#d6

149.Case of diffuse abdominal pain, X-ray show dilated loops ?


A. Acute pancreatitis ?
B. Acute cholecystitis
C. Perforated duodenal ulcer
Answer: Acute pancreatitis may cause paralytic ileus.????
The most commonly recognized radiologic signs associated with acute pancreatitis include the following: 1) Air in the duodenal C-
loop, 2) The sentinel loop sign, which represents a focal dilated proximal jejunal loop in the left upper quadrant, and 3) The colon
cutoff sign, which represents distention of the colon to the transverse colon with a paucity of gas distal to the splenic flexure
Medscape http://emedicine.medscape.com/article/371613-overview#a2

150.Swilling test:
A. Torsion test
B. Cancer test
C. Orcico epidermitis
Answer:?

151.Lady with a mass in left upper quadrant of the breast, aspiration was yellow fluid without masses, Dx?
A. Phyllodem
B. ormal tissue variant
C. ..."no simple cyst"

152.A case of painful nodules in groin area and pain relieved after punching and discharge came out of it:
A. Hidradenitis suppurativa.
B. Furunculus
Answer: A http://dermnetnz.org/acne/hidradenitis-suppurativa.html
153.At inguinal canal base, what artery you can find?
A. Iliac
B. Femoral

Answer : if they ask about artery at the base i think it’s femoral , but if the ask about which artery pass through the canal the
answer will be 3 arteries: artery to vas deferens (or ductus deferens), testicular artery, cremasteric artery. But if they just asked

41
about the base of the canal the answer will be >> The Canal Boundaries :The anterior wall is formed by the aponeurosis of the
external oblique, and reinforced by the internal oblique muscle laterally.The posterior wall is formed by the transversalis fascia.The
roof is formed by the transversalis fascia, internal oblique and transversus abdominis.The floor is formed by the inguinal ligament
(a ‘rolled up’ portion of the external oblique aponeurosis) and thickened medially by the lacunar ligament
Http://teachmeanatomy.info/abdomen/areas/the-inguinal-canal/

154.Bacteroides in an abdomen gunshot wound, what antibiotics are most appropriate?


A. Clindamycin
B. Others options
Answer:
No specific agent is recommended for penetrating abdominal traumas, but it may be a single agent with beta-lactam coverage or
combination therapy with an aminoglycoside and clindamycin or metronidazole. Antibiotics include cefotetan, metronidazole
hydrochloride, gentamicin sulfate, vancomycin hydrochloride, & ampicillin sodium-sulbactam sodium.
Http://emedicine.medscape.com/article/2036859-medication#showall

155.40 years old female with 3 months mass 2.5 cm firm, mammogram and US normal what to do ?
A. Follow up after two cycles
B. Obtain biopsy
Answer: B
Source: answered by GS resident.

156.39 y.old p3+0 complete her family , history of left ovary and endometrial ablation ,complain of dysmenorrhea,..,... Now
another ovarian cyst 6-7cm Managment:
A. Hysterectomy+oophorectomy
B. Remove cyst+ablation affected endometrial
Answer

157.patient presents with signs of infectious mononucleosis with enlarged lymph nodes, when you were palpating the spleen he
became cold and pale. what to do next?
A. urgent gastroscope
B. fluids, urgent ct and abx

Answer: B
This seems like the presentation of splenic rupture, the patient should be stabilized and CT can be done for grading in the stable
patient or FAST scan in the unstable patient.
http://emedicine.medscape.com/article/432823-treatment

158.Why we use CT scan in trauma patient?


A. Can detect retroperitoneum hemorrhage.
B. You can see the hemorrhage with contrast

Answer: A
Medscape:
 CT scans of the abdomen and pelvis usually are performed together, using both IV and oral contrast. Use this study to
identify injuries to abdominal and pelvic organs and to identify bleeding in the retroperitoneum and pelvis

159.Present to ER with severe headache and LP showed blood :


A. Ruptured berry aneurysm,
B. Epidural hematoma
Answer: A

42
160.12 years old football player presented to the ER with nausea and vomiting and tender scrotal swelling , what is the
management ?
A. Urology referral
B. Immediate surgery
Answer: B

161.Patient has got stab wound at the middle of posterior border of sternocleidomastoid muscle , afterthat the patient cannot
elevate his arm above his head , what is the injured nerve ?
A. Axillary nerve
B. Long thoracic nerve

Answer: A

162.Patient with enlargement of the urethra , what you will do ?


A. Annually Renal function test
B. Annually Prostatic specific antigen (PSA)

Answer: B
Patients with BPH who are on watchful waiting or are treated with medical or surgical therapy should undergo monitoring of
their clinical symptoms with intermittent administration of the International Prostate Symptom Score (IPSS). This is an accurate
and reliable way to monitor improvement or progression of symptoms in patients with BPH.
Patients between 40 and 75 years of age may undergo (after discussing potential risks and benefits with their physician) annual
screening for prostate cancer with a digital rectal examination and a serum PSA
Http://bestpractice.bmj.com/best-practice/monograph/208/follow-up/recommendations.html
163.41-year-old patient with recurrent episodes of acute cholecystitis. Ultrasound showed small stones, you will give her?
A. Ezetimibe
B. Fibramait << No idea what this is. Maybe the mean fibrinate!

Answer: Not sure


Explanation: Management of acute cholecystitis starts with supportive care; that is hospital admission, IV hydration and correction
of any electrolyte imbalance. Analgesia often by opioid is administered, however, ketorolac (NSAID) may be enough to relieved
biliary colic. Usually, acute cholecystitis is accompanied with an infection most commonly E.coli for which antibiotics should be
prescribed. After that a surgical definitive treatment is decided. Prevention measurements include use of NSAIDS which can produce
effective analgesia for biliary colic, and may favorably alter its natural history
Http://cursoenarm.net/UPTODATE/contents/mobipreview.htm?11/26/11695#H8

164.MVA victim presented to ER with hypotension, given crystolloid his BP return to normal, and chest tube was inserted. What
to do next?
A. Re-examine the chest
B. ABG
Answer: A

165.You are palpating artery between symphysis puppies and anterior superior spine
A. Femoral artery
B. Internal iliac

ANSWER: A
- Femoral Artery is a large artery in the thigh and the main arterial supply to the lower limb
- It lies midway between the anterior superior iliac spine and the symphysis pubis.
- Https://en.wikipedia.org/wiki/Femoral_artery

43
166.Patient with RLQ pain and swelling, weight loss, colonoscopy done showed mass in RLQ, what is the diagnosis?
A. Appendix tumor
B. Cecum tumor

Answer is B
 Colonoscopy is the most accurate and versatile diagnostic test for colorectal tumors/cancers.
 Appendiceal abnormalities are infrequently seen on colonoscopy and rarely yield a diagnostic biopsy in patients with
appendiceal carcinoma.
Source: http://www.ncbi.nlm.nih.gov/pubmed/19089515

167.When u operate for aneurism


A. More 5.5 diabetic thoracic aorta
B. More 5.5 abdominal atherosclerosis
Thoracic Aortic Aneurysm: elective repair of ascending aneurysms at 5.5 cm and descending aneurysms at 6.5 cm for patients
without any familial disorders such as Marfan syndrome. Patients with Marfan syndrome or familial aneurysms should undergo
earlier repair, when the ascending aorta grows to 5.0 cm or the descending aorta grows to 6.0 cm.
Http://emedicine.medscape.com/article/424904-treatment
Abdominal Aortic Aneurysm: Most people with an aneurysm less than 4.0 cm (1.6 inches) in diameter are advised not to have
immediate surgery, but rather to follow the aneurysm over time; this is known as watchful waiting. Usually, this involves an
ultrasound examination of the abdomen every six months to three years, depending on the size of the aneurysm.On the other hand,
most patients with an asymptomatic aneurysm greater than 5.5 cm (2.2 inches) in diameter or that expands more than 0.5 cm within
a six-month period are advised to have repair.
Http://www.uptodate.com/contents/abdominal-aortic-aneurysm-beyond-the-basics

168.Abdominal pain after lifting heavy object + with defect in abdominal wall, what is the diagnosis?
A) direct inguinal hernia
B) indirect inguinal hernia
Answer: A
Direct: The direct inguinal hernia enters through a weak point in the fascia of the abdominal wall.

Indirect: An indirect inguinal hernia results from the failure of embryonic closure of the deep inguinal ring after the testicle has
passed through it.

169.Cause of pruritus ani?


A) Hemorrhoids
B) perianal abscess
Answer: Not sure - Question not clear enough.

44
Explanation: Merck manual - No age is mentioned though according to group ages the following are the most common cause of anal
pruritis.

170.Can't recall the senior very well but it was symptoms and sign of pancreatic disease that caused thrombophlebitis ?
A- acute pancreatitis
B- pancreatic tumor

Answer: B
Explanation: Migratory thrombophlebitis (ie, Trousseau sign) and venous thrombosis also occur with higher frequency in patients
with pancreatic cancer and may be the first presentation.
Reference: http://emedicine.medscape.com/article/280605-clinical#b1

171.Patient with Calcium renal stones so was given a diuretic to lower the Calcium then he developed Gout ?
A- Hydrochlorothiazide
B- Furosemide

Answer: A
Explanation: Both thiazide and loop diuretics cause hyperuricemia which may lead to
Gout. However, thiazide diuretics decrease calcium urinary excretion, while loop diuretics like furosemide increase calcium urinary
excretion. In calcium renal stones, thiazides are used due to their effect on lowering urine calcium.

45
References: http://www.uptodate.com/contents/diuretic-induced-hyperuricemia-and-gout
Http://www.msdmanuals.com/home/kidney-and-urinary-tract-disorders/stones-in-the-urinary-tract/stones-in-the-urinary-tract
172.Which of these patient peritoneal lavage is indicated?
A- severe head trauma,
B- hypotensive patient with abd distention

Answer: B
Explanation: DPL can be used to evaluate both blunt and penetrating abdominal trauma in patients who are hemodynamically
unstable or who require urgent surgical intervention for associated extra-abdominal injuries. DPL can rapidly confirm or exclude the
presence of intraperitoneal hemorrhage.
Reference: http://emedicine.medscape.com/article/82888-overview

173.Bladder cancer after they removed what you will do next:


A. Intravesical medication
B. Observe and serial assesment
C. And 2 another choices regarding treatment

174.13 Y/O child limping & pain, radiological finding destructed femur head, high WBC, diagnosis:
A) Septic arthritis of the hip
B) legg calve perthes
Answer: A
WBC is normal in legg calve perthes disease. Septic arthritis causing painful joint with fever & high WBC. Radiologic features include:
 X-rays may be normal in the very early stage of the disease
 Joint effusion may be seen
 Juxta-articular osteoporosis due to hyperaemia
 Narrowing of the joint space due cartilage destruction in the acute phase
 Destruction of the subchondral bone on both sides of a joint
 If left untreated, reactive juxta-articular sclerosis and, in severe cases, ankylosis will develop
Http://radiopaedia.org/articles/septic-arthritis

175.Child with sudden abdominal pain bloody diarrhea, management?


A) Radio decompression
B) Immediate surgery
Answer: According to the age of pt, for patient from 5 months to 3 years, most common cause of bloody stool is intussusception and
they usually respond to non-operative management by enema decompression.
Http://emedicine.medscape.com/article/930708-treatment#d1
Intestinal malrotation is suspected with the sudden onset of melena in combination with bilious emesis in a previously healthy, non
distended baby.

Bloody diarrhea and signs of obstruction suggest volvulus, intussusception, or necrotizing enterocolitis, particularly in premature
infants. Acute bloody diarrhea should be considered a medical emergency.
Http://emedicine.medscape.com/article/1955984-clinical#b1

Pediatric surgeon: The only definitive treatment for malrotation is surgical in nature
Http://emedicine.medscape.com/article/930313-treatment#d7

176.Child with yellow and cavities in his teeth, what to give?

46
A) Antiseptic mouthwash
B) Fluoride (found in tooth paste)
Answer: B

177.Case of trauma patient unconscious e abdomen distension what intial treatment :


A. CT
B. IV fluid
C. FAST
Answer: A
Resuscitation procedures will begin simultaneously with the assessment involved in the primary survey.
Reference: http://patient.info/doctor/trauma-assessment

178.Where is the common site of Meckel’s diverticulum?


A. Duodenum
B. Ileum
Answer : B

Meckel’s Diverticulum Most common remnant of vitelline duct that connects yolk sac with primitive midgut .Present most
frequently during the first 5yr of life
Reference: Toronto notes.
179.Old patient with Back pain, dysuria , frequency what is the next investigation:
A. PSA
B. ALP

Answer:A
The approach to dysuria is focused History and P/E to determine cause, any d/c (urethral, vaginal, cervical) should be sent for
gonococcus/chlamydia testing; U/A and urine C&S ± imaging of urinary tract. In this case the age and the back pain are suspicious of
malignancy thus PSA should be measured.
Reference:Toronto notes

180.Old patient developed mass in the lung (CXR was attached)


a. SCC
b. Adenocarcinoma
Answer: it depend on CXR and Hx
If it is arise in the peripheral, areas of the lungs. They also have a tendency to spread to the lymph nodes and beyond. Multiple sites
in the lungs and spreads along the preexisting alveolar walls. It may also look like pneumonia on a chest X-ray. And is very common
in non-smoking women and in the Asian population. Adenocarcinoma is more common than SCC which arise in the central chest
area in the bronchi. This type of lung cancer most often stays within the lung, spreads to lymph nodes, and grows quite large,
forming a cavity.
Http://www.webmd.com/lung-cancer/guide/lung-cancer-types

181.16 years old female. Fever and Chronic diarrhea for 10 months Post meal para umbilical pain Sometimes blood mixed with
stool?
A. Crohn’s
B. Chronic pancreatitis

Answer: A
Crohn's disease Symptoms include watery diarrhea, abdominal pain, fever and weight loss.
Http://www.webmd.com/ibd-crohns-disease/crohns-disease/crohns-disease

47
182.Intra-abdominal bleeding can lead to:
A. Hypovolemia
B. Dehydration
Answer: A

Rapid volume repletion is indicated in patients with severe hypovolemia (ie, decreased peripheral perfusion, as indicated by delayed
capillary refill and cool mottled extremities) or hypovolemic shock (ie, severe hypovolemia plus hypotension)

http://www.uptodate.com/contents/treatment-of-severe-hypovolemia-or-hypovolemic-shock-in-
adults?Source=outline_link&view=text&anchor=H1#H1

183.Patient with abscess in his forehead. Where can you find a palpable LN ?
A. Mastoid
B. Superficial parotid
Answer: Parotid gland drain most of the forehead EXCEPT middle part which is drained by submandibular, but many resources are
saying it’s pre auricular!

184.Orchitis case, pain in scrotum with urine +ve culture, what’s the treatment?
A. IV cectriaxone + Doxycycline
B. IV amoxicillin + clavulanate
Answer: A

Acute epididymitis most likely caused by enteric organisms (eg, men with a negative gram stain or nucleic acid amplification test for
N. Gonorrhoeae or men with epididymitis after urinary-tract instrumentation procedures) may be treated with fluoroquinolones
alone

Acute epididymitis most likely caused by sexually transmitted chlamydia and gonorrhea should be treated with ceftriaxone plus
doxycycline /azithromycin .

Http://www.uptodate.com/contents/evaluation-of-the-acute-scrotum-in-
adults?Source=outline_link&view=text&anchor=H2012423711#H12

185.Thyroid disease associated with papillary cancer?


A. Hashimoto
B. Riedel thyroiditis
Answer: A
Http://www.thyroid.org/patient-thyroid-information/ct-for-patients/vol-6-issue-7/vol-6-issue-7-p-6/

186.Female patient present with laughing and coughing passing out urine O/E there is swelling in labia majora ( stress
incontinence
A. Cystocele
B. Urethrocele
Answer: cystocele
Can’t find any clue to go with each one!

187.Case of painful nodules in groin area and pain relieved after punching and discharge came out of it?
A. Hidradenitis suppurativa
B. Frunculs
Answer: B
Http://www.merckmanuals.com/professional/dermatologic-disorders/bacterial-skin-infections/furuncles-and-carbuncles

48
188.Hereditary chronic pancreatitis is?
A. Autosomal dominant
B. Polygene ?
Answer: A
Ø source: http://www.ncbi.nlm.nih.gov/pmc/articles/PMC177456/

189.Patient did IVP shows filling defect then do us shows hypoecnecnty ?


A. Uric acid stone
B. Papillary
Answer: B

ANY STONE IS hyperechoic


But papillary necrosis is hypo

190.M patient went for surgery; he was given insulin and dextrose. Then developed neurological symptoms (Low Na). What is the
mechanism?
A. Water overload.
B. SIADH.
Answer A?

191.Patient did routine X Ray, they found 2*3 cm nodules that has calcified center, it was in the upper left side of lung he is
asymptomatic, what is the next step?
A. Follow up and repeat x Ray
B. Biopsy the lesion
Answer: A – Kaplan

192.patient for surgery known case of DM2 on glimepiride you will shift patient to which drug during and after surgery?
A. insulin
B. metformin

Answer: On the morning of surgery hold oral hypoglycemic agent because Sulfonylureas will increase the risk for hypoglycemia, and
metformin will increase the risk for lactic acidosis.
If the patient develop hyperglycemia give SC insulin, and correction insulin is given until the patient is eating and either can resume
oral agent.

Reference : uptodate "perioperative management of blood glucose in adult with DM"

193.16 years old female complaining of abdominal pain for 2 days. It started in the periumbilical area then it radiated to the right
lower quadrant. It is associated with anorexia. On examination, right lower mass was felt. What is the management?
A. Surgery immediately
B. Conservative
*Antibiotics weren’t mentioned in the choices.
Answer: ?
Appendectomy remains the only curative treatment of appendicitis, but management of patients with an appendiceal mass can
usually be divided into the following 3 treatment categories:
● Patients with a phlegmon or a small abscess: After intravenous (IV) antibiotic therapy, an interval appendectomy can be
performed 4-6 weeks later.
● Patients with a larger well-defined abscess: After percutaneous drainage with IV antibiotics is performed, the patient can be
discharged with the catheter in place. Interval appendectomy can be performed after the fistula is closed.
49
● Patients with a multicompartmental abscess: These patients require early surgical drainage.
Reference: Medscape.
194.A mass at the middle of the Esophagus:
A. Adenocarcinoma
B. Squamous

Answer: B
We have two types of Esophageal CA:
● Adenocarcinoma usually at the GE junction, due to chronic reflux that cause dysplasia.
● Squamous cell carcinoma in most of the esophagus
th
Reference: Surgical Recall, 6 edition

195.40 years old woman with no pain but you noticed jaundice. She has high direct bilirubin and high ALT what is your
diagnosis?
A. Gilbert’s disease
B. Biliary stone

Answer: B
- Gilbert’s disease classified as a prehepatic (associated with high indirect bilirubin)
- Biliary stone (It can be the answer to this Q, exclude other ddx first): asymptomatic (80%), Biliary colic (10 - 25%, painfull).
- Other ddx: pancreatic head CA, Cholangiocarcinoma and primary biliary cirrhosis
Choledocholithiasis (Stone in common bile duct)
- Sign and symptoms: Epigastric or RUQ pain and tenderness + Jaundice + Cholangitis OR recurrent attack of acute
PANCREATITIS
- Labs: Increased ALP, LFT and total + DIRECT bilirubin
- ERCP: Gold standard for diagnosis of CBD stones

Choleclithiasis:
- In Examination: Tenderness to palpation in the RUQ/epigastric area (Most common). In acute cholecystitis, Murphy’s sign
with inspiratory arrest when palpating the gallbladder fossa. Fever suggests a complication of cholelithiasis such as
cholecystitis, while jaundice tends to accompany cholangitis or pancreatitis.
- Labs: alkaline phosphatase and bilirubin elevations
- The initial test of choice in all patients with suspected biliary pain is abdominal ultrasound: The classic findings include
(Acoustic Shadow "headlight" + movement of the stone with patient repositioning + Gallblader thickening > 4mm +
Pericholecystic fluid)

- Http://bestpractice.bmj.com/best-practice/monograph/873/diagnosis/step-by-step.html
- FIRST AID: SURGERY Clerkship

196.Patient underwent orthopedic surgery. 2 days after the surgery he started to complain of swelling, erythema and tenderness
at the site of surgery. Which organism can cause the wound infection?
A. Clostridium perfringens
B. Other choices included different subtypes of clostridium

Answer: A
C. Perfringens is the only Clostridium species can cause wound infection (Gas gangrene; A medical emergency). Reference:
http://www.ncbi.nlm.nih.gov/books/NBK8219/

197.In appendectomy. Which artery you have to pay attention to during the procedure?
Answer: ?
A. Gonadal vessels
50
B. Iliac artery/vein
One must be cognizant of anatomical structures in the right lower quadrant, in order to avoid injuries during an appendectomy.
These include the right ureter, gonadal vessels, iliac artery/vein,psoas muscle, ileum and cecum.
Reference: http://www.eaes-eur.org/getmedia/7994a09c-344d-4c5e-81d5-4b9a75b8304b/Grantcharov_Appendectomy.pdf

198.What is the cause of skin dimpling in breast cancer?


A. Cooper ligament
B. Lactiferous duct

Answer: A. Tumor involvement of Cooper’s ligaments and subsequent traction on ligaments pull skin inward. Reference: Surgical
th
Recall, 6 edition
199.Pheochromocytoma, associated with which neoplasm?
A. Insulinoma
B. Thyroid cancer

Answer: B
- Pheochromocytoma is associated with Medullary thyroid carcinoma , hyperparathyrodism .
- Pheochromocytoma is a type of MEN IIA ( multiple endocrine neoplasia )
Reference: Step-up medicine P 185 .

200.In thyroidectomy which nerve injury lead to hoarseness of voice?


A. Recurrent laryngeal
B. Right vagus

Answer :A
Recurrent laryngeal nerve: Unilateral damage results in a hoarse voice.
Bilateral damage presents as laryngeal obstruction after surgery and can be a surgical emergency: an emergency tracheostomy may
be needed.
Refrence: http://emedicine.medscape.com/article/852184-overview#a4

201.Patient after RTA had spleen injury, he underwent surgical exploration and splenectomy. Which of the following should be
given?
A. Some kind of antibiotic.
B. Meningiococcal vaccine.

Answer: B
There are multiple vaccines that should be given to a patient if not already vaccinated after a splenectomy, some of which are:
1- Pneumococcus
2- Haemophilus influenzae type b (Hib
3- Meningococcal
4- Flu (influenza).
Reference: http://patient.info/health/preventing-infection-after-splenectomy-or-if-you-do-not-have-a-working-spleen

202.Abdominal radiological investigation showed the inner and outer surface of intestine , what the patient has ?
A) Perforation
B) Obstruction

Answer: A
Normally on X-ray only the inner wall of the bowel is visible. If there is pneumoperitoneum (Gas under the diaphragm) both sides of
the bowel wall may be visible. (Rigler's/double wall sign)

51
Reference:
http://www.radiologymasterclass.co.uk/tutorials/abdo/abdomen_x-
ray_abnormalities/pathology_bowel_gas_perforation
203.What is the most common type of gallstones?
A. Cholesterol gallstones
B. Bile pigmented or Pigmented gallstones
Answer : A
Reference: http://www.mayoclinic.org/diseases-
conditions/gallstones/basics/causes/con-20020461

204.Best imaging for cyst in the breast


A. US
B. Mammogram
Answer: A
- US is used to differentiate between solid and cystic lesion .
- Mammogram visualize large cyst but not small microcystic lesion .
Reference :-
- Http://www.uptodate.com/contents/breast-cysts-clinical-manifestations-diagnosis-and-
management?Source=outline_link&view=text&anchor=H91862703#H91862703
- Http://www.mayoclinic.org/diseases-conditions/breast-cysts/basics/tests-diagnosis/con-20032264

205.A patient presented with splenic injury after abdominal trauma. The surgeon decided to embolize the splenic artery. Which
of the following will be compromised?
A. Stomach fundus
B. Splenic flexure

Answer : A
Reference: http://radiopaedia.org/articles/splenic-artery

52
206.Ligament in inguinal canal?
A. A.Broad
B. B.Round

Answer: B

207.Perianal itching. Most likely?


A. Perianal abscess
B. Hemorrhoid
Answer:B
Hemorrhoids, which cause painful swelling of blood vessels in the anal area, can cause itching.
References:
Http://www.emedicinehealth.com/script/main/mobileart-emh.asp?Articlekey=58920&page=2
Http://www.emedicinehealth.com/script/main/mobileart-emh.asp?Articlekey=58920&page=2

208.Patient history of trauma and right femur fracture with blood loss which is accepted change ?
A. Decrease coronary blood flow
B. Decrease venous capacity

Answer: The cardiovascular system initially responds to hypovolemic shock by increasing the heart rate, increasing myocardial
contractility, and constricting peripheral blood vessels. This response occurs secondary to an increased release of norepinephrine
and decreased baseline vagal tone (regulated by the baroreceptors in the carotid arch, aortic arch, left atrium, and pulmonary
vessels).

209.Patient developed parotid gland swelling after cholecystectomy, what is the cause?
A. Bacterial sialadenitis
B. Immunity

Answer:
Explanation: Swelling of the parotid gland post operatively “called anesthesia mumps” is almost always painless and it resolves
spontaneously over a period of hours or days. The exact cause is not known.

210.Patient came to ER with multiple fractures, no loss of consciousness with low blood pressure, tachycardia, normal RR, O2
saturation = 95%. What is the most appropriate next step?
A. IV fluid
B. Save airway
Answer: B
In accordance with the ATLS protocol the airway has to be assessed first and made sure to be patent before assessing the circulation.
Reference: http://www.who.int/violence_injury_prevention/publications/services/en/guidelines_traumacare.pdf
Management of the trauma patient: primary survey ABCDEs. Airway (with maintenance of C spine): if the patient is alert and
answers the questions with a clear voice, the airway is intact. If not secure, rapid sequence endotracheal intubation or definitive
surgical airway should be established. (Mont Reid surgical handbook)

211.In mastectomy what you will cut? Pectorals major muscle


Answer
Total mastectomy: remove entire brest with nipple.
Preventive mastectomy: is prophylactic, same total mastectomy

53
Partial mastectomy: it’s is a breast-conserving method in which the tumor and the tissue surrounding it are all that’s removed.
Radical mastectomy: is the complete removal of the breast, including the nipple, the overlying skin, the muscles beneath the breast,
and the lymph nodes.
Modified radical mastectomy: entire breast is removed as well as the underarm lymph nodes. But chest muscles are left intact. The
skin covering the chest wall may or may not be left intact. The procedure may be followed with breast reconstruction.
(http://www.webmd.com/breast-cancer/mastectomy?page=2#2)

212.drug to delay surgery?


A. Nefidipine

213.elderly heavy smoker with pain on walking and o/e loss of hair and pallor of feet whats dx?
Answer: pvd
Ref: http://www.healthline.com/health/peripheral-vascular-disease#overview1

214.Trauma with forearm streak … long scenario , how to manage :


A. Oral antibiotic
Question and choices incomplete
Explanation: patient most likely has lymphangitis. A history of minor trauma to an area of skin distal to the site of infection is often
elicited in patients with lymphangitis. Upon clinical examination, erythematous and irregular linear streaks extend from the primary
infection site toward draining regional nodes. These streaks may be tender and warm. Patients with lymphangitis should be treated
with an appropriate antimicrobial agent.

215.Papillary thyroid ca mostly associated with which of the following


A. Hurthle cell
Http://www.nature.com/modpathol/journal/v24/n2s/full/modpathol2010129a.html

216.Name of triangle between hyoid bone and two anterior belly of diagastric?
A. Submental triangle
Submental Triangle: between the anterior belly of the digastric, superior to the hyoid bone, and the midline of the neck

217.Femoral vein relation to the femoral artery?


Answer: Medial

The well-described NAVEL mnemonic has been used to help medical students and surgical trainees remember the relationship of the
femoral sheath structures in a lateral-to-medial direction: femoral N erve, femoral A rtery, femoral V ein, E mpty space, and L
ymphatics.
Source: http://emedicine.medscape.com/article/1923032-overview#a2

218.Patient underwent right radical mastectomy. Which of the following will be affected post operatively?
A) Right arm adduction
Answer: A - pec. Major action
Radical mastectomy is a surgical procedure in which the breast, underlying chest muscle (including pectoralis major and pectoralis
minor), and lymph nodes of the axilla are removed as a treatment for breast cancer.
Http://thewellnessdigest.com/pectoralis-major-anatomy-origin-insertion-actions-innervation/

54
219.Fractured pelvis, injury to gonadal artery, what organ affected? (not sure of the Q)
A. Ovary

Answer: the pelvis forms one major ring and two smaller rings of bone that support and protect the bladder, intestines and rectum.
Organ pelvic protect (Bladder, lower colon, lymph nodes, uterus, and vagina

220.64 years male with medial enlarged prostate, all labs normal, what to do?

Http://emedicine.medscape.com/article/437359-treatment

221.Spontaneous lips swelling?

Answer: Swollen lips may have a range of different causes, from mild conditions to more serious disorders, including infections,
allergy, inflammation, injuries or an underlying medical condition.

222.Patient with bilateral abdominal mass:


Polycystic kidney disease.
Answer: A
Source: http://emedicine.medscape.com/article/244907-overview

223.A patient was injured and now has scrotal pain, what’s the most likely diagnosis (was playing football)
A. Testicular torsion
Answer:A http://www.uptodate.com/contents/evaluation-of-the-acute-scrotum-in-
adults?Source=machinelearning&search=scrotal+pain&selectedtitle=1~25&sectionrank=1&anchor=H671410222#H2012423711

224.What is the cause of death due to flames fire:


A. Smoke inhalation injuries
Answer: A
Source: http://www.nfpa.org/press-room/reporters-guide-to-fire-and-nfpa/consequences-of-fire

225.Dislocation of temporomandibular joint, doctor try to hyper extend the jaw to relocate it. What is the muscle that elevate the
jaw?
A. Temporalis

226.A victim of RTA present with multi trauma . There is ecchymosis around the eye & subconjunctival haemorrhage. During
examination the patient was unable to look superiorly.
A. Blunt trauma to the eye

Answer: Read about orbital fracture


Key diagnostic factor is diplopia on upward gaze, the most common clinical signs seen with adult orbital fractures are periorbital
ecchymoses and subconjunctival hemorrhages
Http://bestpractice.bmj.com/best-practice/monograph/1172/diagnosis/history-and-examination.html
Http://www.ncbi.nlm.nih.gov/pmc/articles/PMC3324224/
227.Cause of abnormality in breast cancer .. dimpling skin. Ligament affected?

Retraction phenomena (concave changes of the skin of the breast) range from a small area of skin flattening in the vicinity of an
underlying tumor or area of thickening, to shrinkage of most of the skin of the breast. They usually result from shortening of
Cooper's ligaments due to fibrosis. Carcinoma is the most common cause and is usually associated with a distinct mass or very
subtle underlying thickening in the breast tissue.

55
228.Pancreatitis chronic vs. Acute

229.What is the name of incision in open cholecystectomy?

Answer: Kocher (subcostal)

230.What is the lymphatic drainage of upper lateral part of breast?

Answer: no choices
Possible answer: the lateral quadrant drains into anterior axillary or pectoral LN (75%)

56
231.What is the most sensitive and specific to diagnose and localize bowl obs ?
 CT
CT is appropriate for further evaluation of patients with suspected intestinal obstruction in whom clinical examination and
radiography do not yield a definitive diagnosis. CT is sensitive for detection of high-grade obstruction (up to 90 percent in some
series), and has the additional benefit of defining the cause and level of obstruction in most patients. In addition, CT can identify
emergent causes of intestinal obstruction, such as volvulus or intestinal strangulation.
Http://www.aafp.org/afp/2011/0115/p159.html

232.Most accurate test for acute colysctitis?

The American College of Radiology (ACR) makes the following imaging recommendations:
 Ultrasonography is the preferred initial imaging test for the diagnosis of acute cholecystitis; scintigraphy is the preferred
alternative
 CT is a secondary imaging test that can identify extrabiliary disorders and complications of acute cholecystitis
 CT with intravenous (IV) contrast is useful in diagnosing acute cholecystitis in patients with nonspecific abdominal pain
 MRI, often with IV gadolinium-based contrast medium, is also a possible secondary choice for confirming a diagnosis of acute
cholecystitis
 MRI without contrast is useful for eliminating radiation exposure in pregnant women when ultrasonography has not yielded a clear
diagnosis
 Contrast agents should not be used in patients on dialysis unless absolutely necessary
Http://emedicine.medscape.com/article/171886-overview

233.Sites of Displaced appendix ?


Read about appendicitis:
Appendicitis is defined as an inflammation of the inner lining of the vermiform appendix that spreads to its other parts. The
appendix has no fixed position. It originates 1.7-2.5 cm below the terminal ileum, either in a dorsomedial location (most common)
57
from the cecal fundus, directly beside the ileal orifice, or as a funnel-shaped opening (2-3% of patients). The appendix has a
retroperitoneal location in 65% of patients and may descend into the iliac fossa in 31%
Http://emedicine.medscape.com/article/773895-overview#a3

234.Patient had splenic trauma and splenectomy planned, if the surgeon ligate the splenic artery high up what structure would be
affected?
Answer: Stomach fundus not sure
The splenic artery supplies the spleen and substantial portions of the stomach and pancreas (Fig 1) (1). The splenic artery courses
superior and anterior to the splenic vein, along the superior edge of the pancreas. Near the splenic hilum, the artery usually divides
into superior and inferior
Terminal branches, and each branch further divides into four to six segmental intrasplenic branches.
Http://pubs.rsna.org/doi/full/10.1148/rg.25si055504

235.Post surgical prophylaxis ?


Answer: LMWH
Heparin has been successfully used in the prophylaxis and treatment of venous and arterial thrombotic disorders. When compared
with unfractionated heparin, lmwhs have fewer serious side effects, such as heparin-induced osteopenia and heparin-induced
thrombocytopenia (HIT).[9] In contrast to UFH, the lmwhs appear not to be inactivated by platelet factor 4. After subcutaneous
injection, lmwhs also exhibit less plasma protein binding (and therefore greater bioavailability) than UFH.
Http://www.medscape.com/viewarticle/437218

236.Diabetic patient developed foot ulcer, diabetes role for this non healing?
Answer: Inhibit phagocytosis
Occur as a result of various factors, such as mechanical changes in conformation of the bony architecture of the foot, peripheral
neuropathy, and atherosclerotic peripheral arterial disease, all of which occur with higher frequency and intensity in the diabetic
population.
Http://emedicine.medscape.com/article/460282-overview

237.Patient with groin abscess after aspirate there was multiple cells ..... Cause?
Immunodefieciency - C5 --)

238.Less complicated of NSAID drugs in PUD?

239.Elderly with abdominal tenderness and dilated bowel loops?


Small bowel obstruction?

240.Patient treated for duodenal ulcer. Now complains of breast enlargement and decrease sexual desire. Which drug?
Answer: cimetidine ?
In men, gynecomastia has been reported. During post marketing surveillance in the 1980s, cases of male sexual dysfunction were
also reported.

241.Acute cholecystitis management?


In acute cholecystitis, the initial treatment includes bowel rest, intravenous hydration, correction of electrolyte abnormalities,
analgesia, and intravenous antibiotics.

242.A question about the artery you see on lateral of bowel in hernia operation?
Answer: Inferior epigastric artery (medial to inguinal ring lateral to bowel)

58
Vessels regularly found during inguinal hernia repairs are the superficial circumflex iliac, superficial epigastric, and external
pudendal arteries, which arise from the proximal femoral artery and course superiorly. The inferior epigastric artery and vein
run medially and cephalad in the preperitoneal fat near the caudad margin of the internal inguinal ring.
Link:
Http://emedicine.medscape.com/article/189563-overview

243.A patient with indirect inguinal hernia. What artery runs medial to it?
A. Inferior epigastric artery
Answer: A
Reference: Surgical Recall, 6th edition

244.A patient presented with RLQ pain, guarding, tenderness and positive obturator sign. What is the most likely diagnosis?
A. Appendicitis.

Answer: A Obturator sign: Pain upon internal rotation of the leg with the hip and knee flexed; seen in patients with pelvic
appendicitis.
th
Reference: Surgical Recall, 6 edition

Explanation: The classic history of anorexia and periumbilical pain followed by nausea, right lower quadrant (RLQ) pain, and
vomiting. Physical exam: Rebound tenderness, pain on percussion, rigidity, and guarding: Most specific finding.
Reference: http://emedicine.medscape.com/article/773895-overview

245.Typical scenario for appendicitis , mechanism: (GS)


Fecal impaction on the app Lumen

Answer:
Appendiceal obstruction has been proposed as the primary cause of appendicitis [7,11-14]. Obstruction is frequently implicated but
not always required for the development of appendicitis.Appendiceal obstruction may be caused by fecaliths (hard fecal
masses), calculi, lymphoid hyperplasia, infectious processes, and benign or malignant tumors. However, some patients
with a fecalith have a histologically normal appendix and the majority of patients with appendicitis do not have a fecalith.
Refrence: http://cursoenarm.net/UPTODATE/contents/mobipreview.htm?16/55/17265

246.Patient with right hypochondrial pain and US showed echogenic shadow ?


Answer : Gallstones (Cholelithiasis) ,
Stone in the gallbladder neck with typical acoustic shadow.
Although it originally referred to ultrasonographic findings of echogenic, nonshadowing, microscopic material within the gallbladder,
the term biliary sludge currently indicates a precipitate of microcrystals occurring in bile with high mucous content. Sludge may
contain microliths. Milk of calcium bile, a calcium carbonate precipitate opaque on plain radiographs, may coexist with cholelithiasis.
Ref: http://emedicine.medscape.com/article/366246-overview#a1

247.The surgeon has documented if the infant has a constant infantile esotropia exceeding 12 PD, surgical realignment should be
performed.
Http://emedicine.medscape.com/article/1198876-treatment#d6

248.Pt. Perform surgery to stomach that rapped her around the esophagus which nerve will be injured:
A) Vagal
Answer: A
Reference: http://www.medscape.com/viewarticle/804147_4
59
249.Patient with metastatic breast cancer presented with SOB , distended neck vain ,apex beat not palpable Distal heart sounds ?
BP 70/... The best step to confirm the Diagnosis is?
IV fursmide IV fluid & urgent echo :::::::::::::::::::::::::::::
Answer : convensional SV Cavography .
Inisial >> doplex US

250.Acute scrotal pain


Reference: http://www.rch.org.au/clinicalguide/guideline_index/Acute_Scrotal_Pain_or_Swelling/ + Toronto notes.

251.Case with leg pain while walking, loss of hair in leg , nail changes , what is the cause , The cause is going to be a peripheral
vascular cause

Reference: Toronto notes.

60
252.Patient with +ve Cullen sign and Grey-Turner sign what is the diagnosis?
A.Acute pancreatitis
Answer:A

Reference:Toronto notes

253.Question about thyroid cancer , i remember that i choose hurthle cell adenoma but not sure if it’s the correct answer or not
hurthle cell adenoma:

Hürthle cell carcinoma of the thyroid gland is an


unusual and relatively rare type of differentiated
thyroid cancer.The cytologic features for Hürthle
cell neoplasms are hypercellularity, with a
predominance of Hürthle cells(large and polygonal
in shape, with indistinct cell borders) ,no
lymphocytes, and scanty or absent colloid.

Reference:
http://emedicine.medscape.com/article/279462-
overview+Toronto notes

254.Patient with appendicitis but it presented as right upper quadrant pain. Cause?

61
A. Subhepatic appendix

Answer: A
Ascending subhepatic appendicitis presenting with right upper abdominal pain may be clinically indistinguishable from acute
pathology in the gallbladder, liver, biliary tree, right kidney and right urinary tract
http://www.sciencedirect.com/science/article/pii/S0378603X12000538

255.Patient present with Right subcostal area pain ... And you do surgery and you find inflamed appendix?
Answer: sub-hepatic appendix
http://www.sciencedirect.com/science/article/pii/S0378603X12000538

256.Forceps forgotten inside patient after surgery, what will you do?
Answer : tell the patient what you found and refer him to OR

257.Patient with osteoid sarcoma. Then he started having pelvic pain. Cartilage and bone tumor in pelvic bone on imaging.
Cause?
a. Chondreoblastic Osteoma
Answer : Osteochondroma or Enchondroma ?

Http://www.uptodate.com/contents/chondrosarcoma?Source=outline_link&view=text&anchor=H2#H2

258.About patient with breast cancer on something and having bleeding on routine check 2x3 cm then became 3x6cm what
suspect: type of cancer? "
Inflammatory breast cancer is a rare and very aggressive disease in which cancer cells block lymph vessels in the skin of the breast.
This type of breast cancer is called “inflammatory” because the breast often looks swollen and red, or inflamed.
Inflammatory breast cancer is rare, accounting for 1 to 5 percent of all breast cancers diagnosed in the United States. Most
inflammatory breast cancers are invasive ductal carcinomas, which means they developed from cells that line the milk ducts of the
breast and then spread beyond the ducts.
Http://www.cancer.gov/types/breast/ibc-fact-sheet

259.After appendectomy he has lung abscess which antibiotic you will choose
(read about it)
Answer: ?

260.Auer rods indicate?


a. Acute myeloid leukemia
Answer: A
Ø source: http://www.pathologystudent.com/?P=4868

261.Microsatellite instability?
a. Colorectal cancer
Answer: A
Http://www.ncbi.nlm.nih.gov/pmc/articles/PMC3037515/

262.Female came for breast screening, her mother and sis had breast ca at 45-48 year

62
What will u do now?
a. BRAC1 Gene Screen

Answer: ?
BRCA 1 And mammogram

Http://ww5.komen.org/breastcancer/breastcancerscreeningforwomenathigherrisk.html
263.Main difference between chron's , ulcerative and IBS should be known
Answer:

264.Patient had trauma, presented with tachycardia, bradychpnea and hypotension, what is the first thing you will do?
a. Needle thoracotomy

Answer: A (tension pneumothorax)


“Classical management of tension pneumothorax is emergent chest decompression with needle thoracostomy. A 14-16G
intravenous cannula is inserted into the second rib space in the mid-clavicular line.
Http://www.trauma.org/archive/thoracic/chesttension.html

265.Patient presented to you with Rt scrotal swelling, he said that it is increasing in size day after day , on examination , it is not
transluminate , what will you do ?
a. Reassure him and get surgical opinion
Answer: A

266.High fat and low fiber diet increases the risk of which of the following cancers?
a. Colorectal cancer
Answer: A

267.Most common parotid gland malignancy?


ANSWER: Mucoepedermoid carcinoma
Medscape

63
268.Staging of lung cancer, tracheal nodes were involved?
Answer : if lung cancer reash trachea it’s T4
N1 Metastasis in ipsilateral peribronchial and/or ipsilateral hilar lymph nodes and intrapulmonary nodes, including involvement
by direct extension
missing data
http://www.uptodate.com/contents/image?Imagekey=ONC%2F80099&topickey=NEPH%2F3840&source=see_link

269.The most common cause of breast bloody discharge:


A. Ductal papilloma
Answer: A
Reference: Surgical Recall, 6th edition

270.Patient came with abdominal swelling and weak abdominal wall. What’s the diagnosis?
A. Direct inguinal hernia

Answer: A

271.A patient presented complaining of dark stool and vomiting blood. He is stable now. What is your investigation?
A. Upper GI endoscope.

Answer: A
Mostly from upper GIT bleeding

272.What is the best study to determine cystic breast lesion?


Answer: ?
US is the best way to evaluate cystic lesions. Reference: http://breast-cancer.ca/ult-bens/
273.What is the most common site of Meckel’s diverticulum?
A. Lower ileum

Answer: A
Mostly Meckel diverticula are found Within 2 feet (100cm) of the ileocecal valve on
th
The antimesenteric border of the bowel. Reference: Surgical Recall, 6 edition

274.What is the most common site of breast mass?


A. Superior and lateral

Answer: A
Approximately one half of breast cancers develop in the upper outer quadrants.
th
Reference: Surgical Recall, 6 edition

275.Patient had Upper GI bleeding. What is the diagnosis?


A. Zollinger ellison syndrome

Answer: Missing a lot of info


Read about Upper GI bleeding. Zollinger ellison syndrome is a condition in which a gastrin-secreting tumor or hyperplasia of the islet
cells in the pancreas causes overproduction of gastric acid, resulting in recurrent peptic ulcers.

276.What is the common presentation of perforated duodenal ulcer?

64
Answer: ?
The most characteristic symptom is the suddenness of the onset of epigastric pain. The pain rapidly becomes generalized although
occasionally it radiates to the right lower quadrant (Valentino’s sign).
Reference: http://www.ncbi.nlm.nih.gov/books/NBK6926/
277.17-year-old complaining of abdominal pain that shifts to the right iliac fossa. What is the most likely diagnosis?
Answer: ?
Appendicitis
Features of the abdominal pain are as follows:
● Typically begins as periumbilical or epigastric pain, then migrates to the RLQ
● Patients usually lie down, flex their hips, and draw their knees up to reduce movements and to avoid worsening their pain
Reference: http://emedicine.medscape.com/article/773895-overview

278.A female patient who has high risk for breast cancer. Which gene is responsible for that?
Answer: ?
BRCA1
The family history characteristics that suggest increased risk of cancer are summarized as follows:
● Two or more relatives with breast or ovarian cancer
● Breast cancer occurring in an affected relative younger than 50 years
● Relatives with both breast cancer and ovarian cancer
● One or more relatives with two cancers (breast and ovarian cancer or 2 independent breast cancers)
● Male relatives with breast cancer
● BRCA1 and BRCA2 mutations
● Ataxia telangiectasia heterozygotes (quadrupled risk)
● Ashkenazi Jewish descent (doubled risk)
Reference: http://emedicine.medscape.com/article/1947145-overview#a6

279.Prevent of hemorrhoid:
A. High fiber diets
Answer: A
The best way to prevent hemorrhoids is to keep your stools soft, so they pass easily. To prevent hemorrhoids and reduce symptoms
of hemorrhoids, follow these tips:
 Eat high-fiber foods. Eat more fruits, vegetables and whole grains. ...
 Drink plenty of fluids. ...
 Consider fiber supplements. ...
 Don't strain. ...
 Go as soon as you feel the urge. ...
 Exercise. ...
 Avoid long periods of sitting.
Reference: http://www.mayoclinic.org/diseases-conditions/hemorrhoids/basics/prevention/con-20029852

280.which thyroid cancer have the best prognosis?


Answer: Papillary cancer. Survival rate 98% at 10 yr Reference: Toronto Notes 2015

281.Which of the following protect against some cancers?


A. Fibrate

Reference : http://www.uptodate.com/contents/cancer-prevention?Source=outline_link&view=text&anchor=H11#H11

65
282.during laparoscopic surgery of inguinal hernia, you find artery superficial going upward?

A. Inferior epigastric artery.


Answer: Inferior Epigastric vessels
Great care must be exercised as the dissection approaches the iliac vessels. In addition, obturator vessels
often cross the dissection planes and may need to be clipped and divided.
The inferior epigastric vessels are identified, and dissection lateral to the vessels leads to the space of Bogros, the cord structures,
and indirect hernias (see the video below). The proper plane of dissection is between the transversalis fascia and the peritoneum.
This is identified by retracting the inferior epigastric vessels upward against the rectus muscle. A plane containing areolar tissue is
identified, and this plane is dissected toward the pelvic sidewall
Referance : medscape
http://emedicine.medscape.com/article/1534321-overview

283.Case Symptoms of cholecystitis what best initial modality ?


A. U/S

Answer: A
Sonography is the preferred initial imaging test for the diagnosis of acute cholecystitis, and scintigraphy is the preferred alternative.
CT is a secondary imaging test that can identify extrabiliary disorders and complications of acute cholecystitis, such as gangrene, gas
formation, and perforation.
Reference:
Http://emedicine.medscape.com/article/171886-workup#c7
284.Post appendectomy, penicillin resistance? What antibiotic use?

Answer: Gentamicin
Gentamicin is not the drug of choice in cases of appendectomy, but consider using this drug if penicillins or other less toxic drugs are
contraindicated.
Reference: http://emedicine.medscape.com/article/773895-medication#4
Answer: Broad-spectrum gram-negative and anaerobic coverage is indicated .Penicillin-allergic patients should avoid beta-
lactamase type antibiotics and cephalosporins. Carbapenems are a good option in these patients.

Reference: http://emedicine.medscape.com/article/773895-treatment#d10

NONPERFORATED APPENDICITIS :
Postoperative management — With both the open and laparoscopic approaches, most patients are discharged within 24 to
48 hours of surgery. Patients may be started on a clear liquid diet post-operatively and advanced to regular diet as tolerated.
Antibiotics are not required postoperatively in nonperforated appendicitis.

PERFORATED APPENDICITIS :
Postoperative management — Postoperatively, these patients often have an ileus, and diet should only be advanced as the
clinical situation warrants. Patients may be discharged once they tolerate a regular diet, usually in five to seven days. The duration of
antibiotic therapy in such patients is discussed elsewhere. (See "Anaerobic bacterial infections", section on 'Antibiotic treatment'.)

Reference: http://www.uptodate.com/contents/management-of-acute-appendicitis-in-adults

285.mass in the neck in cervical area, the best initial test?

Answer: CT
Reference:
Http://www.uptodate.com/contents/evaluation-of-a-neck-mass?Source=outline_link&view=text&anchor=H13#H13

66
286.30 years old female dancer with breast mass that disappeared by aspiration ?
Answer: Complete disappearance of the breast mass after aspiration of non bloody fluid , indicate simple cyst
References : pubmed & medscape
http://www.medscape.org/viewarticle/718139
Http://www.ncbi.nlm.nih.gov/pmc/articles/PMC2855917/#!Po=15.7407

287. Sign of hyperthyroidism with nodule in lateral to mid line


Answer: First rule any thyroid nodule not showing or changing hormone activity and signs it is most likely malignant nodule
Hyperthyroidism signs and symptoms:
 Nervousness
 Anxiety
 Increased perspiration
 Heat intolerance
 Hyperactivity
 Palpitations
Reference: Medscape

288.MVA diagnosed with fracture of base of skull and injury to nerve pass through jugular foramen

Answer: Cranial nerves 9, 10, and 11


Vernet syndrome or jugular foramen syndrome is involvement of the IX, X, and XI cranial nerves with basal skull fracture. Patients
present with difficulty in phonation and aspiration and ipsilateral motor paralysis of the vocal cord, soft palate (curtain sign),
superior pharyngeal constrictor, sternocleidomastoid, and trapezius.
Reference : http://emedicine.medscape.com/article/248108-overview#showall

289.In splenectomy, what organ might be affected?


Answer: Atelectasia of the lower lung +injury of surrounding structure eg. gastric wall , tail of pancreas
((Toronto notes,2015,GS52 ))

290.Female c/o clear fluid from nipple. Breast examination was normal. Brain MRI: pituitary mass. This mass secrets?
A. Prolactin
Answer: A

291.Male patient complain of dysphagia for solid and liquid, radiograph shows dilated esophagus with tapering at end, what is
the diagnosis?
A. Spasm
B. Cancer
C. Achalasia

Answer: C, dysphagia (which is for both solids and liquids, in comparison to dysphagia for solids only in cases of esophageal
carcinoma).A barium swallow of dilated esophagus with tapering at end (bird-beak sign or rat-tail sign) is classical in primary
achalasia, can also occur in pseudoachalasia due to a gastric tumour involving the gastro-oseophageal junction.
Http://radiopaedia.org/articles/achalasia

292.A patient with a family history of multiple sudden cardiac death patientt has Marfan's syndrome features. What might be the
cause of death?
A. Ruptured aortic aneurysm
Answer: A
((First Aid for the USMLE Step 1 CK 2104,p 284 ))

67
293.Tamoxifen SE or Complications?
Answer:
Commonly reported side effects of tamoxifen include: nausea, weight loss, amenorrhea, vaginal discharge, hot flash, fluid
retention, vaginal hemorrhage, and skin changes. Other side effects include: sepsis, infection, constipation, weight gain, alopecia,
menstrual disease, vomiting, ostealgia, increased serum aspartate aminotransferase, cough, oligomenorrhea, edema, and diarrhea.
See below for a comprehensive list of adverse effects. Drug.com https://www.drugs.com/tamoxifen.html

294.Stap wound in the abdomen with bacteroid fragilis what is the best antibiotic to use …
A. ampicillin

295.Patient with BPH, what will you give?


Answer: ?
It depends on the case and the which type of treatment (watchful waiting, medical, surgical)
If it’s about medication you might check the following link
Http://www.urologyhealth.org/urologic-conditions/benign-prostatic-hyperplasia-%28bph%29/treatment

296.her-neu gene amplification


Breast cancers with her2 gene amplification or her2 protein overexpression are called her2-positive in the pathology report. Her2-
positive breast cancers tend to grow faster and are more likely to spread and come back compared to her2-negative breast cancers.
Treatments available specifically for her2-positive breast cancer: herceptin (chemical name: trastuzumab), which works against her2-
positive breast cancers by blocking the ability of the cancer cells to receive chemical signals that tell the cells to grow.
Http://www.breastcancer.org/symptoms/diagnosis/her2

297.BRCA
BRCA1 and BRCA2 are human genes that produce tumor suppressor proteins. These proteins help repair damaged dna and,
therefore, play a role in ensuring the stability of the cell’s genetic material. When either of these genes is mutated, or altered, such
that its protein product either is not made or does not function correctly, DNA damage may not be repaired properly. As a result,
cells are more likely to develop additional genetic alterations that can lead to cancer.
http://www.cancer.gov/about-cancer/causes-prevention/genetics/brca-fact-sheet

298.Hiatal hernia .. During surgery, where is nerve position?

Read about:
• Fertility Q
• Case of cystitis.
• Case about renal colic
• Main difference between ( crohn's , ulcerative and IBS )
• Dumping syndrome post surgery
• Hernia (especially anatomical structures)
• Renal artery stenosis
• Gallbladder stone
• Big arterier and their branches with their supply.
• *dermatomes and nerve supply of upper and lower limb.
• *acute cystitis
• * Margin of neck Quadrants
VAN:
V- Vein lateral
A- Artery middle
N- Nerve medial

68
Plastic Surgery

69
1. Body surface area rule of?
A. 9s
B. 8
C. 7
Answer: A

2. Burn tanning bed swelling red blisters?


A. Grade 1
B. Grade 2
C. Grade 3
Answer: B

3. Child presented with burn in the upper right extremity with bluster what is the degree of the burn
A. 2nd degree more than 15 %
B. 2nd degree less than 15 %
C. 3rd degree more than 15 %
D. 3rd degree less than 15 %
Answer: B

4. Median nerve injury leads to:


A. Complete claw hand
B. Ape hand
C. Partial claw
D. Ulnar Claw

Answer: B
Reference: Textbook of Anatomy - Upper Limb and Thorax

5. Long thoracic nerve damage with winging of scapula, this nerve arises from which part of the brachial plexus?
A. Upper trunk.
B. Posterior cord.
C. Medial cord.
D. Root

Answer: D
Root: C5, C6, C7 http://www.orthobullets.com/anatomy/10135/long-thoracic-nerve

6. In which position will you apply a splint in patients with carpal tunnel syndrome?
A. Dorsiflexion.
B. Plantarflexion
C. Extension
D. Abduction

Answer: A
The ideal position of the wrist is close to neutral with 2° + 9 of dorsiflexion and 2° + 6 ulnar deviation to prevent prolonged flexion or
extension that prevents any raise in the pressure.
Reference: Occupational Therapy for Physical Dysfunction.

7. A patient with thenar atrophy but with no numbness. What is the nerve affected?
A. Axillary
B. Ulnar

70
C. Median (Anterior interosseous nerve)
D. Radial

Answer: C
Thenar atrophy is an eroding of muscle tissue, which can impair control over the thumb and leave the hand disfigured. The problem
can be a complication of several different conditions and disorders, including carpal tunnel syndrome, acromegaly, or direct trauma
to the wrist or thumb.

8. Pt with carpal tunnel what is the diagnostic test:


A. MRI
B. CT
C. X-ray
D. Nerve conduction
Answer: D
Electrophysiological tests (nerve conductive study). Electrical testing of median nerve function is often done to help confirm the
diagnosis.
Reference: http://orthoinfo.aaos.org/topic.cfm?Topic=a00005

9. Artistes cannot abduct his fingers; which muscle is affected?


A. Thenar muscle
B. Anterior interossei
C. Posterior interossei

ANSWER: C Dorsal (Posterior Interossei)

- The palmar interosseous muscles adduct the fingers towards the middle finger. This is in contrast to the dorsal interossei,
which abduct the fingers away from the middle finger.
- SURGERY Clerkship
- Https://en.wikipedia.org/wiki/Palmar_interossei_muscles
- Thenar muscles are 4 muscles that are responsible for fine movements of the thumb; they are supplied by the median &
ulnar nerve.
 Palmar (anterior) interossei adduct the fingers at the MCP joint; they are supplied by the ulnar nerve.
 Dorsal (posterior) interossei abduct the fingers at the MCP joint; they are supplied by the ulnar nerve.
Source: http://teachmeanatomy.info/upper-limb/muscles/hand/
10. Carpal tunnel syndrome. Pt can't work and write. Which muscles affected?
A- Theaner
B- Interossei palmar muscle
C- Interossei dorsal muscle
Answer: A

11. Pronator teres syndrome, which nerve is entrapped:


A. Ulnar
B. Radial
C. Median

Answer: C
Reference: https://www.youtube.com/watch?V=zqho1dzqtty
Pronator teres syndrome is a compression neuropathy of the median nerve at the elbow. It is rare compared to compression at the
wrist (carpal tunnel syndrome) Reference: Wikipedia.

71
12. 40 years old male patient work on the computer a lot. After that get tingles around lateral aspect of the hand. What is the
most accurate test??????????
A. Tinel’s sign
B. Phalen Test
C. Durkan's carpal test

Answer: C
Durkan: (Also known as carpal tunnel compression test) sensitivity: 89% and specificity: 96%. It is considered by some to be a better
provocative test than tinel and phalen tests.
Reference:https://books.google.com.sa/books?Id=d4mqcgaaqbaj&pg=PA110&dq=tinel+phalen+compression&hl=en&sa=X&redir_e
sc=y#v=onepage&q&f=false

13. Diagnosis of carpal tunnel is by?


A. Compression test
B. Kats test
C. Phalen's test

Answer: A
Electrodiagnostic testing, primarily with nerve conduction studies (NCS), sometimes supplemented with needle electromyography
(EMG), is a standard part of the evaluation for CTS because it has a high sensitivity and specificity for confirming the diagnosis.

14. Which indicate how long open wound ..in inflammatory process & how can you tell when it has healed ?
A. Coverd by eshar
B. Covered by epithelium
C. Neovascularization

Answer: B
Reference: http://www.uptodate.com/contents/wound-healing-and-risk-factors-for-non-
healing?Source=outline_link&view=text&anchor=H1070222553#H1070222553
http://emedicine.medscape.com/article/1298129-overview#a5

15. Trauma to shoulder PT can’t raise hand the injury is in:


A. Posterior cord of biracial plexus
B. Medial cord
C. Lateral cord

Answer: A
The axillary nerve is a branch of the posterior cord of the brachial plexus

Http://emedicine.medscape.com/article/1877731-overview#a2

16. Patient after tan bath presented with pain and swelling in chest and back:
A. 1st degree burn
B. 2nd degree
C. 3rd degree
Answer: A
17. Loss of adduction of fingers caused by injury to:
A- ulnar
B- median

Answer: A

72
Ulnar nerve supplies all the intrinsic muscles of the hand except the first two lumbricals and muscles of thenar eminence
[http://emedicine.medscape.com/article/1243669-overview]

18. Patient had injury to the ulnar nerve?


A. Complete claw hand
B. Partial claw

Answer: B
➢Effects of the ulnar nerve injury: ulnar claw hand (Partial claw hand) and hollowing of skin in the first web space on dorsal aspect
of hand.
➢Complete claw hand results from combined lesions of the median and ulnar

Reference: Textbook of Anatomy - Upper Limb and Thorax


19. Patient with chronic osteomyelitis of her leg and there is sinus draining. She has cancer of skin near the sinus which has
eosinophilic inclusions. What is the type of cancer?
a. Squamous cell carcinoma.
b. Basal cell carcinoma.

Answer: A
De novo squamous cell carcinoma emerge in the setting of long-standing ulcers, burn scars, or osteomyelitis. Reference: Journal of
Skin Cancer.

20. Young patient with pain in forearm worsening day by day, x-ray shows "onion peel" best investigation?
A. MRI??

73
ANSWER:
- “onion skinning” is classic sign for Ewing sarcoma
- The diagnostic work-up is usually initiated with a plain radiograph of the affected area
- MRI is preferred in most cases because of its superior definition of tumor size, local intraosseous and extraosseous extent,
and the relationship of the tumor to fascial planes, vessels, nerves, and organs
- Https://yhdp.vn/uptodate/contents/mobipreview.htm?36/56/37769#H6

21. Patient was burn with boiling water after that he develop blister and blanch skin with pressure what is the degree of this
burn?

Answer: second degree burn


Second-degree burns (also known as partial thickness burns) involve the epidermis and part of the dermis layer of skin. The burn site
appears red, blistered, and may be swollen and painful.
Http://www.stanfordchildrens.org/en/topic/default?Id=second-degree-burns-partial-thickness-burns-90-P01757

22. Case of burn (Know how to calculate the TBSA and how to determine the degree of burn).

Reference: Toronto Notes

74
23. Case of burn, how much fluids you will give in the 1st 8 hours?
Answer: ½ of fluids
Parkland formula: 4 ml * kg * TBSA%
Reference: Toronto notes

24. Case of burn (anterior trunk, upper and lower limb), wt. 75, measure the IV fluid this patient need?
Answer: 72% of body is burned = 21.6 L Fluid Requirements, 1st 24 Hours = 10.8L
Fluid Requirements, 1st 8 Hours (1/2 of Total)
Parkland formula: Initial 24 hours: Ringer’s lactate (RL) solution 4 ml/kg/% burn for adults and 3 ml/kg/% burn for children. RL
solution is added for maintenance for children:
● 4 ml/kg/hour for children weighing 0–10 kg
● 40 ml/hour +2 ml/hour for children weighing 10–20 kg
● 60 ml/hour + 1 ml/kg/hour for children weighing 20 kg or higher
This formula recommends no colloid in the initial 24 hours.
Next 24 hours: Colloids given as 20–60% of calculated plasma volume. No crystalloids. Glucose in water is added in amounts
required to maintain a urinary output of 0.5–1 ml/hour in adults and 1 ml/hour in children.
Link: http://www.ncbi.nlm.nih.gov/pmc/articles/PMC3038406/

Fluid Requirements = 4ml x TBSA burned (%) x Weight (kg)


Give 1/2 of total requirements in 1st 8 hours, then give 2nd half over next 16 hours.

25. Patient complaining of pain in the 2nd digit. Which tendon is affected?
Answer: Extensor indices

26. Something about post facial surgery, patient developed some neurological
symptom, which branch is effected options were branches of the main nerves?
Answer: Sensory or Motor?? Q is messing imp info

27. Generalized body scaling? Surface area affected (70-60-50-90)

75
Pediatrics Surgery

76
1. Child was crying since 4 hours ago. He has fever and vomiting. On examination there was a mass on the RUQ. What is your
diagnosis?
A. Appendicitis
B. Gastroenteritis
C. Pancreatitis
D. Intussusception

Answer: D

2. Case scenario of intussusception, what is the gold standard investigation?


A. Abdominal CT
B. CXR
C. Barium study
D. Abdominal X-ray
Answer: C

3. 2 years old with crampy abdominal pain, bleeding per rectum. What is your next step?
a. Barium enema

4. Which one of these layers is involved in chagas and hirschsprung disease?


A. Muscularis externa
B. Mucosa
C. Submucosa

Answer: A
Hirschsprung disease (congenital megacolon) and Chagas disease have different etiologies, but both inhibit intestinal motility by
affecting the myenteric (Auerbach's) plexus located between the layers of the muscularis externa.
Reference: Berkeley coursehero.

5. 7 years old boy brought by his parents, he has nausea, severe vomiting for 20 minutes and now semi comatosed. The parents
mentioned that he has same episode two weeks ago for 5 minutes without deterioration in consciousness. On examination
there is right testicular mass that does not transilluminate with light. What is the best action to do ?
A. Radiology.
B. ESR.
C. Surgical exploration.

Answer: C
The most likely diagnosis is strangulated hernia
Once the diagnosis of a hernia is made, surgical repair (a herniotomy) will be performed.
Reference: http://www.rch.org.au/kidsinfo/fact_sheets/Inguinal_hernia/

6. One month Baby come with abdominal distension and constipation since birth what is the next step:
A. X-ray
B. Biopsy
C. Rectal manometry

ANSWER: A
Plain abdominal x-ray should always be performed initially for neonate with intestinal obstruction. The presence of air-fluid levels is
evidence of obstruction, but is non-specific. Rectal Biopsy is a gold standard for definitive diagnosis.

Http://bestpractice.bmj.com/best-practice/monograph/750/diagnosis/step-by-step.html

77
7. Patient came to you with history of ingestion of 2 safety pins what's your action? They didn't specify where.
A. Surgery
B. Follow up with x-ray
Answer: B
Objects with sharp edges or points present a special problem because of the possibility for erosion or perforation. These include
pins, needles, tacks, razor blades, pieces of glass, or open safety pins. Children who have swallowed such objects should be vigilantly
observed.
Medscape

8. 12 years old male with undescended testes. The surgeon excised it. What is the most likely histology?
A. Malignant transformation
B. Normal histology
Answer: B
6 % of testicular tumors develop in patients with a history of cryptorchidism (undescended testes).
th
Reference: Surgical Recall, 6 edition

9. child with blue dot in testis and painful mass in inguinal area?
A. Incarcerated hernia
B. testicular appendix torsion
Answer: B

10. pic of bowel obstruction with baby 2 week what is the treatment?
A. primary surgery
B. leavler surgery
Answer: A

11. mass moving with tongue protrusion?


A. Thyroglossal cyst
Answer: A

12. Treatment of hirschsprung disease


A. Surgical repair
Answer: A
th
Reference: Surgical Recall, 6 edition

Answer: A (It’s a case of intussusception, there were more details in the history)

13. Child was diagnosed to have meckel's diverticulae. What is the best diagnostic test?
Answer: ?
Meckel’s scan (technetium-99m pertechnetate scanning)
Meckel's scan has a sensitivity of 85 to 97 percent in pediatric patients
Reference: Uptodate

14. Case: battery lodged at esophagus what to do:


A. Urgent endoscopy?

15. Meckel’s diverticulum?

Http://emedicine.medscape.com/article/931229-overview
Meckel diverticulum (also referred to as Meckel's diverticulum) is the most common congenital abnormality of the small intestine; it
is caused by an incomplete obliteration of the vitelline duct (ie, omphalomesenteric duct).

78
Urology

79
1. Patient came with Lt flank pain radiates to groin no fever no change in urine color or amount no sign other than tenderness
this pain most likely:
A. Ureteric stone
B. Renal colic
C. Diverticulitis

Answer : B
The site of obstruction determines the location of pain. Upper ureteral or renal pelvic lesions lead to flank pain or tenderness,
whereas lower ureteral obstruction causes pain that may radiate to the ipsilateral testicle or labia.
Costovertebral angle tenderness on the affected side is common.
Reference: http://www.merckmanuals.com/home/kidney-and-urinary-tract-disorders/symptoms-of-kidney-and-urinary-tract-
disorders/flank-pain

The majority of renal calculi contain calcium. The pain generated by renal colic is primarily caused by dilation, stretching, and spasm
because of the acute ureteral obstruction. The classic presentation for a patient with acute renal colic is the sudden onset of severe
pain originating in the flank and radiating inferiorly and anteriorly; at least 50% of patients will also have nausea and vomiting.

Http://emedicine.medscape.com/article/437096-overview

2. 60 years old male presenting with flank pain, paraspinal mass, hematuria, weight loss O/E palpable firm mass in the lumbar
area, BP 150/90 What is the best initial investigation? (Clear case of RCC, with the RCC triad)?
A. US
B. CT
C. MRI
D. Radionucliotide

Answer: B- CT
Renal cell carcinoma:
The classic triad of RCC (flank pain, hematuria, and a palpable abdominal renal mass) occurs in at most 9 percent of patients; when
present, it strongly suggests locally advanced disease.
Radiographic testing — The usual first test is abdominal computed tomography (CT) or occasionally abdominal ultrasound.

Http://emedicine.medscape.com/article/281340-clinical
Medscape: Contrast-enhanced CT scanning has become the imaging procedure of choice for diagnosis and staging of renal cell
cancer and has virtually replaced excretory urography and renal ultrasonography.

3. A 50-year-old male diabetic and hypertensive, unable to maintain an erection, started on Phosphodiesterase type 5 inhibitors
(PDE5Is)
Which one of the following drug classes should be avoided?
A) Steroids
B) Antibiotics
C) Narcotics
D) Nitrates
Answer: D
PDE5Is prevent the breakdown of cgmp. Nitric oxide donors (ie, nitrates) increase the production of cgmp. Because both PDE5Is and
nitrates increase cgmp, coadministration can generate excess accumulation of cgmp and can trigger marked vasodilation and severe
hypotension.
Link: http://circ.ahajournals.org/content/122/1/88.full

4. Urge incontinence what is best to do?


A) medical

80
B) Surgical
C) Medical and surgical
D) Behavioral and physiotherapy of bladder
Answer: D
INITIAL TREATMENT — Initial treatment includes lifestyle modifications and pelvic floor muscle exercise for all patients with urinary
incontinence (stress, urgency, or mixed), along with bladder training in women with urgency incontinence and for some women with
stress incontinence
We typically treat with these conservative therapies for six weeks before considering subsequent therapies.

Lifestyle modification — We suggest weight loss to improve symptoms of urinary incontinence in obese women. Other lifestyle
modifications generally focus on other dietary changes. These have been less studied but are often helpful to alleviate symptoms.

 Weight loss – Obesity is a known risk factor for urinary incontinence, and weight loss in obese women appears to improve
symptoms of urinary incontinence
 Dietary changes – Some beverages may exacerbate symptoms of urinary incontinence. We ask patients to reduce
consumption of alcoholic, caffeinated, and carbonated beverages
 Constipation – Constipation can exacerbate urinary incontinence and increase the risks of urinary retention [17].
Constipation should be managed and avoided when possible
 Smoking cessation – Smoking has been associated with an increased risk for urinary incontinence [16,18]. However, no
studies have evaluated whether smoking cessation decreases urinary incontinence.
Pelvic floor muscle exercises (Kegel exercises) — We suggest pelvic floor muscle exercises for all women with urinary incontinence.
In women who are able to isolate their pelvic floor muscles to stop urine flow, verbal instruction on timing and frequency of exercise
is usually sufficient. For those with difficulty identifying the proper muscles, supplemental modalities can help women to perform
these exercises properly.

Bladder training — Bladder training is most effective for women with urgency incontinence [21]. Some women who have stress
incontinence only at higher bladder volumes may also benefit from the timed voiding component to keep bladder volumes below
that where stress incontinence occurs. Bladder training and pelvic muscle exercises are often used in combination.
Bladder training starts with timed voiding. Patients should keep a voiding diary to identify their shortest voiding interval. They are
then instructed to void by the clock at regular intervals using the shortest interval between voids identified on their voiding diary as
the initial voiding interval. Urgency between voiding is controlled with either distraction or relaxation techniques (eg, performing
mental math, deep breathing, or by quick contractions of the pelvic floor muscles "quick flicks") [4]. When the patient can go two
days without leakage, the time between scheduled voids is increased. The intervals are gradually increased until the patient is
voiding every three to four hours without urinary incontinence or frequent urgency.

URGENCY INCONTINENCE/OVERACTIVE BLADDER — If initial treatment of urgency, urgency-predominant mixed urinary


incontinence, or overactive bladder (OAB) with incontinence is ineffective, then we suggest a trial of pharmacologic therapy. Women
who fail initial and pharmacologic therapy should be referred to a specialist to consider other options

Pharmacologic therapy — Antimuscarinics are appropriate for women with urgency incontinence symptoms who do not experience
enough improvement with initial treatments. For women who cannot tolerate or do not have sufficient improvement on
antimuscarinics, beta-adrenergic therapy (mirabegron) may be an option. The combination of medication with behavioral therapy is
more effective than either alone, but must be balanced against costs and side effects

5. 46 y. O. Woman with urine incontinence that makes her wake up at night but she urinates herself before going to the
bathroom, what type of incontinence is this?
A. Urge
B. Overflow
C. Stress
Answer: A

81
6. Patient male with urine incontinence, go to the bathroom many times with urine retention. On examination, the bladder was
palpable, what is the incontinence type?
A- Urge
B- Stress
C- Mixed
D- Overflow

Answer: D
Explanation: Overflow incontinence is dribbling of urine from an overly full bladder. Volume is usually small, but leaks may be
constant. Overflow incontinence is the 2nd most common type of incontinence in men. Obstruction leads to a chronically
overdistended bladder, which loses its ability to contract; then it does not empty completely, resulting in overflow.

Reference: https://www.msdmanuals.com/professional/genitourinary-disorders/voiding-disorders/urinary-incontinence-in-adults
7. urge and stress incontinence treatment.

Reference: Toronto Notes.

Stress Incontinence:
With this type, urine leaks due to weakened pelvic floor muscles and tissues. It can happen when pressure on your bladder
increases -- such as when you exercise, laugh, sneeze, or cough.
Urge Incontinence:
This is also called overactive bladder (OAB). With this type, you have an urgent need to go to the bathroom and may not get
there in time.
Overflow Incontinence:
If you can't empty your bladder, you may have overflow incontinence. This means you may dribble urine.
82
8. Urine retention and bilateral hydronephrosis ?
A. Urethral Meatal..
B. Bladder cancer
C. Prostatic cancer
D. Pelvic tumor
Answer: C

Obstruction at or distal to the renal pelvis causes diffuse caliectasis or hydronephrosis. Ureteral obstruction may result from stones,
transitional cell carcinoma, external compression (tumors, enlarged lymph nodes, retroperitoneal fibrosis), blood clots, and fungus
balls. When obstruction has been relieved with a stent, obstruction of the stent can lead to recurrent hydronephrosis.
Bladder tumors can obstruct one or both ureters or ureteral orifices. The most common causes of bilateral obstruction include
bladder outlet obstruction (from prostatic enlargement or posterior urethral valves) and neurogenic bladder. In women, uterine,
cervical, or ovarian cancer should be considered when unilateral or bilateral hydronephrosis is detected.

9. X-ray showed opacity at L2 or L4. Stone at what level:


A- Ureteric
B- Minor calyces
C- Major calyces
D- Renal pelvis
Answer: A
The kidneys are paired retroperitoneal structures that are normally located between the transverse processes of T12-L3 vertebrae,
with the left kidney typically somewhat more superior in position than the right.
The ureter begins its descent to the bladder by running along the medial aspect of the psoas muscle. Here, the ureter lies anteriorly
and slightly medial to the tips of the L2-L5 transverse processes.
It enters the pelvis anteriorly to the sacroiliac joint at the bifurcation of the common iliac vessels (at the pelvic brim) and then
courses anteriorly to the internal iliac artery down the lateral pelvic sidewall.
At the level of the ischial spine it turns forward and medially to enter the posterolateral wall of the bladder, where it runs an oblique
1-2 cm course, before opening into the bladder at the internal ureteric orifice

10. Elderly male, presenting with fever (38.5), dysuria, exam was normal EXCEPT DRE which showed tender enlarged prostate, no
penile secretions, no flank pain, hemodynamically stable, normal level of consciousness. Labs: UA showed 20 WBC +ve citrate
only. Diagnosis?
A- Pyelonephritis
B- Acute Prostatitis
C- Cystitis
D- Gonorrhea
Answer: B

Acute prostatitis clinical presentation and diagnosis: “The clinical presentation of acute prostatitis is generally not subtle. Patients
are typically acutely ill, with spiking fever, chills, malaise, myalgia, dysuria, irritative urinary symptoms (frequency, urgency, urge
incontinence), pelvic or perineal pain, and cloudy urine. Men may also complain of pain at the tip of the penis. Swelling of the
acutely inflamed prostate can cause voiding symptoms, ranging from dribbling and hesitancy to acute urinary retention. Rarely,
patients lack these local symptoms and present instead with constitutional symptoms or a flu-like illness.
On exam, the prostate is often warm, firm, edematous, and exquisitely tender. Common laboratory findings include
peripheral leukocytosis, pyuria, bacteriuria, and, occasionally, positive blood cultures. Inflammatory markers (erythrocyte
sedimentation rate, C-reactive protein) are elevated in most cases. Inflammation of the prostate can also lead to an elevated serum
prostate specific antigen (PSA) level. Thus, if serum PSA testing for prostate cancer screening is planned, it should be deferred for
one month following resolution of acute prostatitis.

Explanation: boggy prostate is prostatitis. Stony hard with obliteration of median sulcus indicates prostate cancer. Rubbery prostate
with uniform enlargement is BPH.

83
Reference First aid USMLE step 1

11. Which of the following renal stones is associated with infection and alkaline urine?
A. Calcium oxalate.
B. Uric acid.
C. Cysteine.
D. Struvite.
Answer: D
Magnesium ammonium phosphate (MAP)
Alkaline urinary ph due to infection with urea-splitting organisms precipitates MAP
Proteus, Pseudomonas, Providencia, Klebsiella, Mycoplasma, Serratia, S. Aureus, NOT E. Coli
Perpetuate UTI because stone itself harbours organism, therefore must remove stone to cure
Infection stone and all foreign bodies must be cleared to avoid recurrence associated with staghorn
Calculi.
Reference: Toronto Notes
Reference: First aid:

Child or neonate diagnosed with UTI which next step before treatment?
A- Urethral culture
B- Cystoscopy
C- Catheter drainage
D- Renal US

Answer: D (if already the patient diagnosed with UA and culture, and the patient is febrile)
- Diagnosis established if urinalysis suggests infection AND if ≥50,000 colony-forming units per of ml a
uropathogen cultured.
- Renal and bladder U/S for all febrile infants with utis looking for anatomical abnormalities, hydronephrosis,
abscess.
Reference:
- Toronto Notes (P 1088)
- Medscape: http://emedicine.medscape.com/article/969643-workup

84
12. You did rectal examination to someone and found there is a mass in front of anterior rectal wall which was diagnosed as
adenoma. What is the location of this neoplasm?
A. Anterior
B. Posterior
C. Lateral
D. Median

Answer: B
The prostate is divided into lobes:
● The anterior lobe is the portion of the gland that lies in front of the urethra. It contains no
glandular tissue but is made up completely of fibromuscular tissue.
● The median or middle lobe is situated between the two ejaculatory ducts and the urethra.
● The lateral lobes make up the main mass of the prostate. They are divided into a right and left lobe
and are separated by the prostatic urethra.
● The posterior lobe is the medial part of the lateral lobes and can be palpated through the rectum
during digital rectal exam (DRE).
Reference: http://www.cdc.gov/cancer/npcr/pdf/abstracting/prostate.pdf

13. A patient had prostate CA and it was removed. Now he presented with metastasis. What is the most probable location?
A. Scalp
B. Brain
C. Bone
D. Lung

Answer: C
The commonest site of metastasis for prostate CA are: Osteoblastic bony lesions, lung, liver, adrenal.
th
Reference: Surgical Recall, 6 edition.
Also “Two triangle Rule that love bone. Thyroid, Breast. Kidneys and Prostate“

14. Patient is having infertility came to infertility clinic. He is complaining of testicular pain. On examination the testicle is
swelled like a bag of worms. What is the diagnosis?
A. Varicocele.
B. Spermatocele.
C. Hydrocele.
D. Testicular cancer.
Answer: A
 Palpating a varicocele can be likened to feeling a bag of worms
 A varicocele is an enlargement of the veins within the scrotum. A varicocele is similar to a varicose vein that can occur
in the leg.
 Varicoceles are a common cause of low sperm production and decreased sperm quality, which can cause infertility
Reference: http://www.mayoclinic.org/diseases-conditions/varicocele/basics/tests-diagnosis/con-20024164

85
Explanation: Varicocele is scrotal varicose veins and is the most common cause of scrotal enlargement in adult males.
Reference Master the board step 2CK

15. Women with dysuria routine urine analysis show epithelial cell diagnosis, What is the source of these cells?
A. Cervical erosion
B. Chlamydia urethritis
C. Vaginal contamination
D. Renal Stone

Answer: C
Women suffering from vaginal infections caused by bacteria may show large amounts of squamous epithelial cells in their urine.
These cells mostly come with bacteria attached and are referred to as clue cells.
Reference: http://www.healthcare-online.org/Epithelial-Cells-in-Urine.html

16. Case of BPH, while prostatectomy, the doctor injured prostatic nerve, this patient will have :
A. Erection inability
B. Urine incontinence
C. Stress incontinence
D. Stool incontinence
Answer: A
Erectile Dysfunction then Retrograde ejaculation. (Answered by a urologist).
17. Stupid question; patient present with flank pain, fever, hematuria, how to prevent the condition?
A. Vit D
B. Calcium
C. Bisphosphonate
D. mannose?
Answer:
If the question is about renal stone we need to control their levels by hydrochlothiazide, which reduces ca in urine.

18. A patient is complaining of recurrent urinary tract infection. Ultrasound revealed kidney stones…. Organism with swarming
motility, more details I don’t remember, what is the most likely organism?
A. Proteus mirabilis
B. Pseudomonas
C. E coli
Answer: A
Struvite stones account for 15% of renal calculi. They are associated with chronic urinary tract infection (UTI) with gram-
negative, urease-positive organisms that split urea into ammonia, which then combines with phosphate and magnesium to
crystalize into a calculus. Usual organisms include Proteus, Pseudomonas, and Klebsiella species
Http://emedicine.medscape.com/article/437096-overview#a5

19. Patient with Rt kidney 14 Cm and left kidney 7 cm .. Arteriography: renal artery stenosis,what to do
A. CT angio
B. Ct abdomen
C. Biopsy
Answer : ??

20. Long Scenario about old male bedridden on foly's catheter he developed Gram -ve bacteria what is the organism
A. E.coli
B. Pseudomonas aergonsa
C. Strep. Puomonia

86
Answer: a
Enteric pathogens (eg, Escherichiacoli) are most commonly responsible
( http://emedicine.medscape.com/article/2040035-overview )
21. Patient with recurrent UTI. Bilateral hydronephrosis. Renal biopsy showed multiple cysts with some description, what is the
diagnosis?
A-Renal cell carcinoma.
B-Polycystic kidney disease.
C-Renal dysplasia.

Answer: B
Approximately 30 to 50 percent of patients with autosomal dominant polycystic kidney disease (ADPKD) will have a urinary tract
infection (UTI) during their lifetime.

Reference: http://www.uptodate.com/contents/urinary-tract-infection-in-autosomal-dominant-polycystic-kidney-disease

22. Patient with adrenal mass on examination. What is the next step before you do FNA?
A. Renin/aldosterone ratio
B. Dexamethasone stress test
C. Free metanephrines

Answer: C
Pheochromocytoma should always be suspected to avoid the risk of lethal hypertensive crises, especially during biopsy or surgery.
Plasma-free metanephrines provide the best test for excluding or confirming pheochromocytoma.
Reference: http://www.medscape.com/viewarticle/442383_3

23. A case of pyelonephritis, what is the next step?


A. Admit and give antibiotics
B. Do investigations
C. Give him antibiotics at home
Answer: scenario not clear.
Diagnosis is usually suggested by the history and physical examination and supported by urinalysis results. Imaging studies are rarely
indicated for the diagnosis of acute pyelonephritis in an adult who presents with typical signs and symptoms. Ambulatory younger
women may be treated in an outpatients setting, pregnant and severely ill patients must be admitted.

24. Man done vasectomy, change his mind and want to reproduce again, they found antisperm antibodies what is the cause?

A. Antigen release or something like that?


B. Cross reactivity with viral infection
C. Inappropriate response of MHC II to antigen presenting cell

Answer: B
Causes of antisperm antibodies are: Trauma, chemical injury, and infection
The above mentioned causes lead to the formation of antisperm antibodies because they lead to the breakage of the blood-testis
barrier, when this barrier is breached, sperm antigens escape from their immunologically protected environment and come in direct
contact with blood elements that launch the antisperm antibodies.
Reference: http://www.newyorkfertilityservices.com/causes-of-infertility/anti-sperm-antibodies/
25. Patient is having unilateral testicular pain and fever. What is the diagnosis?
A. Epidedmo-orchitis.
B. Prostatitis.
C. Testicular torsion.

87
Answer: A
- Epididymitis and epididymo-orchitis are usually caused by a bacterial infection from surgery, urinary catheter, utis, or STD.
Sometimes there is no infection of any kind
- Symptoms include swelling and tenderness, pain, hydrocele and fever.
- Diagnosed by physical examination, urinalysis, and sometimes Doppler ultrasonography
Reference: http://www.merckmanuals.com/home/men-s-health-issues/penile-and-testicular-disorders/epididymitis-and-
epididymo-orchitis

26. Testicular lymph node drainage? ( I am not sure is it testicular or scrotum )


A. Left aortic lymph node
B. Superficial inguinal lymph node
C. Deep inguinal lymph node

Answer: A
- Testis: to para-aortic lymph nodes
Lymphatic drainage of the testis follows the testicular vessels (in the spermatic cord) to the right and left lumbar (caval/aortic) and
preaortic lymph nodes at the second lumbar level.
- scrotal: to superficial inguinal lymph nodes.
Reference: http://teachmeanatomy.info/pelvis/the-male-reproductive-system/testes-epididymis/

27. Best screening method for prostate cancer?


A. Digital rectal examination
B. Cytology for prostate cancer
C. PSA

Answer: C
Digital rectal exam is not as effective as PSA
Reference: http://www.uptodate.com/contents/prostate-cancer-screening-beyond-the-basics

28. 65 patient of difficulty in starting urine and dribbling >PR is normal >PSA is normal >US show medial lobe enlargement = how
to approach
A. Annual Renal function test
B. Repetitive PSA
C. Cystoscope
Answer: C

29. Old patient with back pain and difficulty in urination.


Investigations:
liver is normal
High AlP
High PSA
What's the diagnosis?
A. Prostaitis
B. Prostate cancer
C. bPH
Answer: B (Prostate cancer with bone metastasis most likely)

Explanation: With a PSA level of 4-10 ng/ml, the likelihood of finding prostate cancer is about 25%; with a level above 10 ng/ml, the
likelihood is much higher. Skeletal manifestations are common in metastatic prostate cancer. Elevated alkaline phosphatase aids in
the diagnosis of metastatic bone disease.
Reference: http://emedicine.medscape.com/article/1967731-overview

88
30. Rx uncomplicated 5 yrs cystitis
A. IM Ceftriaxone
B. IV copra
C. Oral….
Answer: C
Children with cystitis usually do not require special medical care other than appropriate antibiotic therapy. A 4-day course of an oral
antibiotic agent is recommended for the treatment of cystitis. Antibiotic agents used include Sulfamethoxazole and trimethoprim
(SMZ-TMP), Amoxicillin and clavulanic acid, Cephalexin, Cefixime, Cefpodoxime, Nitrofurantoin.
Reference: http://emedicine.medscape.com/article/969643-treatment#d12

31. Prostate cancer marker:


A. Carcinoembryonic antigen
B. Alpha fetoprotein
C. Acid phosphatase
Answer: C
Explanation: In this questions C is the right answer. However in general prostate specific antiget (PSA) is a better answer if given in
the answers since acid phosphatase is an old marker that is now widely replaces by PSA
Reference http://www.ncbi.nlm.nih.gov/pubmed/2431533

32. Abdominal solid mass (renal I guess but not sure ) confirmed by?
A. CT
B. MRI
C. US
Answer: C
US detect renal mass, CT for perforation ((Step-Up to Medicine, 4E,p300))

33. To decrease UTI recurrence?


A - decrease urea, decrease PH, increase osmolarity
B - increase urea, decrease PH, increase osmolarity
Answer : B

Biochemical properties are normally important in making bacterial survival difficult in urine: acid ph, high urea content, and high
osmolality.
Https://www.auanet.org/education/adult-uti.cfm

34. Treat HTN and BPH??


A. Atenolol
B. Prazosin
Alpha blockers: These medications relax the muscle of the prostate and bladder neck, which allows urine to flow more easily. There
are at least five medications in this category: terazosin (Hytrin), doxazosin (Cardura), tamsulosin (Flomax), alfuzosin (Uroxatral), and
silodosin (Rapaflo). Terazosin and doxazosin were initially developed to treat high blood pressure, but were later found to be useful
for men with BPH.
Http://www.uptodate.com/contents/benign-prostatic-hyperplasia-bph-beyond-the-basics

35. A patient who has hematuria was diagnosed with renal stones. What is the most likely organism?
A. Schistosoma haematobium
B. E coli
Answer: B

89
36. Which testicular tumor is radio sensitive?
A- Seminoma
B- Teratology
Answer: A
Seminoma is extremely radiosensitive, and low-stage disease is treated with radiation to the inguinal and retroperitoneal areas. Men
with nonseminomatous germ cell tumors of the testis can be managed with observation, chemotherapy or RPLND.
Link: http://www.aafp.org/afp/1999/0501/p2539.html

37. Patient with back pain and signs of retention, what to do first?
A. PSA
B. Urodynamic

ACUTE MANAGEMENT — The initial management of acute urinary retention (AUR) is prompatient bladder decompression by
catheterization.
Options for bladder decompression — Bladder decompression can be accomplished with urethral or suprapubic catheterization.
There are no uniform guidelines for bladder decompression. Most patients will have an initial attempatient at urethral
catheterization.
Urethral catheterization — An initial attempatient at urethral catheterization is appropriate for most patients, particularly in
patients for whom AUR is expected to resolve (eg, patients with urinary tract infections or AUR secondary to medication effect).
Urethral catheterization is contraindicated in patients who have had recent urologic surgery (eg, radical prostatectomy or urethral
reconstruction) and these patients should have suprapubic catheterization. Although there is a theoretical risk to placement of a
urethral catheter in the setting of acute bacterial prostatitis, these patients may have an attempatient at gentle urethral
catheterization by an experienced clinician.

Indwelling catheter — A 14 to 18 gauge French catheter should be inserted as first-line therapy in most patients with AUR [15].
Some patients may have an obstruction that does not readily allow passage of the catheter. If the patient has had a prior
transurethral procedure (eg, transurethral resection of the prostate), a partially obstructing urethral or prostatic scar may be
present. In this case, the obstruction may be bypassed by downsizing the catheter to a 10 or 12 gauge French indwelling catheter.

Suprapubic catheter — Placement of a suprapubic (SP) catheter is sometimes necessary in patients who have contraindications to or
fail urethral catheterization (eg, those with recent urologic surgery, acute prostatitis, urethral stricture disease, severe benign
prostatic hyperplasia (BPH), or other anatomic abnormalities)

38. Patient has difficulty to urinate and back pain. High prostatic enzyme. Most likely diagnosis?
A. BPH
B. Prostatic cancer
Answer: B

Symptoms — Most men with early stage prostate cancer have no symptoms attributable to the cancer.
 Urinary frequency, urgency, nocturia, and hesitancy are seen commonly but are usually related to a concomitant benign
prostate enlargement.
 Hematuria and hematospermia are uncommon presentations of prostate cancer but their presence in older men should
prompatient consideration of prostate cancer in the differential diagnosis. These symptoms are also present in men with
benign prostatic hyperplasia (BPH) and are more likely to be caused by BPH than cancer.
 Bone pain may be the presenting symptom in men with metastatic disease but an initial diagnosis when bone metastases
are present has become unusual
Men with abnormal prostate exams (nodules, induration, or asymmetry) should be referred to a urologist for a prostate biopsy, with
a the histologic diagnosis based upon tissue obtained from the biopsy. A prostate biopsy may also be indicated based upon
abnormal PSA values.

90
39. 2 YO with scrotal cyst?
A. Spermatocele
B. Hydrocele
Answer: B

40. Prostatic ca w/ back pain investigation?


A. Back sincitograph
B. Back CT
Answer: B
He has mets so it should be CT Toronto notes
Http://emedicine.medscape.com/article/379996-overview#showall

41. Old male patient presented with urgency and frequncy symptoms whats the first thing that u will do ?
A- PSA
B- transrectal US
Answer: ???

Lower urinary tract symptoms may be divided into voiding and storage, and men may present with a combination of the two
symptom groups. Voiding symptoms include weak stream, hesitancy, and incomplete emptying or straining and are usually due to
enlargement of the prostate gland. Storage symptoms include frequency, urgency and nocturia and may be due to detrusor
overactivity.
Http://www.racgp.org.au/download/documents/AFP/2011/October/201110arianayagam.pdf

42. What is the most common part of urethra could be injured during catheterization?
A. Membranous urethra
B. Prostatic urethra
C. Penile urethra
Answer A (narrowest part)
Urethral injury due to trauma is a common complication associated with Foley catheters especially if the catheter is inserted
forcefully or roughly. A significant urethral injury can even cause death. (http://patientmodesty.org/urinarycatheterizationrisks.aspx)
^Urology Resident

43. Child develop straddle injury after playing with bicycle presented with scrotal and penile hematoma “I think” which structure
would probably be injured:
A. Penile urethra
B. Prostatic urethra
C. Membranous urethra
Answer: C

^Urology Resident

The membranous urethra is prone to injury from pelvic fracture. The bulbar urethra is susceptible to blunt force injuries because of
its path along the perineum. Straddle-type injuries from falls or kicks to the perineal area can result in bulbar trauma. Conversely,
the penile urethra is less likely to be injured from external violence because of its mobility, but iatrogenic injury from catheterization
or manipulation can occur.
Reference: http://emedicine.medscape.com/article/451797-overview#a11

91
44. Cancer associated with Smoking?
A. Bladder
B. Colorectal
th
Answer: A. Reference: Surgical Recall, 6 edition

45. Old patient with COPD, DM and now he’s diagnosed to have BPH, what medication will you use?
A. B-Blocker
B. Terazosin

Answer: B
Terazosin is an alpha blocker
46. The narrowest part of male urethra is?
A. Membranous urethra.
B. Penile urethra.
Answer: A
membranous urethra (1 cm long): passes through the urogenital diaphragm, surrounded by sphincter urethra the shortest and
narrowest portion.
Reference:
Http://radiopaedia.org/articles/male-urethra

47. bladder ca surgical removed and the area of cancer now clear and no invasion of the muscle
A. Follow up with cystoscopy and biopsy
B. Intravesical chemotherapy..

Answer: B
Reference : uptodate + toronto notes
http://www.uptodate.com/contents/overview-of-the-initial-approach-and-management-of-urothelial-bladder-
cancer?Source=outline_link&view=text&anchor=H3#H3

48. Patient with hematuria diagnosed bladder cancer did resection what is the next step ?
A. BCG intrbladder
B. Mitomycin-C

Answer: B
Within the first 24 hours, a single intravesical instillation of (40 mg in 20 ml of saline) has been shown to reduce the frequency of
tumor recurrence.
Reference: http://emedicine.medscape.com/article/1951622-overview#a4

49. 3 years old boy with UTI,, what you’ll do before starting treatment (Ascending urethrogram was not an option)
A. -US
B. -culture urethra !!

Answer: (the answer should be: send urine culture)


 -Urine culture should be done beside urine analysis , but do not delay antibiotic.
- Renal ultrasonography should be considered for any child with a first febrile UTI in whom good follow-up cannot be
ensured.
- Blood cultures are not recommended .
Source :
http://emedicine.medscape.com/article/969643-workup

92
Toronto note :
U/s in recommended for all febrile "infants" with UTI
uptodate : US is done if :
• Child with recurrent UTI at any age
• Children younger than 2 yrs with a first febrile UTI
• Children who dont respond as expected to appropreate ttt .
• Children of any age with with a UTI who have family history of renal or urologic disease , poor growth or HTN.

50. Non-invasive papillary tumor of the bladder, size 2 cm ( more details ). What is the next step?
A. Intra-vesical BCG
B. Radical
Answer: A
Explanation: non-invasive papillary carcinoma of the bladder is staged Ta. The management of which is intra-vesicle BCG.
Reference: http://emedicine.medscape.com/article/438262-treatment#d10

51. Why to use inulin to estimate GFR? Active biological


A. Passive free in glumurs
Inulin is useful as an indicator of GFR because the kidneys handle it in a unique way. Unlike most other substances in the blood,
inulin is neither reabsorbed into the blood after filtration nor secreted through peritubular capillaries. Thus, the amount of inulin
cleared through the urine is indicative of the amount of plasma filtered by the body’s glomeruli.
Https://www.khanacademy.org/test-prep/mcat/biological-sciences-practice/biological-sciences-practice-tut/e/measuring-
glomerular-filtration-rate-of-the-kidneys-with-inulin
st
+ Davidson, 21 edition page 464

52. Patient on chemotherapy Prevention of hemorrhagic cystitis:


Answer: Mesna
The best treatment for HC is prevention. Effective prophylactic regimens include continuous bladder irrigation, suprahydration, and
parenteral or oral mesna (2-mercaptoethane sodium sulfonate, a sulfhydryl compound). However, once HC is established, the
optimal treatment depends upon the degree or grade of hematuria

53. Patient presented with non-traumatic acute urinary retention. How you will treat the patient?
a. Foley catheter

Answer: A
Reference: UpToDate

54. Pt presented with urethral injury (already established) with blood coming out from the meatus, what will you do for him?
A. Insert suprapubic catheter.
Answer: ?
Rectal and pelvic examinations are of utmost importance during the initial evaluation to rule out the presence of an open fracture.
Blood in the vaginal vault or in the rectum should raise the level of suspicion for an open injury. Palpable bony spicules within the
rectum or vagina may be present indicating an open injury. A high-riding prostate may also be detected on rectal examination,
indicating the presence of a periurethral or periprosthetic hematoma occurring secondary to genitourinary injury >> Pelvic
stabilization for patients with pelvic fractures who are hemodynamically unstable. Reference:
http://www.sportssurgerynewyork.com/articles/diagnosis-management-pelvic-fractures.pdf

In any male patient with suggestive symptoms or signs urethral injury, the diagnosis is confirmed by retrograde urethrography. This
procedure should always precede catheterization. Urethral catheterization in a male with an undetected significant urethral injury

93
may potentiate urethral disruption (eg, convert a partial disruption to a complete disruption). Female patients require prompt
cystoscopy.

55. Elderly patient is having bilateral Hydronephrosis:


A. BPH

Answer: A There are a lot of causes check the link.


Reference: http://emedicine.medscape.com/article/436259-clinical#b5

56. A man presented with pain and mass in the flank. What is the diagnosis?
Answer: ?
DD of flank pain and mass:
● Renal abscesses: patient have pain because inflammation and edema produce stretch of renal capsule.
Sometimes, a flank mass may be palpable.
● Polycystic kidney disease.
● Renal vein thrombosis.
● Renal cell carcinoma (pain, mass and hematuria)
Reference: Medscape + rightdiagnosis

57. Pt with testicular mass after excision you took a sample and see it under microscope it showed (I can't recall) what is the type
of malignancy...? - seminoma - yolk sac -???
Answer: no enough information.
• Histology:
 Seminoma
Large cells in lobules with watery cytoplasm
"fried egg" analogous to dysgerminoma of the ovary
 Embryonal carcinoma
Glandular/papillary
 Yolk sac (endodermal sinus) tumor
Yellow, mucinous Schiller-Duval bodies resemble primitive glomeruli
 Choriocarcinoma
Disordered syncytiotrophoblastic and cytotrophoblastic elements
 Teratoma
Contain three tissue types
 Leydig cell
Contains Reinke crystals
Reference: usmlebullets.com
58. One cell type testicular mass, what is the diagnosis?
A. Seminoma

94
Answer: ?

Most common urologic diagnosis in men <50 yr. Prevalence 2-12%

* KEEPS Klebsiella sp.


E. Coli (90%), other Gram-negatives Enterococci
Proteus mirabilis, Pseudomonas
S. Saprophyticus

95
Reference: Toronto Notes

59. Renal artery stenosis, Angie was done , what is next ?


Since the renal artery stenosis is already detected by the angiography, there is no role for CT scan further this point. The next
step is to differentiate between atherosclerosis & the fibromuscular dysplasia since the management of them differ.
There is similar question with ( renal artery biopsy ) was one of the choices. I think it is the proper next step.
Angioplasty with stenting?

When renal function is normal or nearly normal, specialists recommend revascularization for prevention of renal
insufficiency if the patient meets the following criteria:
• The degree of stenosis is more than 80-85%
• The degree of stenosis is 50-80%, and captopril-enhanced scintigraphy demonstrates an activation of intrarenal renal
artery stenosis
Reference: Medscape

60. Pt presented to you with Rt scrotal swelling, he said that it is increasing in size day after day, on examination , it is not trans-
illuminate , what will you do ?
A. Reassure him and get surgical opinion

Answer: the question is missing important info


 Differential diagnosis in this case can be hematocele or testicular cancer and both of them need further evaluation by US
and depending on the results further investigations may be ordered.
Reference: https://www.merckmanuals.com/professional/genitourinary-disorders/symptoms-of-genitourinary-disorders/painless-
scrotal-mass

61. scrotal pain with dysuria


Answer: It is mostly a case of epididymo-orchitis:
1- it is's caused by STD ( gonorrhea or chlamydia ), treat with Ceftriaxone+ Azithromycin, OR with doxycycline .
2- if from urinary pathogens, fluoroquinolones are the preferred antibiotic, as they have excellent penetration into the
testes. (Trimethoprim-sulfamethoxazole OR Amoxicillin-clavulanate can be used)

Http://emedicine.medscape.com/article/2018356-overview

62. gene mutation of suprarenal tumor?

The Li-Fraumeni syndrome is caused by inherited mutations that inactivate the p53 tumor suppressor gene. This syndrome causes
few cases of adrenal cancer in adults (1 of every 20), but is often the cause of adrenal cancer in children. In fact, about 8 of every 10
cases of adrenal cancer in children are caused by Li-Fraumeni syndrome. Many other adrenal cancers have also been found to have
abnormal p53 genes that were acquired after birth (not inherited).
Http://www.cancer.org/cancer/adrenalcorticalcancer/detailedguide/adrenal-cortical-cancer-what-causes
Mutations in the VHL gene cause von Hippel-Lindau syndrome.
Http://ghr.nlm.nih.gov/condition/von-hippel-lindau-syndrome
Many different syndromes

96
63. Patient diagnose with renal cell carcinoma which gene do u think have mutation?
A. PKD
B. PKHD-1
C. VHL

Answer: C
Referance : http://emedicine.medscape.com/article/281340-overview#a3

64. Kidney tumor oncogene?


A. P53
Answer:
-MET proto oncogene responsible for hereditary papillary renal cell cancer
Ref : PubMed
http://www.ncbi.nlm.nih.gov/pmc/articles/PMC2929006/

-Alteration of p3 chromosome result in alteration of either tumor suppressor gene ( VHL , TSC ) or oncogene ( MET )
Ref : Medscape
http://emedicine.medscape.com/article/281340-overview#a2

65. A patient presented with trauma on the right side of abdomen, he then developed abdominal swelling and hematuria?
A. Wilms tumor
Explanation: Wilms tumor is the most common primary renal neoplasm of childhood & most common abdominal mass in children.
80% with unilateral abdominal mass. They may also develop hypertension, gross hematuria, abdominal pain and vomiting.
Reference Toronto notes

66. IHD or (DM + HTN) with BPH ? Medication? The choices were: 2 types of ACE 1 Beta Blocker 1 choice of I do not know the
optimal Rx is alpha blocker (prazocin etc).

Answer: Alpha blockers are preferred for such a scenario (HTN & BPH) (uptodate, Medscaper…etc). In practice, a “prostate”
selective alpha blocker like Tamsulsin is used for BPH along with 1st line Antihypertensive drugs for HTN (ACE/ARB, CCB, Thiazide.) If
no Alpha blockers in the answers, then avoid BB because they mask the symptoms of DM. ACEI would be the first choice especially
that they are first line in the new HTN guidelines (JNC8) plus the fact that they are useful for diabetic nephropathy, and they are not
diuretics (BPH).
67. Case of urine incontinence. What mechanism of action of oxybutynin?
Competitive antagonist of acetylcholine at postganglionic -muscarinic receptors

68. Patient diagnosed with papillary renal carcinoma, treatment?


Http://emedicine.medscape.com/article/281340-treatment

69. Patient with bladder cancer post-surgery what to inject?


BCG (bacillus Calmette-Guérin)
Reference: http://emedicine.medscape.com/article/1950803-overview

70. Man done vasectomy, change his mind want to reproduce again, they found antisperm antibodies what is the cause?
A. Antigen release or something like that?
B. Cross reactivity with viral infection
C. Inappropriate response of MHC II to antigen presenting cell
Answer: A
Antigen releasing
97
Urology resident
Http://www.medscape.com/viewarticle/571266_2

71. Ureteral stone what is the radiological study to detect it?


A. Abdominal ultrasound
B. CT with contrast
C. CT without contrast
There was No x-ray in options
Answer: C
Bailyand love
Http://radiopaedia.org/articles/ureteric-calculi
Refrence: http://www.merckmanuals.com/home/kidney-and-urinary-tract-disorders/stones-in-the-urinary-tract/stones-in-the-
urinary-tract

72. Left side pain from lion to groin Dx:


A) Renal stone
B) Acute app
C) Acute pancreatitis
D) Duodenal ulcer
Answer: A
Reference: http://emedicine.medscape.com/article/437096-overview

73. Enuresis MCQ.

74. Vesicoureteral reflux, read about it.

98
ENT

99
1. Young female came to your office complaining of swelling in front of her parotid gland. No tenderness, no secretion coming
out, what is the most likely diagnosis?
A. Parotid ca
B. Sialadenitis
C. Mumps
D. LN enlargement

Answer: a
The most common presentation of parotid ca. Is a painless, asymptomatic mass; >80% of patients present because of a mass in the
posterior cheek region. Some people infected with the mumps virus have either no signs or symptoms or very mild ones. When signs
and symptoms do develop, they usually appear about two to three weeks after exposure to the virus and may include: swollen,
painful salivary glands on one or both sides of your face (parotitis).
Reference: medscape and myoclinic

2. How to assess hearing in adult:


A. Recording music
B. Recording dog parking
C. Sound of watch
D. Paper flabbing
Answer: c
Reference: aafm

3. Which part of the ear is responsible of angular acceleration:


A. Saccula
B. Macula
C. Organ of corti
D. Semilunar canal
Answer: d
http://neuroscience.uth.tmc.edu/s2/chapter10.html

The anterior and posterior semicircular canals are oriented vertically at right angles to each other. The lateral semicircular canal is
about 30-degree angle from the horizontal plane. The orientations of the canals cause a different canal to be stimulated by rotation
of the head in different planes. Thus, the horizontal canal detects angular acceleration of the head (such as when you spin in a
rotating chair), while the superior and posterior canals detect vertical head movements (such as when you bend forward to pick
something up from the floor).

4. What is the most sensitive part to the linear acceleration?


A. Semicircular canal.
B. Organ of corti.
C. Utricle.
D. Saccule.
Answer: c
Explanation: the otolith organs sense gravity and linear acceleration such as from due to initiation of movement in a straight line.
Persons or animals without otolith organs are imbalanced.. A set of hair cells are coupled to a mass of stones. When the stones

100
accelerate, with respect to the hairs, they exert a shearing force on the hairs. This force is detected by the hair cells and sent to the
brain via branches of the vestibular nerve. The utricle sends input to the brain via the superior division of the nerve, and the saccule,
via the inferior division. There is considerably more complexity to the organization of the utricle and saccule, including different
types of hair cells and detail to the sensory macule (patch of sensory cells) that we have omitted.
The otolithic organs sense motion according to their orientation. The utricle is largely horizontal in the head, and largely registers
accelerations acting in the horizontal plane of the head (called the axial plane by radiologists). The saccule is largely vertical, actually
parasagittal, in the head, and registers accelerations in the vertical plane (called parasaggital or coronal plane).
Otolithic organs= the saccule and utricle >> for linear acceleration
Reference: http://www.tchain.com/otoneurology/disorders/bppv/otoliths.html + medscape

5. Pregnant 32 year old came with hearing loss, her mother same condition what is the cause?!
A. Acoustic neuroma
B. Presb
C. Perforated tm
D. Otosclerosis
Answer: d?
Answer a is not supported with other presenting symptoms such as vertigo or tinnitus. Answer b is unlikely because of the patient’s
age. Answer c no risk factors such as recurrent otitis media to support this choice. Otosclerosis: fusion of stapes footplate to oval
window so it can't vibrate. It’s ad, more common in female; progress during pregnancy ( hormone responsive)
Reference: toronto note

6. 6 years old female underwent tonsillectomy and started bleeding 24 hours after surgery. You examined the surgical site and
found no abnormality. Her platelet count, bleeding time, platelet function, pt, aptt and clotting time are all normal, what test
confirms her diagnosis:
A. Fibrinogen
B. Plasminogen
C. Clot lysis test
D. Vwf
Answer: c

7. 17 years old male presented with hearing loss in the left ear (picture of conductive hearing loss audiogram). What is your
diagnosis?
A. Presbycusis
B. Otosclerosis
C. Interosseous
D. Otitis media

Answer: b
Explanation: otosclerosis is traditionally diagnosed by characteristic clinical findings, which include progressive conductive hearing
loss, a normal tympanic membrane, and no evidence of middle ear inflammation.

101
8. Patient with nasal sx. What is the "best" treatment for it ?
A. Steroid
B. Antihistamine.
C. decongestants
D. Ab

Answer: a
Explanation:
 Prevention with avoidance of the precipitating allergen:
• Close the windows and use air conditioning to avoid pollen.
• Get rid of animals to which the patient is allergic.
• Cover mattresses and pillows.
• Use air purifiers and dust filters.
 Intranasal corticosteroid sprays
 antihistamines: loratidine, clemastine, fexofenadine, brompheniramine
 intranasal anticholinergic medications: ipratropium
 desensitization to allergens that cannot be avoided

Reference: master the board


9. How to examine child ear? Pull pinna?
A. Inf backwards
B. Sup backward
C. Inf forward
D. Sup forward
Answer a?
B for adult
Explanation:
For children above 3 years: sup backward
For children under 3 years: inf backwards
Reference: http://www.atitesting.com/ati_next_gen/skillsmodules/content/physical-assessment-
child/equipment/ap_ear_nose_throat.html

10. Patient in his work expose to noise what type of hearing problem may has:
A. Conductive hearing loss
102
B. Non sensory hearing loss
C. Sensory hearing loss
D. Vomiting
Answer: c sensorineural hearing loss (noise induced snhl)
11. Picture of tonsillitis, what is the best drug for treatment?
A. Ceftriaxone.
B. Acyclovir.
C. Ampicillin.
D. Doxycycline.
answer: ?
Explanation: the best is penicillin or amoxicillin; penicillin allergic patient are treated with cephalexin if the reaction only rash , if
allergy is anaphylactic use clindamycin or a macrolides
Reference: master the board step 2 CK

12. Painful ear, fever with tm having hemorrhagic vesicles on pe. What’s the cause?
A. Otitis externa
B. Fungal
C. Streptococcus pyogenes
D. Pseudomonas
Answer :
This is called myringitis
(particularly streptococcus pneumoniae), or mycoplasma otitis media.
http://www.merckmanuals.com/home/ear,-nose,-and-throat-disorders/middle-ear-disorders/infectious-myringitis
Myringitis is a form of acute otitis media in which vesicles develop on the tympanic membrane>>> most common cause of acute
otitis media is streptococcus pneumonia. However, it is not hemorrhagic.
This article explained bollous hemorrhagic myringitis as a form of otitis externa, hence, the etiologic organism could be the same
(pseudomonas) (http://oto.sagepub.com/content/148/2/347.extract )
While this article state streptococcus pneumoniae being a possible organism for hemorrhagic vesicles
(http://www.sbccp.org.br/arquivos/oto-2013-02-bullous-hemorrhagic.pdf)
13. Young patient presented with fever, vertigo, nystagmus no hearing loss, for the past 2 days, cerebellum signs are present,
what is the diagnosis?
A. Central vertigo.
B. …… neuritis.
C. Menieres.
D. Bppv.
Answer from medscape : a- central vertigo .

14. 3 weeks post antibiotic treatment of acute otitis media reveals non erythematous tympanic membrane which doesn't move
with insufflation with fluid collection, what to do next?
A. Antibiotic
B. Watchful waiting
C. Decongestant
D. Anti-histamine
Answer: b
Otitis media with effusion usually resolve spontaneously within 3 months
http://patient.info/doctor/otitis-media-with-effusion

according to Toronto notes this is an “otitis media with effusion” the treatment for which is expectant waiting for resolution of the
effusion. No clinical evidence supports the use of antibiotics.

103
15. Most sensitive test for maxillary sinusitis?
A. CT
B. US
C. Transillumination
D. X-Ray

Answer: a
Explanation: ct scanning is the preferred imaging method for rhinosinusitis. A screening sinus ct scan is adequate for diagnosis and
less expensive than other methods but is necessary only in cases of treatment failure or chronic rhinosinusitis.
Reference: http://emedicine.medscape.com/article/232670-workup#c13

16. Scenario consistent with sialadenitis (the pain in the submandibular area ) they are asking about the location of obstruction :
A. Submandibular
B. Sublingual
C. Parotid
D. Submental
Answer:a
Ductal stone (mainly hydroxyapatite) in adults, sand/sludge in children, leading to chronic sialadenitis .80% in submandibular gland,
<20% in parotid gland, ~1% in sublingual gland. The clinical presentation pain and tenderness over involved gland ,intermittent
swelling related to meals and digital palpation reveals presence of calculus.

17. Patient with thyroid nodule and tortious dilated artery that compress the external laryngeal nerve, which of the following
will be affected
A. Tone of the voice
B. Lateral adduction of vocal cord
C. Abduction of vocal cord
D. Sensation above the vocal cord

Answer: a

18. Child with ear secretion with defenses and move pinna what is diagnosis ?
A. Otitis media
B. Cholesteatoma
C. Otitis externa
D. Foreign body
Answer: c

19. Man complaining of ear pain. On examination he feels pain when moving ear pinna and there is erythema of auditory canal
and normal tympanic membrane. What is the most likely diagnosis?
A. Otitis externa.
B. Otitis media.
C. Squamous cell carcinoma.

Answer: a
Explanation: Clinical features:
acute:
 Pain aggravated by movement of auricle (traction of pinna or pressure over tragus)
 Otorrhea (sticky yellow purulent discharge)
 Conductive hearing loss aural fullness 2o to obstruction of external canal by swelling andpurulent debris

104
 Posterior auricular lymphadenopathy
 Complicated OE exists if the pinna and/or the periauricular soft tissues are erythematous and
 Swollen

Reference: toronto notes 2015

20. Otalgia, fever, sore throat? What is the nerve that refers ear pain?
A. Glossopharyngeal
B. Vagus
C. Sphenopalatine
D. Nasopalatine
Answer : a

This is referred otalgia. The general ear region has a sensory innervation provided by four cranial nerves and two spinal segments.
Hence, pathology in other "non-ear" parts of the body innervated by these neural pathways may refer pain to the ear. These general
pathways are:
 Via trigeminal nerve [cranial nerve v]. Rarely, trigeminal neuralgia can cause earaches. Oral cavity carcinoma can also cause
referred ear pain via this pathway.
 Via facial nerve [cranial nerve vii]. This can come from the teeth, the temporomandibular joint (due to its close relation to the
ear canal), or the parotid gland.
 Via glossopharyngeal nerve [cranial nerve ix]. This comes from the oropharynx, and can be due to pharyngitis, pharyngeal
ulceration, tonsillitis, or to carcinoma of the oropharynx (base of tongue, soft palate, pharyngeal wall, tonsils).
 Via vagus nerve [cranial nerve x]. This can arise from the laryngopharynx in carcinoma of this area, or from
the esophagus in gerd.
 Via the second and third spinal segments, c2 and c3.

21. Nerve can cause problems in eye and ear ?


A. Oclouomotor 3
B. Trochlear 4
C. Facial 7
D. Abducent 6
Answer: c
Motor innervation to 1) stapedius muscle (adjusts stapedius bone in response to loud sounds) 2) posterior belly of digastric muscle
3) muscles of facial expression (buccinator, platysma, occipital muscles)
Innervation of 1) lacrimal gland (tearing of the eye) 2) sublingual glands (salivation) 3) submandibular gland (salivation) 4) mucous
membranes of nose.

22. Patient developed imbalance, tinnitus and decrease hearing what drug he took:
A. ...mycin (this is most probably the correct answer as this suffix is probably denoting an aminoglycoside)
A. Isoniazid
B. Ethambutol
C. Pyramedizine
Ototoxic medications:
• Aminoglycosides. Aminoglycoside antibiotics (e.g., kanamycin, neomycin, amikacin, streptomycin, gentamicin) exhibit
[3,4]
cochleotoxicity but also affect the stria vascularis, causing vestibular problems. they produce damage through the
ability to generate free radicals in the inner ear.
• Loop diuretics. Loop diuretics (e.g., furosemide, ethacrynic acid, bumetanide) affect the potassium gradient of the stria
[2,3]
vascularis, as well as the electrical potential of the endocochlear structure. these medications produce tinnitus and

105
hearing loss. The hearing loss may be perceptible to patients or may be apparent only with audiometric testing. Their
[12]
toxicity is dose-related. thus, ototoxicity is more likely when the patient receives a rapid infusion of injectable loop
diuretics in renal failure, which allows the medications to accumulate. Furosemide-related ototoxicity is usually
reversible but may be permanent in rare instances (e.g., in patients with renal failure).
• Cisplatin affects the cochlea and stria vascularis through its ability to generate free radicals within the inner ear.
• Salicylates impact the cochlea. In high doses, they cause tinnitus and loss of hearing; both are usually seen only with
[7]
higher doses and regress upon discontinuation in most instances. the relationship between salicylate serum
concentrations and the level of hearing loss is linear.

23. Lump in throat barium enema and endoscopy normal?


A. Squamous cancer
B. GERD
C. Peptic ulcer
D. Pharyngeal Globus
Answer: D

Globus sensation is a functional esophageal disorder characterized by a sensation of a lump or foreign body in the throat. Globus
sensation has also been referred to as globus pharyngeus and the misnomer "globus hystericus." it is characterized by a sensation of
a lump, retained food bolus, or tightness in the throat that is not due to an underlying structural lesion, gastroesophageal reflux
disease, mucosal abnormality, or an esophageal motility disorder.
Patients have a sense of a lump, a retained food bolus, or tightness in the throat. Globus sensation is not painful and is typically
worse when swallowing saliva (dry swallow) and less noticeable when swallowing solids or liquids. In approximately 70 percent of
patients, globus symptoms are intermittent.

Http://www.uptodate.com/contents/globus-sensation?Source=outline_link&view=text&anchor=H480081804#H480081804

24. Man was has cholecystectomy now complaining of unilateral parotid swelling ,he has hx of mumps, facial nerve intact ,no
decrease in salivation ,lab was show cloudy saliva?
A. Sarcoidosis granuloma
B. Sialadenitis
C. Parotid cancer
D. Mumps
Answer: b
Patients typically present with erythema over the area, pain, tenderness upon palpation, and swelling. Frank cellulitis and induration
of adjacent soft tissues may be present. Purulent material may be observed being expressed from the wharton duct, particularly
upon milking the gland.
The most common organism is staphylococcus aureus. Other bacterial organisms include streptococcus viridans, haemophilus
influenzae, streptococcus pyogenes, and escherichia coli. The infection is often the result of dehydration with overgrowth of the oral
flora. The most common causes are postoperative dehydration, radiation therapy, and immunosuppression (eg, diabetes mellitus,
organ transplant, chemotherapy, human immunodeficiency virus). .medscape

25. Elderly patient with sudden onset hearing loss, loss of balance, and loss of eye blinking, where is the tumor?
A. Foramen ovale
B. Foramen secundum
C. Acoustic
Answer: C, acoustic neuroma or called schwannoma
Symptoms of acoustic neuroma: unilateral hearing loss, tinnitus, facial numbness and weakness due to compression on trigeminal
and facial nerve and imbalance. Source: bmj best practice.
http://www.nidcd.nih.gov/health/hearing/pages/acoustic_neuroma.aspx

106
26. Patient with an urti with hoarseness and painful swallowing, where is the site of infection
A. Larynx
B. Bronchiole
C. Trachea
Answer: larynx:

27. Most common or specific sings for otitis media:


A. Pain
B. Hearing loss
C. Discharge
Answer: a
Most common: pain. Most sensitive: immobility
Reference: master the board , uqu

28. Prevent symptoms of allergy to pollens and dust by using what:


A. Terbutaline
B. Montelukast
C. Ipratropium bromide
answer: b
Reference: toronto note

http://www.aafp.org/afp/2010/0615/p1440.html

107
29. Patient have sleep apnea & on pacemaker not responding to cpap (continuous positive airway pressure) what you do ?
A. Nose surgery
B. Chest surgey
C. Do cpap again
Answer: c?

30. 55 years with unilateral nasal chronic obstruction and epistaxis what you suspect
A. Polyp
B. Malignancy
C. Fibroangioma
Answer: b

31. A person fell and developed bruise and laceration over the nose. X-ray shows no fracture or displacement, what to do?
A. Ask for ct
B. Refer to ent
C. Nasal packing
Answer: c

32. 18 years old ear examination is normal and there is audiogram picture, what's the diagnosis?
A. Presbycusis
B. Otosclerosis
C. Otitis media
Answer : source of diagram: toronto notes.

33. A 35-year-old smoking man has white plaques in his mouth. What will you do?
A. Excisional biopsy
B. Close follow up
C. ?Lymph node
Answer: a , to roll out cancer

108
Leukoplakia — oral leukoplakia is a precancerous lesion that presents as white patches or plaques of the oral mucosa ( picture 2a-
c ). It represents hyperplasia of the squamous epithelium, which is believed to be an early step in the transformation of clonally
independent premalignant lesions from hyperplasia, to dysplasia, to carcinoma in situ, to invasive malignant lesions.
Risk factors for oral leukoplakia are similar to that of squamous cell carcinoma; it is particularly common in smokeless tobacco users
[ 7 ]. Any indurated areas should be biopsied to rule out carcinomatous changes.
https://yhdp.vn/uptodate/contents/mobipreview.htm?25/11/25786

34. Unilateral obstruction nose then foul smell, nasal discharge , erosin of vestible , normal in exam of nose
A. X ray head and chest
B. Nasal exam in ga
C. Antiboitic
Answer: this is a case of rhinosinusitis
http://www.uptodate.com/contents/acute-sinusitis-and-rhinosinusitis-in-adults-treatment

35. Patient presention with hearing loss , dizziness , vertigo what are you going to do to locate the lesion ?
A. Audiogram
B. MRI cerebellopontine
C. Ct of temporal area
Answer: a

36. Most common cause of tinnitus is:


A. Hypertension
B. Chronic use of salicylate
C. Sensorineural hearing loss (SNHL) – Got 100% in ENT
Answer: C
Toronto
Subjective tinnitus ( only heard by the patient ):
common otologic cause is: presbycusis. Common drug cause: asa
Objective tinnitus ( can be heard by others ):
benign intracranial htn
Reference: toronto note.
http://www.webmd.com/a-to-z-guides/understanding-tinnitus-basics

37. Nasal bleeding, first thing to do:


A. Balloon
B. Press on soft part of the nose
C. Press the carotid
Answer: b
http://cursoenarm.net/uptodate/contents/mobipreview.htm?12/45/13010

38. Patient has congestestion in tonsils and paratonsillar and c/o pain and cann't open mouth ?
A. Quinsy
B. Peritonsillar abscess
C. Tonsillitis
Answer: a or b (both are the same)
Explanation: the inability to open his mouth is trismus, which along with uvular deviation and dysphonia are the triad for
peritonsillar abscess (quinsy). The paratonsillar swelling also supports the diagnosis. There is most likely a mistake when writing the
choices since both a and b are the same.
Reference: toronto notes 2015, page ot42, otolaryngology

109
39. Young patient with conducting hearing loss and semicircular canal dehiscence
A. Glue ear
B. Otosclerosis
C. Tympanosclerosis

40. Nerve supply to tensor tympani and stapedius :


A. Trigeminal and facial
B. Facial and auditory
C. Trigeminal and ....

Answer :a
The stapedius is a muscle with a high oxidative capacity. It is innervated by the stapedial branch of the facial nerve (7th nerve). The
tensor tympani is innervated by a branch of the mandibular division of cn v, via the otic ganglion. While the nerve passes through
the ganglion, it does not synapse there.
Reference: http://www.dizziness-and-
balance.com/disorders/hearing/tinnitus/tensor%20tympani%20and%20stapedius%20myoclonus%20tinnitus.html

41. Patient came with green discharge nasal, congestion, 3 time took ab no benefit >> what is new tx?
A. Abx
B. Histamine
C. Steroid
Pharmacologic management: patients are often successfully treated with oral antihistamines, decongestants, or both; regular use of
an intranasal steroid spray may be more appropriate for patients with chronic symptoms.
Reference: http://emedicine.medscape.com/article/134825-overview

42. Allergic rhinitis symptoms caused by chemicals produced from?


A. Mast cells
B. Nk cells
C. Cd4
Answer: a
The mediators that are immediately released include histamine, tryptase, chymase, kinins, and heparin. The mast cells quickly
synthesize other mediators, including leukotrienes and prostaglandin d2.
Reference: http://emedicine.medscape.com/article/134825-overview#a5

43. Neck infection can spread to the mediastinum through:


A. Carotid artery
B. Para-pharyngeal space
C. Retropharyngeal space
Answer: c

44. Male patient presented with history of unilateral nasal obstruction and epistaxis. Ct scan done and shows soft tissue filling
the whole nose in that side. What is the cause of these symptoms?
A. Nasal polyps
B. Angiofibroma
C. Maligenant tumour
Answer: b
Reference: http://emedicine.medscape.com/article/872580-overview#a9
45. Patient with injury to accessory nerve which triangle injured?
A. Sub mental
110
B. Sub mandibular
C. Posterior
Answer: c

46. Swelling between ear and base of jaw with inflammation, what is the complication of it?
A. Facial n paralysis
B. Hearing loss
C. Encephalitis
Answer: c
CNS involvement is the most common extra salivary complication of mumps. Http://reference.medscape.com/article/966678-
overview#showall

47. Case of auricular hematoma (4 cm) came to ER. What's your initial management?
A. Give oral antibiotic and discharge with clinic follow up.
B. Incision and drainage and give oral antibiotic
C. Admit the patient and start iv antibiotic
Answer: b
-uptodate:
<2 cm = aspiration
> or 2 = i&d
Incision and drainage: a small (5 mm) incision is often all that is necessary. Gently separate the skin and perichondrium from the
hematoma and cartilage and completely express or suction out the hematoma. Be careful not to damage the perichondrium. If a
drain is placed, the patient should always be given antibiotics upon discharge. Http://emedicine.medscape.com/article/82793-
overview#showall

48. Old patient with epistaxis when he went from outside to home in winter “cold to warm place" what you going to give him:
A. Nasal steroid
B. Send home
C. Ephedrine nasal spray "not sure about this option"
Answer:
Hot and dry environments. The effects of such environments can be mitigated by using humidifiers, better thermostatic control,
saline spray, and antibiotic ointment on the kiesselbach area.
http://emedicine.medscape.com/article/863220-overview#a5

49. Case scenario dizziness vertigo progressive hearing loss what to see in ct?
A. Hemorrhage
B. Stroke
111
C. Normal CT
Answer: c
Ménière’s disease describes a set of episodic symptoms including vertigo (attacks of a spinning sensation), hearing loss, tinnitus (a
roaring, buzzing, or ringing sound in the ear), and a sensation of fullness in the affected ear.

50. pt with unilateral hearing loss , tinnitus, ..how to confirm the diagnosis :
A. tympanometry
B. tympanogram
C. MRI of cerbellopintine angel
Answer: C
51. adolescent with epistaxis for 10 min with no Hx of trauma, on examination: there was posterior oozing, Rx?
A. Conservative
B. posterior tampon
C. constrictive spray
Answer: B

52. Patient with recurrent episodes of vertigo, she feels of ear fullness , snhl and nausea , what is the diagnosis ?
A. Menere's diseases
B. Benign progressive vertigo
Answer: a
Diagnostic criteria for menere’s disease: 2 spontaneous episodes of rotational vertigo ≥20 min, audiometric confirmation of snhl and
tinnitus and/or aural fullness. (must have all three). Source: toronto notes.
http://www.nidcd.nih.gov/health/balance/pages/meniere.aspx

Meniere's disease is a disorder of the inner ear that causes episodes in which you feel as if you're spinning (vertigo), and you have
fluctuating hearing loss with a progressive, ultimately permanent loss of hearing, ringing in the ear (tinnitus), and sometimes a
feeling of fullness or pressure in your ear. In most cases, meniere's disease affects only one ear.
http://www.mayoclinic.org/diseases-conditions/menieres-disease/basics/definition/con-20028251

53. 40 years old male, complaining of sudden dizziness attack for 30-40 min, associated with nausea, vomiting, unilateral tinnitus
and hearing loss.
A. Acoustic neuroma
B. Neuritis
C. Meniere disease

Answer: c
Explanation: meniere’s disease: a cause of recurrent vertigo with auditory symptoms more common among females.
Hx/pe: presents with recurrent episodes of severe vertigo, hearing loss, tinnitus, or ear fullness, often lasting hours to days. Nausea
and vomiting are typical. Patients progressively lose low-frequency hearing over years and may become deaf on the affected side.
Reference: 3rd edition uqu > ent > q 104.

112
54. Which of the following medication causes vertigo?
A. Ethambutol
B. Streptomycin

Answer: b
Source: mercks manual
http://www.drugs.com/sfx/streptomycin-side-effects.html

55. Rhinorrhea, cough and conjunctivitis etiology ?


A. Rhinovirus
B. Adenovirus
Answer: adenovirus (conjunctivitis gave it away)

56. Patient came to ER with inability to breath from one nostrils and on examination the nostril is edematous and swollen what's
the initial management?
A. Local decongestant
B. Steroid
Answer: b

57. Year old man with chronic unilateral nasal obstruction, imaging shows flushy mas1
A. Polyp
B. Chronic forging body effect
Answer: polyp

58. Child complaining of painless unilateral hearing loss. On examination, tympanic membrane was opaque (erythematous).
A. Serous otitis media
B. Acute otitis media

Answer: a or b?
Explanation: answer: a there is no fever or pain, so most likely not infection. And this usually goes with otitis media with effusion.
Answer : b. Serous otitis media is a form of chronic otitis media ( according to toronto notes ) in which the tympanic membrane is
perforated while erythematous tympanic membrane is a diagnostic factor for aom.
Reference: http://emedicine.medscape.com/article/858990-overview
Aafm

History Physical findings Suggested cause of chl

Sudden painless Cerumen Complete canal occlusion

Sudden painful Narrow canal with debris Oe

Normal canal with red, immobile tm Com

Gradual painless Normal tm Otosclerosis

Reddish- blue pulsatile mass behind intact tm Glomus tumor

Perforated tm Cholesteatoma

113
59. Child with croup what is the best initial investigation to diagnosis5.
A. Chest x-ray
B. Pharyngeal swab
Answer:
Chest x ray ….(diagnosis of croup is made clinically. Cxr is needed in atypical presentation which will show “steeple sign “ from
subglotic narrowing ). Source: toronto notes.
60. Man working in factory, loud noises, his father and grandfather worked there. Hearing loss both of them. What to tell him?
A. Use cover protection for ear
B. Familial hearing loss

Answer: A
This is most likely a case of noise induced hearing loss.
http://emedicine.medscape.com/article/857813-overview

61. Indications of tonsillectomy? " recurrence not included in the choices "
A. Sleep apnea syndrome
B. Obesity

Answer: a
http://reference.medscape.com/article/872119-overview#a10

62. This a case of otitis media with effusion child with middle ear fluid and bulging in ear drum the rx?
A- Tympanostomy
B- Grommet insertion
Answer: • surgery: myringotomy ± ventilation tubes ± adenoidectomy (if enlarged or on insertion of second set of tubes after first
set falls out)
• ventilation tubes to equalize pressure and drain ear
Reference: toronto note

63. Case of serous otitis media with effusion , how to relieve symptoms (similar to the above q)?
A. Myringotomy

114
B. Grommet insertion
Answer: a
Reference: toronto note
In illustrated textbook of pediatric, it is mentioned that if conservative measures didn’t work (which is expectant management for
three months) insertion of ventilation tubes is indicated. If the ventilation tubes failed then insertion of ventilation tubes with
adjuvant adenoidectomy is indicated.
In mercks manual, myringotomy with tympanostomy tube insertion is indicated if no improvement occurs within 1-3 months.
Typmanostomy tubes are suggested at 6-12 months of continued bilateral ome or 4 months with bilateral hearing loss.

64. Case of otitis media what is complication:


A. Encephalitis
B. Hearing loss
answer: b
Extracranial: hearing loss, speech delay, ™ perforation, extension of suppurative process to adjacent structures ( mastoiditis,
petrositis, labyrinthitis ), cholesteatoma, facial nerve palsy, ossicular necrosis, vestibular dysfunction.
Intracranial: meningitis, epidural and brain abscess, subdural empyema, lateral and cavernous sinus thrombosis, carotid artery
thrombosis
Others: sigmoid sinus thrombophlebitis.
Source: toronto notes.

65. Runny nose and unilateral nasal obstruction for more than one year, on exam the inferior turbinates were swollen. What’s
the most likely diagnosis ?
A. Allergic rhinitis
B. Sinusitis
Answer : a?
Turbinates swollen : allergic rhinitis

66. 40 years old man presents with decreased hearing but his own sound feels louder than before to him. What is diagnosis?
A. Presbycusis
B. Otitis media
Answer: b, but if there is osteosclerosis it will be more appropriate.
Explanation: the presentation is most likely conductive hearing loss, which is can be caused by otitis media but not presbcusis.
67. Scenario about adult male complaining of severe ear pain with discharge inflamed tympanic membrane ( missed some
points) what is the diagnosis ?
A. Otitis externa
B. Cholesteatoma
Answer: b
The majority (98%) of patients with cholesteatoma have ear discharge or hearing loss or both in the affected ear.
Both can present with these symptoms, however, more accurate diagnosis can be made based on the smell of discharge and more
information on the appearance of the auditory canal and the tympanic membrane.
If the discharge is smelly and attic crust is visualized in retraction pockets or white mass behind intact tympanic membrane this is
likely to be cholesteatoma.
https://en.wikipedia.org/wiki/cholesteatoma
O.e is not associated with tympanic membrane infection

115
68. A patient has fronciosis in the left side of the nose, then he developed orbital edema. Which vein will carry this effect ?
A. Maxillary
B. Ophthalmic
Answer: ophthalmic vein is the most accurate answer, if it not one of choices then “ethmoidal “, if both are not present choose:
ophthalmic a.
Ophthalmic vein. Source: lippincott's illustrated q&a review of anatomy and embryology.

69. Child c/o unilatral nasal foul smelling discharge for two weeks ,what is your treatment?
A. X-ray of the head
B. Antibiotics
Answer: choices incomplete and answer probably missing. If not, answer is a
Explanation: based on symptoms child has foreign body, and between the two choices a is more suitable, even though imaging is not
usually done according to medscape and only occasionally done according to lecture notes.
References: http://emedicine.medscape.com/article/763767-overview#a5
- Lecture notes: diseases of the ear, nose and throat, 11 edition – page 191

70. scenario of patient with mastoiditis what is the proper antibiotic:


A. Amoxicillin
B. Azythro
Answer:?
Vancomycine and ceftricaxione are recommended until the culture become available.
Reference: http://emedicine.medscape.com/article/2056657-treatment

71. Case about deafness and paralysis of tympanic muscle and other muscle i can not recall it
A. Trigeminal
B. Facial
The q not clear mean the tensor tympani is a muscle within the ear,innervation of the tensor tympani is from the tensor tympani
nerve, a branch of the mandibular division of the trigeminal nerve
https://en.wikipedia.org/wiki/tensor_tympani_muscle
If the q mean tympanic membrane so, the nerve is facial n.

72. Patient after tooth extraction he feel numbness in his left ant tongue, what nerve is sensory supply to this area :
A. lingual ( right answer)
B. Infralvoular
the lingual nerve is a branch of the mandibular division of the trigeminal nerve (cn v3), which supplies sensory innervation to
the tongue. It also carries fibers from the facial nerve, which return taste information from the anterior two thirds of the tongue, via
the chorda tympani.

73. Child with signs and symptoms of acute otitis media what treatment will you give?
A. All answers were antibiotics
B. Amoxicillin
Answer: b.
The recommendations support the use of amoxicillin as the first-line antimicrobial agent of choice in patients with aom.
Toronto notes 2015

74. Patient with history of pinna pain with discharge from the ear, best mx?
A. Oral feuroxim
B. Topical neomycin
Answer: b
Sources: http://emedicine.medscape.com/article/994550-overview

116
75. Child has an URTI then has ear pain, Rinne test is negative but Weber test show the sound is louder in the affected area
A. otitis media
B. Mastoiditis
Answer: a

76. Patient with submandibular mass that increase in size with eating. Diagnosis?
A. Acute sialadenitis
B. Canal calculi
Answer: b
Acute sialadenitis is an acute inflammation of a salivary gland. Patients typically present with erythema over the area, pain,
tenderness upon palpation, and swelling. Frank cellulitis and induration of adjacent soft tissues may be present. Purulent discharge
upon milking gland. Http://emedicine.medscape.com/article/882358-clinical#showall canal calculi (sialolithiasis): more than 80%
occur in the submandibular gland or its duct, 6% in the parotid gland and 2% in the sublingual gland or minor salivary glands.
Sialolithiasis typically causes pain and swelling of the involved salivary gland by obstructing the food related surge of salivary
secretion. Http://www.nature.com/bdj/journal/v193/n2/full/4801491a.html

salivary gland problems that cause clinical symptoms include:


Obstruction: obstruction to the flow of saliva most commonly occurs in the parotid and submandibular glands, usually because
stones have formed. Symptoms typically occur when eating. Saliva production starts to flow, but cannot exit the ductal system,
leading to swelling of the involved gland and significant pain, sometimes with an infection. Unless stones totally obstruct saliva flow,
the major glands will swell during eating and then gradually subside after eating, only to enlarge again at the next meal. Infection
can develop in the pool of blocked saliva, leading to more severe pain and swelling in the glands. If untreated for a long time, the
glands may become abscessed.
Refrence: http://www.entnet.org/content/salivary-glands

77. What is the most common site of malignancy in paranasal sinus?


A. Maxillary
117
Answer: a
Reference: 3rd edition uqu > ent > q 92 + medscape.
Reference: http://emedicine.medscape.com/article/847189-overview#showall

Comment about the question: the presentation is most likely related to otitis media with effusion, however, the name serous otitis
media is given to a condition where there is serous drainage from the ear.

78. Post tonsillectomy patient has loss of taste of the posterior 1/3 of tongue. What is the nerve injured?
A. Glossopharyngeal nerve injury
Answer: a
Reference: toronto notes 2015

79. In presbycusis (snh loss). Which structure in the inner ear gets affected?
A. Hair cells.
Answer: a
Presbycusis casuses hair cell degeneration.
Reference: toronto notes 2015

80. Old guy can’t tolerate loud noises which nerve is affected?
Answer: facial nerve (due to loss of protective muscle (stapedius))
Reference: toronto notes 2015

81. A patient with chronic tonsillitis complaining of fever, chills, left throat pain and uvula deviation. What is the diagnosis?
Answer: peritonsillar abscess
Reference: toronto notes 2015

82. "otitis media case + child with acute ear pain , redness and bulging of tympanic membrane, whats the diagnosis ?
A. Acute otitis media
Answer: a
Explanation: clinical features:
• triad of otalgia, fever (especially in younger children), and conductive hearing loss
• rarely tinnitus, vertigo, and/or facial nerve paralysis
• otorrhea if tympanic membrane perforated
• infants/toddlers ƒ ear-tugging (this alone is not a good indicator of pathology) ƒ hearing loss, balance disturbances (rare) ƒ
irritable, poor sleeping ƒ vomiting and diarrhea ƒ anorexia
• otoscopy of tm ƒ hyperemia ƒ bulging, pus may be seen behind tm ƒ loss of landmarks: handle and long process of malleus not
visible
Reference: toronto note 2015

83. The most common cause of otitis media in all age group ?
Answer: streptococcus pneumonia, haemophilus influenzae, moraxella catarrhalis.
Reference: aafm

84. Best treatment for allergic rhinitis


Answer: ?
The management of allergic rhinitis consists of 3 major categories of treatment, (1) environmental control measures and allergen
avoidance, (2) pharmacological management, and (3) immunotherapy.

118
Glucocorticoid nasal sprays are presently the most effective single-agent maintenance therapy for allergic rhinitis
Answer of one who got 100 % in ent
Reference: http://www.uptodate.com/contents/pharmacotherapy-of-allergic-rhinitis#h25107621

85. Nasal obstruction, rhinorrhea, pale swelling, sneezing and tearing eyes, diagnosis ?
Answer: allergic rhinitis
Explanation:
Clinical features
• nasal: obstruction with pruritus, sneezing
• clear rhinorrhea (containing increased eosinophils)
• itching of eyes with tearing
• frontal headache and pressure
• mucosa: swollen, pale, “boggy”
Reference: Toronto notes 2015

86. Case of boy recently bought a cat developed allergy ?


Answer: allergic rhinitis?

87. What causes mixed hearing loss?


Answer: causes of mixed hearing loss vary wildly. Typically, the sensorineural hearing loss is already present and the conductive
hearing loss develops later and for an unrelated reason. Very rarely, a conductive hearing loss can cause a sensorineural hearing
loss. Some diseases that might present with mixed type hearing loss: chronic otitis media, otosclerosis, ear trauma, barotrauma and
head trauma.
http://www.coastalhearingcenters.com/mixed-hearing-loss/

88. Patient complaining of euphonia & doing laryngoscope it was normal & ask him to coughing he is cough what diagnosis?
A. functional euphonia
Answer: a
89. Intracranial growth compressing the maxillary artery. Which one of the following is affected?
Answer: ?
Explanation: the maxillary artery can be defined as one of the continuations of the external carotid artery, and distributes the blood
flow to the upper (maxilla) and lower (mandible) jaw bones, deep facial areas, cerebral dura mater and the nasal cavity. Hence it is
considered a blood vessel which supports both hard and soft tissues in the maxillofacial region.
Nb know the anatomy and branches.

90. 60-year-old male with unilateral parotid swelling tender but no facial nerve dysfunction?
Answer: ?
Explanation: if there is fever along with the tenderness the answer would be bacterial parotitis , if no systematic sx usually there will
be no tenderness where the answer would be benign pleomorphic adenoma , facial nerve involvement would rise the suspicion
toward malignancy .
Reference: medscape

91. Sore throat, palpable neck mass, hoarseness voice, hearing loss ask about tx?
A. Surgery and radiotherapy
(i thought about nasopharyngeal ca) from toronto note:

119
92. Most common cause of deafness in children is
Recurrent otitis media is the most common conductive cause of hearing loss
Http://www.asha.org/public/hearing/causes-of-hearing-loss-in-children/

93. Bleeding in posterior inferior area of nasal septum. Which vessel affected?
Answer: sphenopalatine artery
Supply to that area: external carotid > internal maxillary > sphenopalatine artery > nasopalatine.
Source: toronto notes.

94. Otitis media pt. Not taking antibiotic probably then after week came back by tenderness and swelling behind the ear,
diagnosis?
A. Acute mastoiditis
Answer: a

95. Thick white plaques in throat associated with gingivitis , diagnosis ?


Answer: could be fungal infection ? ????

96. Neck mass; pathology report: ki67 +ve what else you expect
A. Bcl2 over expression
Answer: Burkitt lymphoma
97. Boy with unilateral nasal obstruction, foul smelling diagnosis ?
A. Foreign body
Answer: a
98. Red erythematous epiglottis...causative organism
Answer: case of epiglottitis
H.influenzae tybe b

99. Hemorrhagic vesicles on tympanic membrane


Answer: bullous myringitis
http://www.msdmanuals.com/professional/ear,-nose,-and-throat-disorders/middle-ear-and-tympanic-membrane-
disorders/myringitis

120
100.Baby come with ear pain and discharge, in examination erythema and edema in the ear canal what is the diagnosis?
A. Otitis externa
Answer: a
Source: http://emedicine.medscape.com/article/994550-overview

101.Management of cholesteatoma?
http://emedicine.medscape.com/article/860080-overview

102.Chronic om with green discharge oxidase +ve organism ? (ent)


A. Pseudomonas
Answer is a. Pseudomonas which is an oxidase positive gram negative rod. (first aid step 1)
Questions about menires disease:
Ménière disease is a disorder of the inner ear that is also known as idiopathic endolymphatic hydrops. Endolymphatic hydrops refers
to a condition of increased hydraulic pressure within the inner ear endolymphatic system. Excess pressure accumulation in the
endolymph can cause a tetrad of symptoms: (1) fluctuating hearing loss, (2) occasional episodic vertigo (usually a spinning sensation,
sometimes violent), (3) tinnitus or ringing in the ears (usually low-tone roaring), and (4) aural fullness (eg, pressure, discomfort,
fullness sensation in the ears).
http://emedicine.medscape.com/article/1159069-overview

103..in case of external laryngeal nerve injury the ability to produce pitched sounds is then impaired along with easy voice
fatiguability

104.Loss of sensation on jaw line.


 If asking about affected nerve, then it would be “mandibular branch” of the 5 cranial nerve, trigeminal nerve.
th

105.Long case ask u for languages assessment for 3 y child?

106.Old age with recurrent parotid enlarge with tender and erythema?
http://emedicine.medscape.com/article/882461-overview

107.The nerve that supply the largest part of tongue is ?


Answer: hypoglossal nerve

121
Supply the anterior 2/3 of the tongue
Sensory = anterior 2/3: lingual from trigeminal while taste by chora tympani from facial
Posterior 1/3: glossopharyngeal nerve (ix)
Motor innervation: - cn xii (hypoglossal) except palatoglossus muscle cn x (vagus)
http://medicalopedia.org/2695/nerve-supply-of-tongue/

108.Loss of sensation over maxilla and mandible, which nerve ?


Answer: trigeminal
The trigeminal nerve is the largest and most complex of the 12 cranial nerves (cns). It supplies sensations to the face, mucous
membranes, and other structures of the head. It is the motor nerve for the muscles of mastication and contains proprioceptive
fibers.
http://emedicine.medscape.com/article/1873373-overview

109.Overuse of vasoconistrictor nasal drops ?


Answer: rebound phenomena
Rhinitis medicamentosa (rm), also known as rebound rhinitis or chemical rhinitis, is a condition characterized by nasal congestion
without rhinorrhea or sneezing that is triggered by the use of topical vasoconstrictive medications for more than 4-6 days
http://emedicine.medscape.com/article/995056-overview

110.Patient with neck pain and headache in occipital area for months ,, there is a lomitation in movrmement ? What is

111.3rd branch of maxillary artery :


Spenopalatine ??
Answer: sphenopalatine artery is from third portion.

The maxillary artery, the larger of the two terminal branches of the external carotid artery, arises behind the neck of the mandible,
and is at first imbedded in the substance of the parotid gland; it passes forward between the ramus of the mandible and the
sphenomandibular ligament, and then runs, either superficial or deep to the lateral pterygoid muscle, to the pterygopalatine fossa.
It supplies the deep structures of the face, and may be divided into mandibular, pterygoid, and pterygopalatine portions.

122
112.Nerve responsible for the tonsils?
Answer: glossopharyngeal nerve (cn ix) & lesser palatine branch of sphenopalatine ganglion (branch of cn ix) explanation: nerve
supply to the tonsil is from the glossopharyngeal nerve.
nerve supply of tonsils are innervated via tonsillar branches of the maxillary nerve and the glossopharyngeal nerve
Link: http://www.drtbalu.co.in/tonsil.html
http://emedicine.medscape.com/article/1899367-overview#a2

113.Nerve responsible for the sinuses?


Answer: maxillary artery & its branches
Explanation:the maxillary sinus is supplied by branches of the internal maxillary artery, which include the infraorbital, alveolar,
greater palatine, and sphenopalatine arteries. It is innervated by branches of the second division of the trigeminal nerve, the
infraorbital nerve, and the greater palatine nerves
Link: http://emedicine.medscape.com/article/1899145-overview#a2

114.Case of serous otitis media with effusion , how to relieve symptoms ?


st
Confirmed aom :1 : oral or rectal analgesia plus antibiotic therapy. If unresponsive to antibiotics or with severe pain :
tympanocentesis .bmj http://bestpractice.bmj.com/best-practice/evidence/intervention/0301/0/sr-0301-i8.html

115.50 years old patient complaining of unilateral nasal obstruction associated with epistaxis, ct done showing ............, what's
the diagnosis:
A. Malignancy (my answer, i got 100% in ENT)

116.A child was diagnosed with otitis media. The isolated organism was streptococcus pyogenes. He is currently on amoxicillin,
what would you add if there is no enough response?
Answer: clavulanic acid
st
Amoxicillin is the first line therapy for om. If the 1 line therapy fails, which is defined as at least 3 days of treatment with high dose
amoxicillin, a second line treatment is to be administered. The recommendations suggested administering the following three
preparations: high-dose oral amoxicillin-clavulanate, oral cefuroxime and intramuscular ceftriaxone

117.32 years old female, presenting with decrease in hearing, her mother lost hearing in her 30s. Diagnosis ?
A. Acoustic neuroma

An acoustic neuroma, also called a vestibular schwannoma, is a benign slow-growing tumor affecting the vestibulo-cochlear nerve.
95% of acoustic neuroma (an) are unilateral. Initial symptoms include unilateral hearing loss, balance disturbances or vertigo,
tinnitus, and a feeling of fullness in the ear. Most unilateral vestibular schwannomas are not hereditary and occur sporadically.
Bilateral vestibular schwannomas affect both hearing nerves and are hereditary usually associated with a genetic disorder called
neurofibromatosis type 2 (nf2).

118.Patient came with hearing loss, her mother has history of the same, what is the diagnosis ?
Most likely inheritance pattern is autosomal dominant as only the mother is affected
Approximately 50% of all cases of congenital deafness are genetic. Approximately 70% of cases of hereditary deafness are
nonsyndromic, and the remaining 30% are syndromic, associated with specific deformities or medical problems. Of nonsyndromic
hearing losses, 75-85% are inherited in an autosomal recessive pattern, 15-20% are inherited in an autosomal dominant pattern, and
1-3% are inherited in an x-linked pattern.

Waardenburg syndrome

123
waardenburg syndrome is the most common cause of autosomal dominant syndromic hearing loss. The syndrome includes dystopia
canthorum, a broad nasal root, confluence of the medial eyebrows, heterochromia irides, a white forelock, and bilateral or unilateral
sensorineural hearing loss

Branchio-oto-renal syndrome
Branchio-oto-renal syndrome is the second most common cause of autosomal dominant syndromic hl. This condition manifests as
renal abnormalities, preauricular pits, deformed auricles, and lateral branchial cysts. The hearing loss may be conductive, snhl, or
mixed.
Neurofibromatosis type 2
Neurofibromatosis type 2 (nf2) is associated with vestibular schwannomas, meningiomas, ependymomas, juvenile cataracts, and
other intracranial and spinal tumors. The gene for nf2 has been mapped to chromosome 22q12.2 and is thought to be a tumor-
suppressor gene. It has about 50% penetrance. In the wishart type of nf2, the disease manifests in childhood or early adulthood.

119.Where we can palpate the facial nerve


Answer:
Link: http://teachmeanatomy.info/head/cranial-nerves/facial-nerve/
http://emedicine.medscape.com/article/835286-overview
120.Pt with epistaxis (they describe the site and asked about the artery affected) Vague description.

Kiesselbach's plexus, in Little's area, is a region in the anteroinferior part of the nasal septum where four arteries
anastomose to form a vascular plexus of that name. The arteries are
• Anterior ethmoidal artery (from the ophthalmic artery)
• Sphenopalatine artery (terminal branch of the maxillary artery)
• Greater palatine artery (from the maxillary artery)
• Septal branch of the superior labial artery (from the facial artery)
• Posterior ethmoidal artery
Reference: Wikipedia: https://en.wikipedia.org/wiki/Kiesselbach's_plexus

121.Submental pain, tender, palpable---- ? Submental calculi (sialolithiasis)

122.What is the most common malignant sinus tumor?


Answer: Maxillary
Approximately 60-70% of sinonasal malignancies (SNM) occur in the maxillary sinus and 20-30% occur in the nasal cavity itself. An
estimated 10-15% occur in the ethmoid air cells (sinuses), with the remaining minority of neoplasms found in the frontal and
sphenoid sinuses http://emedicine.medscape.com/article/847189-overview#a6

123.medication for resistant hiccups?


Answer:
Chlorpromazine is the most thoroughly studied and appears to be the drug of choice.
Reference: medscape http://emedicine.medscape.com/article/775746-medication
Similar question in another exam with different choices:

124.gradual hearing loss in a 17 yo male with normal tympanic membrne.


Answer if unilateral: acoustic neuroma, if with noise exposure and bilateral: noise induced
Ref: http://www.aafp.org/afp/2003/0915/p1125.html

124
Ophthalmology

1. Acute closed angle glaucoma what drug is contradicted?


A. Timolol
125
B. Pilocarpine
C. Acetazolamide
D. Cimetidine
Answer: D
Antihistamines. H2 receptor agonists (cimetidine), adrenergic agonists. Epinephrine, ipratropium bromide,
etc https://www.reviewofoptometry.com/ce/meds-that-dont-mix-with-glaucoma-patients
Timolol: a synthetic compound that acts as a beta blocker and is used to treat hypertension, migraines, and glaucoma.)
[2]
Pilocarpine: It is a non-selective muscarinic receptor agonist in the parasympathetic nervous system, which acts
[3]
therapeutically at the muscarinic acetylcholine receptor M3 due to its topical application, e.g., in glaucoma and xerostomia.
The answer is probably going to be an anticholinergic agent such as:
Anticholinergics include the following:
. Atropine, scopolamine
. Glycopyrrolate
. Benztropine, trihexyphenidyl
Precipitating factors include drugs (ie, sympathomimetics, anticholinergics, antidepressants [SSRIs], anticonvulsants, sulfonamides,
cocaine, botulinum toxin), dim light, and rapid correction of hyperglycemia.

2. 24 years old female newly diagnosed type 2 DM, she is wearing glasses for 10 years, how frequent she should follow with
ophthalmologist?
A - 6 months
B - 12 months
C - 5 years
D - 10 years
Answer: B
Evidence:
- For type 1 diabetic: retinal screening annually beginning 5 years after onset of diabetes, general not before onset of puberty.
- For type 2 diabetic: screening at the time of diagnosis then annually.
rd
Reference: 3 Edition UQU > Ophthalmology
3. Cavernous sinus aneurysm leading to diplopia & blurred vision, what nerve affected?
A - Trochlear
B - Abducens
C - Optic
D - Ophthalmic (V1)

Answer: B
Evidence: Cavernous sinus syndrome presents with variable ophthalmoplegia, decrease corneal sensation, Horner syndrome and
occasional decreased maxillary sensation. 2° to pituitary tumour mass effect, carotid-cavernous fistula, or cavernous sinus
thrombosis related to infection. CN VI is most susceptible to injury. Reference: FA USMLE step 1

4. What is the function of superior rectus muscle? (answer not known)


A - in down
B - out down
C - up in
D - up out
Answer: ? | Reference: Gray’s anatomy
o The superior rectus is an extraocular muscle that attaches to the top of the eye. It moves the eye upward.
American Association for Pediatric Ophthalmology and Strabismus
(Contraction of the superior rectus results in elevation, intorsion, and adduction of the eye)
Reference: http://www.yale.edu/cnerves/cn3/cn3_3.html
5. patient complaint of red eye and decreased vision in left eye since 3 days, in examination there are ciliary flush, not fully
reactive pupil and floaters in anterior chamber. What is the diagnosis:
126
A - Uveitis
B - Glaucoma
C - Retinitis
D - Keratitis

Answer: A | Reference: http://www.merckmanuals.com/professional/eye-disorders/uveitis-and-related-disorders/overview-of-


uveitis
6. Which of the following cause loss of colour vision?
A - acute angle closure glaucoma
B - mature cataract.
C - severe iridocyclitis.
D - optic atrophy.

Answer: D (Loss of colour of vision out of proportion to the loss of visual acuity is specific to optic nerve pathology)
Reference: http://www.uptodate.com/contents/optic-neuritis-pathophysiology-clinical-features-and-diagnosis

7. Patient had laser for refractive correction. One of post-op complications of this procedure is dryness of the eye. How to
prevent this dryness?
A - blockage of lacrimal duct
B - blockage of lacrimal canal
C - blockage of punctum
D - blockage of lacrimal sac

Answer: C | Reference: http://eyewiki.aao.org/Dry_Eyes_After_Laser_in_situ_Keratomileusis_(LASIK)#Therapeutic_Options

8. Patient wakes up with mucopurulent discharge and red eye. (other version: mucopurulent discharge, sticky eyes upon waking
up) What is dx?
A - viral.
B - bacterial (other version: bacterial conjunctivitis)
C - allergy
D - hypersensitive

Explanation: From the presentation, most likely Bacterial conjunctivitis.


Discharge: More purulent than in viral conjunctivitis, with more mattering (generally white or yellow mucous discharge [green in
some cases]) of the eyelid margins and greater associated difficulty prying the eyelids open following sleep; patients may report
waking up with their eyes “glued” shut.
Reference: http://emedicine.medscape.com/article/1191730-overview

Answer: B | Reference: http://www.medscape.com/viewarticle/522242


9. Which layer removes excessive water from cornea:
A - stroma
B - tear film
C - endothelium
D - epithelium

Answer: C | Reference: http://www.ncbi.nlm.nih.gov/pubmed/7122038

10. 20 y.o had trauma to left eye, he has corneal ulcer, photophobia, and pain, how do you manage?
A - debridement with Burr and systemic antibiotics.
127
B - cotton swab debridement and local steroid.
C - irrigation, systemic antibiotics and cycloplegia.
D - local antibiotics, cycloplegia and referral.

Answer: D | Reference: Toronto notes (Culture prior to treatment, & topical antibiotics every hour)
Evidence: Bacterial ulcer is most common cause in contact lens wearer. Acanthamoeba ulcer is due to swimming while wearing
contact lens. Fungal ulcer is due to trauma with vegetable material. Treatment for corneal ulcers, regardless of cause, begins with
Antibiotics. Source; merck’s manual.
References: Toronto note; https://www.merckmanuals.com/professional/eye-disorders/corneal-disorders/corneal-ulcer

11. patient with bilateral eye myopia each 2 month change his glasses (other version: Long scenario about patient with eye
problems, he has myopia, bilateral decreased vision, glasses were advised, after month he became not satisfied, there is
refractive error and astigmatism:):
A - Keratoconus
B - Keratomalacia
C - Keratoglobus
D - Keratectasia

Answer: A

12. A young male complaining of redness and discomfort in his eyes during the exam time, his visual acuity is 6/6 in both eyes
unaided. Both the anterior and posterior chambers are normal. What is the most likely diagnosis: (answer not sure of)
A - Myopia
B - Anisometropia
C - Astigmatism
D - Hyperopia

Answer: D?

13. patient wakes up from sleep with watery discharge from RT eye with redness. everything else is normal. What is the Tx?
A - topical steroid
B - topical Antihistamine
C - topical antibiotics
D - reassurance

Answer: D? (answer not sure of) [reassurance? (viral) or (eye dryness)]

14. Picture of eyelid swelled there is pain and increased tearing. what is the most likely sign? (answer not known)
A - decrease vision
B - discharge
C - uveitis
D - scleritis

Answer: ? eyelid swelling is not a specific sign for any of the choice
Reference: http://www.webmd.boots.com/eye-health/guide/swollen-eyes

15. Pic of eye ... mother got tear in her eye by her daughter fingernail. . Treatment is?
A. Antibiotics drops
B. Antibiotics ointment

128
C. Steroids drops
D. Steroids ointment
Answer: Antibiotic ointment.
Explanation: Corneal Abrasion antibiotic ointment. Ophthalmic corticosteroids tend to promote the growth of fungi and reactivation
of herpes simplex virus and are contraindicated.
Reference: http://www.merckmanuals.com/professional/injuries-poisoning/eye-trauma/corneal-abrasions-and-foreign-bodies

16. Treatment of orbital pseudo tumor?


A. Radiation
B. Antibiotics
C. Surgical excision
D. Systemic steroid
Answer: D (mainstay therapy of orbital pseudotumor is Systemic Steroid)
Explanation: Orbital inflammatory disease (pseudotumor) usually is treated medically with systemic steroids.
Reference: http://emedicine.medscape.com/article/1218892-treatment

17. Pt. Came complain of foreign body sensation in his eye after splash something on him.. after removing what should you give
him?
A) Antibiotics oral
B) Antibiotics topical
C) Steroids oral
D) Steroids topical
Answer: B
Reference: http://emedicine.medscape.com/article/82717-overview#a5

18. Patient presenting with red eye and ciliary flush, acute painful vision loss in his right eye, and headache.His right eye had
cloudy oedematous cornea, his left eye showed superior temporal rim cupping of the disc. Tonometer showed high IOP in the
right eye, and normal IOP in left eye. Which of the following is most likely explanation for his presentation?
a) bilateral glaucoma.
b) uveitis in right eye and congenital anomaly in left eye
c) macular degeneration in right eye.
d) retinal detachment in right eye.
Answer: A

19. Best distance between "Snell Chart" & the patient?


A) 3 Meters
B) 4 Meters
C) 6 Meters
D) 9 Meters
Answer: C
Reference: Toronto Notes 2015

20. you did examination on elderly Hypertensive man you find increase in cupping he is not complaining of anything what is the
cause of cupping?
A) Acute angle glaucoma
B) HTN
C) DM
D) Retinal detach.
Answer: incomplete
(Acute angle glaucoma  painful; retinal detach  curtain like vision loss or floaters)

129
21. Painful mass in medial side of eye to the nose, what is the management?
A. Oral antibiotic
B. Topical antibiotic
C. Topical corticosteroid
D. Surgical drainage

Answer: B
If it's Hordeolum (Stye) which is Painful infectious inflammation, the treatment is topical antibiotics (ex. Erythromycin), but if it's
chalazion which is Painless granulomatous inflammation, the treatment is incision and curettage or intrachalazion steroid

22. A picture of an eye with fluorescent, what is the Dx?


A. Bacterial keratitis
B. Fungal keratitis
C. Herpetic keratitis
D. Amoebic keratitis
Answer: C
Lecture notes Ophthalmology: The pathognomonic appearance is of a dendritic ulcer.

Figure 7.6 A dendritic ulcer seen in herpes simplex infection.

23. Patient complains of dry eyes, Doctor prescribed eye drops:


A. 2 drops in lower fornix
B. 1 in lower fornix
C. 2 in upper fornix
D. 1 in upper fornix
Answer: B
Instructions for patients using eye drops or ointments
Eye drops:
Wash hands before and after using drops.
Shake the bottle.
Instil in the lower conjunctival fornix and ideally keep the eye closed for 1-2 minutes after application.
Only one drop is needed per dose.
Minimise systemic absorption and adverse effects by closing eyes after administering eye drops, gently but firmly pressing the tear
duct against the nose for at least one minute, and then removing excess solution with absorbent tissue.
http://patient.info/doctor/eye-drugs-prescribing-and-administering
Only one drop of each drug is required
https://www.nursingtimes.net/download?ac=1289059
24. Hemiplegic patient comes with abnormality in the eye; may be nystagmus, where is the lesion in the brain?
A. Pons
130
B. Medullar oblongata
C. Internal capsule
D. Midbrain
Answer: B

25. Man got new glasses, after one week he cannot see well by the new glass , after examination change the glass and referred
to ophthalmologist.. What is the cause of change the old glass?
A. Glucose in lens
B. Cataract
C. Glaucoma
D. keratitis
Answer : A
From ophthalmic consultant

26. If you are examining the right eye by using with light reflex and there was no changing of pupil of the right eye but there was
changing in pupil of the left eye(consensual) , so where is the lesion ?
A. Rt optic nerve
B. Lt optic nerce
C. Rt occulomotor nerve
D. Lt occulomotor nerve
Answer: C
Afferent  optic nerve  when light shone on 1 eye both eyes are stimulated
Efferent  oculomotor nerve  constrict the ipsilateral pupil

http://www.fpnotebook.com/eye/exam/PplryLghtRflx.htm

27. pic of eye with fluorescence (it shows dendritic herpes ulcer) what to give :
A- antiviral
B- antifungal
C- topical antiviral
D- topical antifungal

Answer: C
Explanation: treatment of herpitic keratitis: topical antiviral such as trifluridine, consider systemic antiviral such as acyclovir.
Dendritic debridement. And NO STEROIDS initially – may exacerbate condition.
Reference: Toronto Notes 2015, page OP19, ophthalmology

28. patient with photophobia and tearing got injured by his brother finger nail what is the treatment:
A. steriod ointment
B. steriod drop
C. Antiviral
D. fitting contact lens

Answer: C
Corneal abrasion is probably the most common eye injury and perhaps one of the most neglected. It occurs because of a disruption
in the integrity of the corneal epithelium or because the corneal surface scraped away or denuded as a result of physical external
forces.
http://emedicine.medscape.com/article/1195402-medication#5
29. Pt. complaining of floaters, flashes and hx of DM and HTN what is the cause
A. DM
B. HTN
131
C. Retinal detachment
D. Cataract
Answer: c. retinal detachment. Symptoms supported by Hx of DM. Mayoclinic http://www.mayoclinic.org/diseases-
conditions/retinal-detachment/symptoms-causes/dxc-20197292

30. Type of Glaucoma you will do Trabeculectomy for?


A. ACAG,
B. COAG,
C. CCAG,
D. Congenital
Answer: B
The indications for trabeculectomy are as follows: Primary open-angle glaucoma, Primary angle-closure glaucoma not
responsive to iridotomy and/or iridoplasty, Secondary open-angle glaucoma, Secondary angle-closure glaucoma, Childhood
glaucomas. Medscape http://emedicine.medscape.com/article/1844332-overview

31. Boy scratched his mother's eye, inflammation developed, what to do;
A. Steroid ointment
B. Steroid drops
C. Antiviral
D. Contact eye
Corneal Abrasion: Topical ABx, Topical NSAIDS, Most cases resolve with no treatment.

32. Foreign body sensation after exposure to (something). He treated now what to give
A. oral antibiotics
B. oral steroid
C. local antibiotics
Answer:

33. Patient got a swelling under his eyebrow to which lymph node does it drain?
A. parotid
B. submandibular
C. sub mental
D. Cervical
Answer: A
Reference: http://teachmeanatomy.info/neck/vessels/lymphatics/

34. Patient with progressive decrease of vision, His vision acuity is about 8/20, (and he had something -8 but I did not remember
what is that) and virtuous atrophy. What is the type of myopia in this patient?
A. Physiological
B. Pathological
C. Curvature
D. Index
Could not find an answer

35. When examine Trochlear nerve we will said to patient to move his eye
A- Medial upward
B- Medial downward
C- Lateral upward
D- Lateral downward

Answer: D
132
The trochlear nerve is tested by examining the action of its muscle; the superior oblique. When acting on its own this
muscle depresses and abducts the eyeball.

Reference: Wikipedia: https://en.wikipedia.org/wiki/Trochlear_nerve#Examination

36. A known case COPD and DM, diagnosed to have primary open angle glaucoma. What is the optimal treatment for glaucoma?
A - Topical Timolol
B - Topical Carteolol
C - Systemic Acetazolamide
Answer: C

37. Pt came to ER with acut eye pain and headache , nausea and vomiting case of acute glaucoma?
A. IV acetazolamide + pilocarpi drop
B. Oral acetazolamide
C. Topical acetazolmaid and topical pilocarpine
Answer: A

38. patient with unilateral painful vision loss. What is the cause?
A - papilledema
B - toxic amblyopia
C - optic neuritis
th
Answer: C | Reference: Lecture note on ophthalmology, 9 edition
39. A patient with eye movement abnormalities and ptosis. What is the nerve is involved?
rd
A-3
th
B-4
th
C-6
Answer: A
Evidence:
rd
- Oculomotor (3 cranial nerve): Eye movement (SR, IR, MR, IO), pupillary constriction (sphincter pupillae: Edinger-Westphal
nucleus, muscarinic receptors), accommodation, eyelid opening (levator palpebrae). Loss of levator palpebrae function  ptosis
th
- Trochlear (4 cranial nerve): Eye movement (SO)
th
- Abducens (6 cranial nerve): Eye movement (LR)
Reference: FA USMLE step 1

40. Patient came for annual check-up & found to have cupping of disk. What is the diagnosis?
A - Retinal detachment
B - Diabetic retinopathy
C - Chronic open angle glaucoma

Answer: C | Reference: http://www.glaucoma.org/treatment/optic-nerve-cupping.php

41. Adult complaining of pain when moving the eye, fundoscopy reveal optic disc swelling. What is the most likely diagnosis?
A - Optic neuritis.
B - Central vein occlusion.
C - Central artery occlusion.
Answer: A (Central vein and artery present with painless)
42. A patient presented with pterygium. What is the possible complication?
A - Corneal scarring
B - Corneal perforation

133
C - Strabismus

Answer: A | Reference: http://eyewiki.aao.org/Pterygium#Complications

43. a 47-year-old man with HTN & DM, past history of treatment of mycobacterial infection. He gave history of blurred and
decreased vision. On examination there is a flame shape on the retina, cotton wool spots and macular oedema (Other
investigations were included as well). What is the cause?
A - Retinal vein occlusion.
B - Ethambutol.
C - DM.

Answer: C
Evidence: Diabetic retinopathy is common in patients with chronic poor glycaemic control. Features include:
Microaneurysms (The earliest clinical sign of diabetic retinopathy), Dot and blot haemorrhages, Flame-shaped haemorrhages,
Retinal oedema and hard exudates, Cotton-wool spots, Venous loops and venous beading [Their occurrence is the most significant
predictor of progression to proliferative diabetic retinopathy (PDR)], & Macular oedema (Leading cause of visual impairment in
patients with diabetes) | Reference: http://emedicine.medscape.com/article/1225122-overview

44. Pic about periorbital swelling, what other symptoms could the patient have?
A - discharge
B - ↓ vision
C - uveitis

Answer: B (decrease vision if they mean (eye cellulitis) in this scenario)


Evidence: Symptoms and signs of orbital cellulitis include swelling and redness of the eyelid and surrounding soft tissues,
conjunctival hyperemia and chemosis, decreased ocular motility, pain with eye movements, decreased visual acuity, and proptosis
caused by orbital swelling
Reference: http://www.merckmanuals.com/professional/eye-disorders/orbital-diseases/preseptal-and-orbital-cellulitis

45. long case of eye trauma then repair then developed most likely endophthalmitis done enucleation, found in the uvea
lymphocyte and CD4 I think, what's is the cause? (Repeated)
A - cross reactivity
B - activation of lymphocytes
C - antigen release

Answer: C (antigen release)

46. Case of patient with progressive painless visual loss -8 6/18 is it:
A - Physiological
B - Pathological
C - Curvature Index alized area sta
Answer: B (pathological which could be keratoconus)
47. when to refer conjunctivitis to ophthalmologist:
A - Photophobia
B - Bilateral conjunctivitis
C - Mucopurulent discharge

Answer: A
Evidence: Ophthalmologist referral is indicated for any patient with conjunctivitis presented with one of the following Sx:

134
eye tenderness, Difficult seeing clearly, Difficult keeping the eyes open or sensitivity to light, Severe headache with nausea, Recent
trauma to the eye, Use of contact lenses. Reference: http://www.uptodate.com/contents/conjunctivitis-pinkeye-beyond-the
basics?source=outline_link&view=text&anchor=H8#H8

48. Patient had eye pain and photophobia upon examination by slit lamp there was circumcorneal congestion, keratic
precipitates and posterior corneal opacity. what is the diagnosis:
A - anterior uveitis
B - acute keratitis
C - acute angle glaucoma

Answer: A | Reference: http://emedicine.medscape.com/article/798323-clinical#b4

49. cup picture attached asking for the diagnosis: (no stem of Q, answer not known)
A - Glaucomatous
B - Papillitis
C - Diabetic retinopathy

Answer: see picture of each disease.

50. Child has red eye and tearing since birth:


A - Congenital glaucoma
B - herpes keratitis
C - bacterial conjunctivitis

Answer: A | Reference: Ophthalmology Consultant

51. Distichiasis?
A. Malrotation of eye lashes
B. Pigmentation of eyelashes
C. Extra row of eyelashes
Answer: C
Explanation: defined as the abnormal growth of lashes from the orifices of the meibomian glands on the posterior lamella of the
tarsal plate (see following image).

Reference: http://emedicine.medscape.com/article/1212908-overview

52. Patient with vesicles in forehead and supraorbital region for one day, what will you do?
A. Antiviral
B. Antiviral and refer to ophthalmologist.
C. Reassure
Answer: B
135
Explanation: The cause most likely is Herpes zoster ophthalmicus, it presents with dermatomal forehead rash and painful
inflammation of all the tissues of the anterior and, rarely, posterior structures of the eye.
Reference: http://www.merckmanuals.com/professional/eye-disorders/corneal-disorders/herpes-zoster-ophthalmicus

53. Best to diagnosis corneal abrasion?


a) Fluorescein eye test
b) Slit lamp test
c) Others
Answer: A

54. Lower visual field loss with flashes and other symptoms, what is the Dx?
A. HTN
B. DM
C. Retinal detachment
Answer: C

55. Patient came with feeling of foreign body sensation after removing fly from eye. What to give him?
A. Topical antibiotics
B. Oral antibiotics
C. Topical steroids

Answer: A
this sounds like foreign body induced corneal abrasion which should be treated by removing the foreign body then topical antibiotics
(erythromycin)
http://bestpractice.bmj.com/best-practice/monograph/500/treatment/details.html

56. Pic of eye with dendritic lesion Stained by fluorescein.. Management?


A) Antiviral
B) Antifungal
C) Others
Answer: A
Explanation: Most likely caused by Herpes Simplex virus, treat with Antiviral.
Reference: http://bestpractice.bmj.com/best-practice/monograph/561/treatment/details.html#expsec-13

57. child with eye itching for one month (no other symptoms in scenario), and have Hx of asthma, what's the Dx?
A. Bacterial conjunctivitis
B. Viral conjunctivitis
C. Venral conjunctivitis
Answer: C
Vernal conjunctivitis is long-term (chronic) swelling (inflammation) of the outer lining of the eyes. It is due to an allergic reaction.
often occurs in people with a strong family history of allergies. These may include allergic rhinitis, asthma, and eczema
https://www.nlm.nih.gov/medlineplus/ency/article/001390.htm

58. High myopia tx


A) leser ...-
B) refractory ...
C) lens - ...
Answer: phakic Intraocular lens?

136
59. Case of patient having irritation in the eyes with watery discharge. ..what usually associated with it
A) Enotropia
B) Ectropia
C) Strabismus
Answer: A

60. Patient around 50 y with unilateral eye discoloration they give discretion of something in the sclera I think! What you’ll tell
him! I DON” T KNOW
A) he’s in high risk of visual loss
B) this is a manifestation of systemic disease
C) vitamin A deficiency
Answer: Incomplete, I think it is bitot spot and is associated with vit. A deficiency

61. Increase ICP, with bilateral impaired abduction;


A. Oculomotor
B. Abducens
C. Trochlear

Answer: B
affects the lateral rectus muscle, impairing eye abduction. The eye may be slightly adducted when the patient looks straight ahead.
http://www.merckmanuals.com/professional/neurologic-disorders/neuro-ophthalmologic-and-cranial-nerve-disorders/sixth-cranial-
nerve-palsy

62. Child opaque lens with signs of inflammation?


A. Cataract
B. Neuroblastoma
C. Retinoblastoma
Answer: C

63. Patient with exophthalmos and swollen lids and you can feel its pulse, TFT normal. What’s your diagnosis?
A. Hyperthyroid
B. Cellulitis
C. Cavernous sinus thrombosis
Answer: C
source: The wills eye manual 6th edition

64. Post cataract surgery ciliary injection & fever, normal extra ocular movements;
A. Endophthalmitis
B. Panophthalmitis
C. Cellulitis

Answer: A
Endophthalmitis: after one-day vs Sterile postoperative inflammation: in same day
Cellulitis: decrees range of movement
http://emedicine.medscape.com/article/1201260-overview

65. Patient has a cat and then she C/O eye pain and erythema in the eye?
A. Allergic conjunctivitis
B. Contact dermatitis
137
C. Bacterial conjunctivitis
Answer: A

66. During examination of the eye of uncontrolled diabetic patient ,what you may find ?
A. Central Retinal artery ischemia (I think)*
B. Glaucoma
C. Papiledema
Answer: B
http://www.glaucoma.org/glaucoma/diabetes-and-your-eyesight.php
Central Retinal artery ischemia patients might have history of atrial fibrillation, endocarditis, coagulopathies, atherosclerotic
disease, hypercoagulable state

67. At day care center 10 out of 50 had red eye in the 1st week, another 30 develop same condition in the next 2 nd wk ,,,what's
the attack rate?!
A. 40%
B. 60%
C. 80%

Answer: C

68. Pic of congested eye, history of URTI one week ago , eye has watery discharge not purulent , what will support what is the
Diagnosis: (What is the most likely you will find ?)
a. Papilla
b. Follicles
c. Papilloma
Answer: B
Explanation: Follicles usually seen in viral and chlamydial infections; Symptoms make viral conjunctivitis most likely. Patients with
adenoviral conjunctivitis may give a history of recent exposure to an individual with red eye at home, school, or work, or they may
have a history of recent symptoms of an upper respiratory tract infection.
Reference: http://emedicine.medscape.com/article/1191370-clinical
Toronto Notes 2015, page OP15, ophthalmology

69. A passenger on a bus sustains an injury to his eye with the newspaper of a neighboring person. His vision becomes blurry and
he develops redness and pain in his eye.
A. Abrasion
B. Laceration
C. Infiltration
Answer: a
Corneal abrasion is the most common type of injury involving the cornea. Contact with dust, dirt, sand, wood shavings, metal
particles or even an edge of a piece of paper can scratch or cut the cornea. Because the cornea is extremely sensitive, abrasions can
be painful. In case of such an injury, prompt medical care should be obtained.
http://emedicine.medscape.com/article/1195402-overview
70. Retinoblastoma affect the vision MRI show intact optic N (something like that) Ttt?
A. Chemotherapy
B. Steroid
C. Removal of the eye

Answer: A

138
Chemoreduction — Since most retinoblastomas are large at the time of presentation, chemoreduction is often used to reduce
tumor volume, which enhances the success of local therapies. Chemoreduction has become a critical component of the
initial treatment of retinoblastoma and has improved the ocular salvage rate [69,106-111]. The most common
chemoreduction regimen contains carboplatin, vincristine, and etoposide given approximately every four weeks.
FYI: Enucleation (removal of the eye) usually is indicated for large tumors (>50 percent of globe volume) with no visual potential,
blind, painful eyes, and/or tumors that extend into the optic nerve
Refrence: http://cursoenarm.net/UPTODATE/contents/mobipreview.htm?28/14/28897

71. Esotropia 25 degree, initial management:


A. Orthoptic
B. Exercise prism
C. Medial muscle recession
Answer: C
In patients with acquired esotropia, surgery is indicated when the deviation is greater than 15 PD and stable.
http://emedicine.medscape.com/article/1198784-treatment#d6

72. Patient with tumor affect eye what is the cause ?


C. Ewing sarcoma
D. Neuroplastoma
E. Nephroblastoma
Answer:B

Metastasis is most commonly from breast and lung in adults, neuroblastoma in children. Usually infiltrate the choroid, but may also
affect the optic nerve or extraocular muscles. Patients present with decreased or distorted vision, irregularly shaped pupil, iritis,
hyphema.
Reference: Toronto notes

73. patient c/o red eye and tearing but there is no itching ,hx of trauma , what u will do ?
A- Give him topical AB
B- topical corticosteroid
C- reassure
Answer: A (C is also possible)
Explanation: Since there is history of trauma, along with redness and tearing, patient most likely has corneal abrasion. Treatment:
topical antibiotic, consider topical NSAID, cycloplegic, consider patch (if no contact lenses)
Most abrasions clear spontaneously within 24-48 h (so, answer C is possible)
Reference: Toronto Notes 2015, page OP17, ophthalmology

74. women came complain of eye pain case her daughter scratch her eye with her nail , there is picture of (i think slit lamp shows
abrasion on cornea), what to give?
A. drop steroid
B. ointment steroid
C. contact lens
answer: A, antibiotic, eye drops- NSAIDS
Corneal abrasions heal with time. Prophylactic topical antibiotics are given in patients with abrasions from contact lenses.
Traditionally, topical antibiotics were used for prophylaxis even in noninfected corneal abrasions not related to contact lenses, but
this practice has been called into question. Some ophthalmologists advocate the use of diclofenac (Voltaren) or ketorolac (Acular)
drops (NSAIDS).
Medscape: http://emedicine.medscape.com/article/1195402-overview

75. Picture with history of eye inflammation, no ophthalmoplegia :


139
A. Chalazion
B. Stye
C. Orbital cellulitis
Answer: A or B according to the picture and scenario

Answer: see pictures to match it with the description.


Explanation:
- chalazion is noninfectious obstruction of a meibomian gland causing extravasation of irritating lipid material in the eyelid soft
tissues with focal secondary granulomatous inflammation. Disorders that cause abnormally thick meibomian gland secretions
(eg, meibomian gland dysfunction, acne rosacea) increase the risk of meibomian gland obstruction.
- Hordeolum (stye) is an acute, localized swelling of the eyelid that may be external or internal and usually is a pyogenic (typically
staphylococcal) infection or abscess. Most hordeola are external and result from obstruction and infection of an eyelash follicle
and adjacent glands of Zeis or Moll glands. Follicle obstruction may be associated with blepharitis.

Reference: Merck’s manual. Pic from Google

76. Retinal vessels displaced, old age come with headache ?


A. Acute angle glaucoma
B. Diabetic retinopathy
C. Macular degeneration
(No open angle glaucoma in choices)
Answer: might be A, headache goes more with that!
140
77. The most presenting signs of retinoblastoma:
A. Leukocoria
B. Nystagmus
C. Strabismus
Answer: A
Reference: http://emedicine.medscape.com/article/1222849-clinical#b4
The most common presenting sign in Fundoscopy is white pupillary reflex (Leukocoria)

78. Elderly female, came with right eye pain, vomiting...


A. Hyperthyroid
B. Acute angle closure glaucoma
C. Hyphema
Answer: B
Acute angle closure glaucoma: Red, Painful eye, unilateral. Decreased visual acuity, blurred vision due to corneal edema. “Halos
around light”. N&V, abdominal pain. Fixed mid-dilated pupil. Management: laser iridotomy. – Toronto Notes Optha
Hyphema: http://emedicine.medscape.com/article/1190165-overview#showall

79. Pt. with controlled DM & HTN see flashes and loss part of visual field?
A. Retinal detachment
B. Diabetic retinopathy
C. Hypertensive retinopathy
Answer: A
Initial symptoms commonly include the sensation of a flashing light (photopsia) related to retinal traction and often accompanied by
a shower of floaters, shadow in the peripheral visual field, and vision loss.
http://emedicine.medscape.com/article/798501-clinical
80. SCD patient comes with sudden painless loss of vision cloudy retina and cherry red spot finding the macula. What is your
diagnosis?
A. Retinal artery occlusion
B. Retinal vein occlusion
C. Retinal detachment
Answer: A
Typical funduscopic findings of a pale retina with a cherry red macula (ie, the cherry red spot) result from obstruction of blood flow
to the retina from the retinal artery, causing pallor, and continued supply of blood to the choroid from the ciliary artery, resulting in
a bright red coloration at the thinnest part of the retina (ie, macula).
For more reading:
http://emedicine.medscape.com/article/799119-clinical - b5
- Coagulopathies from sickle cell anemia or antiphospholipid antibodies are common etiologies for central Retinal artery occlusion in
patients younger than 30 years

81. Eye surgery for high myopia in one eye:


A. LASIK
B. IOL
C. PRK
*Most likely answer: Phakic intraocular contact lenses
http://emedicine.medscape.com/article/1221828-overview#a9
http://emedicine.medscape.com/article/1221604-overview#a8
(In the above links you will find indications for both LASIK and PRK but not sure what the answer is.)
141
82. Iris neovascularization caused by? (missing options)
A. Non-proliferative diabetic retinopathy.
B. Central retinal vein occlusion

Answer: B
Evidence:
- The three most common causes are: DM, Central retinal vein occlusion and carotid artery obstructive disease.
Reference: http://www.aao.org/eyenet/article/diagnosis-treatment-of-neovascular-glaucoma?julyaugust-2006

- Abnormal iris blood vessels (neovascularization) may obstruct the angle (secondary glaucoma) and cause the iris to adhere to the
peripheral cornea, closing the angle (rubeosis iridis). This may accompany proliferative diabetic retinopathy or central retinal vein
occlusion due to the forward diffusion of vasoproliferative factors from the ischaemic retina.
th
Reference: Lecture note on ophthalmology, 9 edition

83. commonest cause of iris neovascularization is: (missing options)


A. DM
B. HTN

Answer: A

84. Corneal epithelial defects (CED), in addition to this presentation, what else this patient might have? (missing options)
A. Photophobia
B. Visual loss

Answer: A | Reference: http://eyewiki.aao.org/Corneal_Epithelial_Defect#Management

85. Long term topical steroid drops can cause: (missing options)
A. Glaucoma.
B. posteior subcapsular cataract

Answer: A | Reference: http://www.ncbi.nlm.nih.gov/pmc/articles/PMC4228634/table/T1/

86. Trabeculectomy is an operation used for which of the following conditions? (missing options)
A. Open angle glaucoma
B. Closed angle glaucoma

Answer: A
Evidence: Trabeculectomy is effective for chronic angle-closure glaucoma. However, compared to primary open-angle glaucoma, any
aqueous-draining procedure in an eye with a shallow anterior chamber and a chronic closed angle poses the risk of further
shallowing the anterior chamber or precipitating malignant glaucoma.
Reference: Toronto notes; http://emedicine.medscape.com/article/1205154-treatment#showall

142
87. A patient with painful swelling (dacryocystitis). What is the best management? (missing options, answer not known)
A. Oral antibiotic
B. Drain surgically

Answer: ?
Evidence: In general, dacryocystitis is a surgical disease. Surgical success rates in the treatment of dacryocystitis are approximately
95%. Acute cases are best treated surgically after the infection has subsided with adequate antibiotic therapy.
For acute dacryocystitis, an external dacryocystorhinostomy is preferred after several days of initiating antibiotic therapy.
Rarely, dacryocystorhinostomy must be performed during the acute phase of the infection to facilitate clearing of the infection.
Reference: Medscape.

88. Father came with his 6 years old daughter to the clinic, she has esotropia. What will you do? (missing options, no answer)
A. Glasses.
B. Surgery.
Answer: ? [Nonsurgical treatments include patching, correction of full hyperopic refractive error (glasses) then if not improved
surgery] | Reference: http://eyewiki.aao.org/Accommodative_Esotropia#General_treatment

89. (pic) Eye lid inflammation what most associated symptoms: (missing options)
A. secretion
B. vision loss

Answer: A | Reference: http://emedicine.medscape.com/article/1211763-clinical


90. stage of hypertensive or increase IOP, ask about third stage optic change? (missing details of Q, missing options, no answer)
A. Papilloedema
B. nipping of vein

Answer: ?
Evidence:
Grade I: mild arteriolar narrowing
Grade II: arteriovenous crossing, AV nicking
Grade III: retinal haemorrhage, exudate, cotton wool
Grade IV: papilloedema
Reference: http://www.retinalphysician.com/articleviewer.aspx?articleID=109106

91. Bacterial keratitis occurs with contact lenses microorganism: (missing options)
A. S. Aurea (in general)
B. Pseudomonas Aeruginosa (contact lens wearer)

Answer: B | Reference: http://www.ncbi.nlm.nih.gov/pubmed/3942549/ ; http://www.aao.org/eye-health/diseases/what-is-


bacterial-keratitis

92. Painless loss of vision in right eye with headache. First thing to do? (missing options)
A. Prednisolone
B. CT

Answer: A (Prednisolone)

143
93. Type of gonorrhoea discharge in eye: (missing options)
A. Purulent
B. Mucopurulent if chlamydia

Answer: A
94. Conjunctivitis patient with bottom eyelash turned inwards? (missing options, answer not clear from options)
A. Ectropian
B. Entropian

Answer: ? (eyelashes turned inward: trichiasis; Lid margin turned inward: entropion)
Reference: http://emedicine.medscape.com/article/1212456-overview

95. A patient who presented with ciliary flush, bilateral eye redness and pain. Examination revealed keratic precipitate and
presence of cells in the anterior chambers (classical scenario of uveitis) what is the treatment:
A. systemic corticosteroid
B. cyclopentolate with topical steroids
Answer: B
Source: Toronto notes 2014 OP20

96. Patient struck her eyes by her finger and came with burning pain and crying, what is the suspected symptom or
complications:
A. Blindness
B. Photophobia

Answer: B (missing options)


Evidence:
- symptoms associated with corneal abrasion are: pain, blurred vision, tearing and photophobia.
- complication include: bacterial keratitis, corneal ulcers, traumatic iritis, and recurrent erosion syndrome.

97. How you test trochlear nerve :


A. Adduction and downward
B. Abduction and downward

Answer: A
http://teachmeanatomy.info/head/cranial-nerves/trochlear-nerve/

98. Patient presented with dilated pupils, nausea vomiting and abdominal pain. What is the most likely diagnosis?
A. digoxin toxicity
B. angle glaucoma

Answer: B
classic presentation of Acute angle closure glaucoma.
http://emedicine.medscape.com/article/798811-clinical

99. An old man presents after a crash with blurred vision, on o/c: anterior chamber clouded or something ...what causes these
symptoms?
A. Retinal detachment
B. Cataract.
Answer: B
144
100.Case of acute glaucoma, what is the treatment?
A. Acetazolamide
B. Timolol
Answer: A
Explanation: Carbonic anhydrase inhibitors decrease aqueous humour formation and are used commonly as first-line therapy
(generally managed based on the cause and initially will be both topical and systemic treatment.)
Reference: http://bestpractice.bmj.com/best-practice/monograph/372/treatment.html

101.He was playing tennis and he get a force on his eye .. There was a bleeding in anterior chamber of the eye .. Which of the
following you must exclude first?'
A) Presence of Foreign body
B) sorry I forget the other answer
Answer: increase in intraocular pressure
Reference: http://www.healthline.com/health/hyphema#Overview1

102.How to prevent eye infection?


A. eye patches
B. hand hygiene
Answer: B
 Generally avoiding close contact with infected individual and their eye secretions, as they are highly contagious in case of
bacterial and viral conjunctivitis.
 Infected individual are advised of frequent hand washing, avoiding sharing towels and linens, avoiding public pools.
 Workers and students often are excused from work or school during the first several days of treatment to decrease the
possibility of spread.
Source: http://emedicine.medscape.com/article/1191730-treatment#d11

103.lateral movement of one eye of a child when you close the other eye?
A. squint (strabismus)
B. nystagmus
Answer: A.
Strabismus is misalignment of the eyes, which causes deviation from the parallelism of normal gaze.
By the direction of the squinting (turning) eye:
An eye that turns inwards is called an esotropia.
An eye that turns outwards is called an exotropia.
An eye that turns upwards is called a hypertropia.
An eye that turns downwards is called a hypotropia.
104.patient presented with foreign body sensation in his eyes, itchiness and grittiness. (not sure if mentioned watery discharge or
no). he is taking (a drug) and antihistamine. Also he mentioned that he sit in front the screen for six hours daily. What is the
cause of his disease? ( the scenario was not very clear whither it is allergic or just simple dryness)
A. mast cell degranulation and histamine release.
B. Corneal dryness
Answer: b
http://emedicine.medscape.com/article/1210417-overview

105.sudden onset of proptosis limited eye movement pain swelling vision is normal
A. orbital cellulitis,
B. cavernous sinus thrombosis
Answer: A
145
Toronto note 2015

106.pic of eye (there is proptosis and redness of upper eyelid on one eye) what is diagnosis;
A. Orbital cellulitis
B. Chalazion
Answer: A
The question has missing details like the presence of fever, pain on movement but in general Proptosis and ophthalmoplegia are the
cardinal signs of orbital cellulitis.
Reference: http://emedicine.medscape.com/article/1217858-clinical#showall

th
Reference: Lecture note on ophthalmology, 9 edition

107.Eye picture red conjunctiva with white ring around cornea


A. Vernal conjunctivitis
B. Chlamydia
answer: A
VKC is characterized by symptoms consisting of severe itching, photophobia, foreign body sensation, mucous discharge (often
described as “ropy”), blepharospasm, and blurring of vision
Classic signs of allergic conjunctivitis include injection of the conjunctival vessels as well as varying degrees of chemosis (conjunctival
edema) and eyelid edema. The conjunctiva often has a milky appearance due to obscuration of superficial blood vessels by edema
within the substantia propria of the conjunctiva.
http://emedicine.medscape.com/article/1191467-overview

146
108.Boy has a cat developed itching for a month with Red eye and watery with discharge No lymphadenopathy and general exam
normal:
A. Cat scratch Dermtitis Allergic
B. Conjunctivitis
Answer: B
reference: http://emedicine.medscape.com/article/1191467-clinical

109.what is the diagnosis of DM patient with history of sudden unilateral vision loss pupil afferent affected -retinal hemorrhages
and macular edema?
A. retinal artery occlusion -
B. retinal vein occlusion
answer:b
reference : https://quizlet.com/20317777/ophthalmology-flash-cards/

110.patient had red eyes, tearing, change in pupil shape didn't improve with patch:
A. keratitis
B. glaucoma
Answer:?

111.HIV patient for eye exam. Upon examination he’s shown to have cotton wool spot appearance. (Other details of eye exam
are missing) Which virus may cause this?
A. HIV
B. CMV
Answer: B
 CMV retinitis is the most common intra-ocular infection in HIV-infected patients.
Source: http://www.icoph.org/med/ppt/hiv.pdf http://emedicine.medscape.com/article/1227228-overview

112.Patient with Post cataract surgery he came with Painful eye , yellow, red reflux , muscle intact what is 1DDx?
A. Enophalamitis
B. Cellulitis
Bacterial endophthalmitis (see the image below) is an inflammatory reaction of the intraocular fluids or tissues caused by microbial
organisms. On physical examination, general findings in bacteria endophthalmitis are as follows: Hypopyon, Loss of red reflex …etc
http://emedicine.medscape.com/article/1201134-overview

113.56 male patient came with Pterygium , what you will tell him ?
A. Pre malignant lesion
B. Affect his vision
Answer: b
fibro vascular triangular encroachment of epithelial tissue onto the cornea, usually nasally
may induce astigmatism, decrease vision / torento note 2015
http://emedicine.medscape.com/article/1192527-overview

147
114.severe acute onset eye pain fixed pupil hazy cornea , redness o think , what u will give :
A. IV Acetazolamide
B. Topical Pilocarpine
Acute Angle-Closure Glaucoma Look for the sudden onset of an extremely painful, red eye that is hard to palpation. Walking into
a dark room can precipitate pain because of pupilary dilation. The cornea is described as “steamy” and the pupil does not react to
light because it is stuck. The cup-to-disc ratio is greater than the normal 0.3. The diagnosis is confirmed with tonometry. Treat with:
· Intravenous acetazolamide, Intravenous mannitol , Pilocarpine, beta blockers, and apraclonidine to constrict the pupil and
enhance drainage , Laser iridotomy (Master the boards)

115.Case with red eyes with watery discharge, there is history of recurrent of similar attack, which can aggravate this condition:
A. Pollens exposure
B. Ultraviolet exposure
Answer: A, allergic conjunctivitis

116.Headache, pain in Rt eye ,Halos around light :


A. Hyphema
B. digoxin toxicity
Answer: Glaucoma or Cataracts - Not mentioned in the options/Scenario missing details
Explanation: Halos are troublesome, bright circles of light that surround headlights and other light sources.Halos Around Lights may
be associated with:
● Cataract
● Fuchs' Dystrophy
● Glaucoma
● Kerataconus
Link: http://www.aao.org/eye-health/symptoms/halos-around-lights
http://www.webmd.com/eye-health/halos-and-glare-causes-prevention-treatment

117.Which of the following can cause painful loss of vision?


A. Retinal detachment
B. Acute angle glaucoma
Answer: B
Acute angle glaucoma presents with a red painful eye which can progress to irreversible loss of vision within hours if left untreated.
Retinal detachment presents as sudden onset flashes of lights and floaters followed by peripheral field loss (curtain of blackness)
and loss of central vision if macula was off.
Toronto notes 2015

148
118.Patients complains of unilateral eye pain, redness and photophobia. What is the most likely diagnosis?
A. Keratitis
B. Uveitis
Answer: B
Acute iritis Keratitis
Discharge No Profuse tearing
Pain ++ (tender globe) ++ (on blinking)
Photophobia +++ Varies
Blurred vision ++ ++
Pupil Smaller Same or smaller
Injection Ciliary flush Diffuse
Cornea Kertic precipitates Infiltrate, edema, epithelial defects
Intraocular pressure Varies Normal or increased
Anterior chamber +++ cells and flares Cells and flares or normal
Other Posterior synechiae
Toronto notes 2015

119.Neovascularization on the retina, caused by?


A. DM proliferative Retinopathy
B. HTN
Answer: A
Reference: http://emedicine.medscape.com/article/1225122-overview#a3

120.Diabetic controlled on medication developed (eye down and out) dropped upper eyelid limited movement of eye medially
and up and down.
A. Right oculomotor palsy
B. Right facial palsy
Answer: A
http://emedicine.medscape.com/article/1198462-overview

121.Picture of Snellen's test. What is the visual level of this patient?


Answer: ? (missing details of Q, missing options, answer not known)
e.g. A patient can read until the 3rd line, the visual acuity is 20/70.

122.Lacrimal gland tumour lead to proptosis in which direction? (missing options)

149
A - Down\in
Answer: A
Evidence:
- Maxillary sinus growth: Superior
- Lacrimal gland tumour: Inferomedial
- Frontal or ethmoidal sinus lesion: Inferolateral
Reference: Textbook of Ophthalmology

123.What is the treatment of chronic ptosis? (missing options, answer not known)
Answer: ? | Reference: http://eyewiki.aao.org/Aponeurotic_ptosis#General_treatment

124.Corneal epithelial defects (CED) (Another term of corneal abrasion or ulcer) due to fingernail injury. What is the treatment?
A - Double patch. (missing options)
Answer: A | Reference: http://eyewiki.aao.org/Corneal_Epithelial_Defect#Management & an intern who had the same Q in his
exam and got the full mark in ophthalmology section

125.How to administer eye drops and ointments? (missing options)


A - One drop in the lower fornix
Answer: A | References: http://patient.info/doctor/eye-drugs-prescribing-and-administering ;
https://www.nursingtimes.net/download?ac=1289059

126.A patient came with drooling and lacrimation (cholinergic reaction). What to give? (missing options, answer not known)
- Physostigmine
Answer: ? (No atropine in the choices; Physostigmine is NOT the right answer, it is an acetylcholinesterase inhibitor (It can be the
cause of the cholinergic reaction)

127.A patient with follicular keratosis. What will you check? (missing options)
A - Eyes
Answer: A (Because of Vit. A deficiency; Keratosis pilaris is another name of follicular keratosis)
Reference: http://disorders.eyes.arizona.edu/category/clinical-features/keratosis-pilaris

128.A patient with increased ICP. What will you check? (missing options)
A - Papilledema
Answer: A

129.Eye drops that is contraindicated in acute closed angle glaucoma? (missing options, answer not known)
Answer: ? (α1-agonists (Epinephrine) because of Mydriasis side effect) | Reference: FA USMLE step 1
Explanation: The pathophysiology of drug-induced angle-closure glaucoma is usually increased pupillary block (ie, increased iris-lens
contact at the pupillary border) from pupillary dilation. The classes of medications that have the potential to induce angle closure
are topical anticholinergic or sympathomimetic dilating drops, tricyclic antidepressants, monoamine oxidase inhibitors,
antihistamines, antiparkinsonian drugs, antipsychotic medications, and antispasmolytic agents.

130.A male patient presented with breast enlargement and decreased sexual desire. Labs: Hyperprolactinemia. What visual
deficiency this patient might have? (missing options)
A - Bitemporal hemianopia
Answer: A

131.How to test visual acuity? (missing options)

150
A - Snellen chart
Answer: A | Reference: Toronto notes

132.A patient comes with signs and symptoms of glaucoma. What is treatment? (missing options, answer not known)
Answer: ? (IV acetazolamide, topical pilocarpin and B blocker) | Reference: UQU; Toronto notes

It depends on the type of Glaucoma, whether Open or Close Angle Glaucoma. In any case, the goal of treatment is reduction of the
pressure before it causes progressive loss of vision.
Major drug classes for medical treatment: alpha-agonists, beta-blockers, carbonic anhydrase inhibitors, miotic agents, and
prostaglandin analogs. Also surgical options include trabeculectomy or aqueous shunt which is most commonly used.
http://emedicine.medscape.com/article/1206147-overview

133.Diabetic patient with high cup to disc ratio. What is the cause? (missing options, answer not known)
Answer: ?
Evidence: Patients with DM were previously thought to have a greater risk of developing primary chronic glaucoma with loss of
visual field. However, more recent papers suggest that DM is not a greater risk factor, but simply that glaucoma was found more
readily. Patients with PDR are at risk of developing secondary glaucoma, particularly neovascular (rubeotic) glaucoma. Rubeosis iridis
is the growth of new vessels on the iris in eyes with advanced retinal ischaemia. Rubeosis – neovascularization of iris (NVI) may
induce a severe form of intractable glaucoma with growth of new vessels in the anterior chamber angle (NVA). If uncontrolled, NVA
leads to closure of the aqueous fluid drainage route in the anterior chamber angle of the eye by fibrovascular tissue.
Reference: http://www.icoph.org/dynamic/attachments/taskforce_documents/2012-sci-
267_diabetic_retinopathy_guidelines_december_2012.pdf

134.(pic) Eye with entropian: (no stem of Q, missing options)


Answer: Entropion is a malposition resulting in inversion of the eyelid margin.

151
135.orbital cellulitis 2 times: (no stem of Q, missing options)
Answer: Orbital cellulitis and preseptal cellulitis are the major infections of the ocular adnexal and orbital tissues. Orbital cellulitis is
an infection of the soft tissues of the orbit posterior to the orbital septum, differentiating it from preseptal cellulitis, which is an
infection of the soft tissue of the eyelids and periocular region anterior to the orbital septum. Patients with orbital cellulitis
frequently complain of fever, malaise, and a history of recent sinusitis or upper respiratory tract infection. Other signs include:
Conjunctival chemosis, Decreased vision, Elevated intraocular pressure, Pain on eye movement. The patient with orbital cellulitis
should be promptly hospitalized for treatment, with hospitalization continuing until the patient is afebrile and has clearly improved
clinically. Medical management is successful in many cases. Consider orbital surgery, with or without sinusotomy, in every case of
subperiosteal or intraorbital abscess formation.

136.Ring lesion in eye: (missing options)


A - Acanthamoeba infection
Answer: A | Reference: http://eyewiki.aao.org/Acanthamoeba_Keratitis#Signs
It could be Kayser-Fleischer Ring (brownish-green copper ring)

137.keratitis caused by parasites: (missing options)


Answer: Acanthamoeba | Reference:
http://www.cdc.gov/parasites/acanthamoeba/health_professionals/acanthamoeba_keratitis_hcp.html

Acanthamoeba keratitis is characterized by pain out of proportion to findings. In one study, 95% of patients complained of pain.
Patients may also complain of decreased vision, redness, foreign body sensation, photophobia, tearing, and discharge. Symptoms
may wax and wane; they may be quite severe at times.
Reference: http://eyewiki.aao.org/Acanthamoeba_Keratitis
- Skin manifestations: ulcers, nodules, or subcutaneous abscesses.
- CNS: meningeoencephalitis ;Mental status changes, Seizures, Hemiparesis, Fever, Headache, Meningismus, Visual disturbances,
Ataxia, Nausea and vomiting, Hallucinations, Personality change, Photophobia, Sleep disturbances.
Disseminated disease without CNS involvement may manifest as skin lesions, sinusitis, pneumonitis, or a combination.
Reference: Medscape

138.what test is used for distant vision: (missing options)


Answer: visual acuity

139.Red eye pic after pharyngitis and fever, what is your dx? (missing options)
Answer: adenovirus conjunctivitis
Evidence: Pharyngitis caused by adenovirus is common among young children and military recruits. Patients with pharyngitis present
with sore throat (more intense than that of a common cold), high fever, dysphagia, and red eyes
Reference: http://emedicine.medscape.com/article/225362-clinical
140.(pic) disc cup ask for diagnosis: (missing details of Q, missing options, answer not known)
Answer: ? (The normal cup-to-disc ratio is 0.3. A large cup-to-disc ratio may suggest glaucoma or other pathology)

141.(pic) of cornea after nail injury, diagnosis? (missing options)


Answer: corneal abrasion

142.Sudden eye swelling redness and pain, hazy cornea: (no stem of Q, missing options)
Answer: Acute angle closure glaucoma | Reference: introduction to clinical emergency medicine text book ; UpToDate

143.Case of chronic eye irritation and Watery secretion + eye Enotropia Enotropia: (no stem of Q, missing options)
Answer: conjunctivitis

152
144.diabetic female come to the Ophthalmologist with Rt eye pressure 44 mmHg, Lt eye pressure 22 mmHg , examination of
retina of Rt eye show disc to rim ratio and nasal deviation of retinal vessels .what is the most likely Dx? (missing options)
Answer: Glaucoma | Reference: Toronto

145.URTI + conjunctivitis with hyperemia + watery discharge what other ocular symptom you expect: (missing options)
A - epithelial nummular keratitis
Answer: A

146.Ectropion pic: (no stem of Q, missing options)


Answer: Ectropion is an abnormal eversion (outward turning) of the lid margin away from the globe.

147.In retinal detachment where the fluid collect?


A. Photoreceptors and retinal pigment epithelium
Answer: A
 Cleavage in the plane between the neurosensory retina and the RPE.
•Three types
1. rhegmatogenous (most common) caused by a tear or hole in the neurosensory retina, allowing fluid from the vitreous to
pass into the subretinal space, tears may be caused by PVD, degenerative retinal changes, trauma, or iatrogenically,
incidence increases with advancing age, in high myopes, and after ocular surgery/trauma
2. Tractional Š caused by traction (due to vitreal, epiretinal, or subretinal membrane) pulling the neurosensory retina away
from the underlying RPE, found in conditions such as DR, CRVO, sickle cell disease, ROP , and ocular trauma
3. Exudative, caused by damage to the RPE resulting in fluid accumulation in the subretinal space, main causes are
intraocular tumors, posterior uveitis, central serous retinopathy
reference: Toronto notes OP 24

148.Scenario of a patient with sky blue sclera?


A. Osteogenesis imperfect
Answer: A.
Osteogenesis imperfecta (OI) is a genetic disorder characterized by bones that break easily, often from little or no apparent cause.
http://www.oif.org/site/PageServer?pagename=fastfacts

149.Watery discharge congested eye preaurcual lymph node enlarge diagnosis?


Answer: Viral conjunctivitis
Explanation: From the giving symptoms the cause most probably Viral conjunctivitis (adenovirus).
Reference: http://emedicine.medscape.com/article/1191370-workup
http://www.merckmanuals.com/professional/eye-disorders/conjunctival-and-scleral-disorders/viral-conjunctivitis
Reference: http://bestpractice.bmj.com/best-practice/monograph/500/diagnosis/tests.html

153
150.corneal ulcer diagnosed by?
Answer: Diagnosis is made by slit-lamp examination.
Reference: http://www.merckmanuals.com/professional/eye-disorders/corneal-disorders/corneal-ulcer

151.Young with sudden unilateral painless eye blindness what the DIAGNOSIS ?!
Answer: review the pic in old Q
Explanation: the most common reason for painless sudden visual loss is ischemia?
Reference: http://emedicine.medscape.com/article/1216594-overview

152.Neonate presented to the hospital with eye discharge ..the mother had a vaginal discharge before delivery ,,, what is the
organism
Answer : Gonorrhea
Explanation: there are many other differentials such as silver nitrate solution and chlamydial, staphylococcal, and herpetic
infections.
Reference: http://emedicine.medscape.com/article/1192190-overview#a5

153.Patient c/o follicular keratosis, what you will look to check? Eye
a) vit. A deficiency
Answer: A

154.patient sensitive to light what is your action:


A) wear sunglasses protect form sunlight
Answer: Incomplete

155.Scenario, flashes & floaters in the eye, how to treat?


Answer: posterior vitreous detachment
http://www.geteyesmart.org/eyesmart/diseases/floaters-flashes/causes.cfm

156.Injury of vagus nerve cause which of eye symptoms??


A) One of the choises was ptosis
Answer:??

157.Herpes keratitis of eye, scenario with picture, how to treat?


Answer:
The mainstay of therapy is antiviral treatment either in the form of topical therapy with trifluridine 1% eight to nine times a day or
oral administration of acyclovir or valacyclovir for 10 to 14 days. If trifluridine drops are used, care is to be taken to ensure antiviral
drops are discontinued within 10-14 days due to corneal toxicity. Epithelial debridement of the dendrites may also be utilized in
conjunction with antiviral therapy to help reduce viral load. Topical corticosteroids are contraindicated in the treatment of active
HSV epithelial keratitis.
http://eyewiki.aao.org/Herpes_Simplex_Virus_Keratitis#Management

158.A picture of glaucomatous cupping.

154
http://www.optic-disc.org/tutorials/glaucoma_evaluation_basics/page13.html
normally 0.2- 0.4 Physiological cupping =0.5, but could be pathological if there are abnormalities by the other investigation or there
is difference btw two eyes; Pathological > 0.5

159.Treatment of high myopia, name of complicated ophthalmological surgeries?


Answer: Treatment of high myopia: Eyeglasses or Contact Lenses or Refractive Surgery.

160.Which dye used to examine the cornea?


Answer: fluorescein dye
Reference: https://www.nlm.nih.gov/medlineplus/ency/article/001017.htm

161.Cover test +, what is the complication of the condition?


Answer: Amblyopia
the patient has Squint and the treatment is batching but if the treatment was ignored or delayed the patient will develop amblyopia.
162.Patient develop 2 cm dome shape mass in the dorsum of the hand , it's cover by keration, What’s the diagnosis?
A. Keratoacanthoma.
Answer: A
Medscape:
 Lesions typically are solitary and begin as firm, roundish, skin-colored or reddish papules that rapidly progress to dome-
shaped nodules with a smooth shiny surface and a central crateriform ulceration or keratin plug that may project like a
horn.
 Most keratoacanthomas occur on sun-exposed areas. The face, neck, and dorsum of the upper extremities are common
sites.

163.Cover test to right eye cause lateral movement in left eye?


A. strabismus.
Nystagmus

164.Glaucoma drugs side effect?


Answer:
http://www.glaucoma.org/treatment/glaucoma-medications-and-their-side-effects.php

165.Vesicles on the eye and forehead?


a. Herpes zoster ophthalmicus.
Answer: A
http://www.aafp.org/afp/2002/1101/p1723.html

155
166.baby with inward deviation of rt eye , most likely due to :
Answer: refraction error
Explanation: Most strabismus is caused by Refractive error, muscle imbalance
Reference: http://www.msdmanuals.com/professional/pediatrics/eye-defects-and-conditions-in-children/strabismus

167.a case about one patient who had a bruit over the eye!
Answer: cavernous sinus thrombosis

168.Young ,very rapid reduce vision:


a-Glaucoma
reference: http://www.kellogg.umich.edu/patientcare/conditions/glaucoma.html

169.retinal tear between which layers?


Neuroretina and its pigment epithelium (RPE)
Reference:Lecture Notes Ophthalmology

170.patient with unilateral bacterial conjunctivitis , what is the antibiotic ?


A.topical erythromycin
Answer:A
The condition is usually self-limiting, although a broad-spectrum antibiotic eye drop will hasten resolution.Abe that can be used:
Trimethoprim with polymyx B, Gentamicin, Tobramycin,Neomycin, Ciprofloxacin, Ofloxacin ,Gatifloxacin ,Erythromycin.
Fluoroquinolones should be reserved for more serious infections to minimize bacterial resistance.
Reference: http://emedicine.medscape.com/article/1191730-treatment

171.Patient having esoteopia, what part of brain responsible for that ?!


Strabismus: http://patient.info/doctor/strabismus-squint

The fibers of the oculomotor nerve arise from a nucleus in the midbrain, which lies in the gray substance of the floor of the cerebral
aqueduct and extends in front of the aqueduct for a short distance into the floor of the third ventricle
https://en.wikipedia.org/wiki/Oculomotor_nucleus

172.patient compain of ptosis how can. Repaire m ?

173.A patient is complaining of the severe eye pain and ciliary congestion post operatively. What is the most likely diagnosis?
Answer is: no choices
Possible answer: Endophthalmitis
- Endophthalmitis is a common post ophthalmic operations complication.
Source: Toronto notes “page OP23”
A good source to differentiate between ciliary & conjuctival congestion: https://pgblazer.com/difference-between-
conjunctival-congestion-and-circumcorneal-congestion/

174.Ectropion or entropion (I’m not sure) 25 degrees abnormality, treatment:


Answer: no choices
156
- Entropion: (inversion of an eyelid) is caused by age-related tissue relaxation, post-infectious or posttraumatic changes,
or blepharospasm. Eyelashes rub against the eyeball and may lead to corneal ulceration and scarring. Symptoms can
include foreign body sensation, tearing, and red eye.
- Ectropion: (eversion of the lower eyelid) is caused by age-related tissue relaxation, cranial nerve VII palsy, and
posttraumatic or postsurgical changes. Symptoms are tearing (due to poor drainage of tears through the nasolacrimal
system) and symptoms of dry eyes.
- Treatment of Ectropion and entropion: surgical repair.
Source: http://emedicine.medscape.com/article/1844045-overview#showall

175.What is the pathophysiology of retinal detachment?


o Retinal detachment occurs due to a break in the retina (called a retinal tear) that allows fluid to pass from the vitreous
space into the sub retinal space between the sensory retina and the retinal pigment epithelium.
http://emedicine.medscape.com/article/798501-overview
176.Girl with crossed eyes?
o Squint

177.baby born with white eye opacity can’t see what behind lens:
- congenital cataract
A cataract is an opacification of the lens. Congenital cataracts usually are diagnosed at birth. If a cataract goes undetected in an
infant, permanent visual loss may ensue.
Reference: http://emedicine.medscape.com/article/1210837-overview

178.patient with herpetic keratitis what associated with:


- cornea h....
Read more about Herpetic Keratitis: http://emedicine.medscape.com/article/1194268-clinical#b3

179.one has blow trauma to the eye present with sub-conjunctival hemorrhage and weak up ward gaze?
Orbital base (blowout) fracture
Reference: http://emedicine.medscape.com/article/1284026-overview#a10

180.Picture of entropion, what is dx


Entropion is a malposition resulting in inversion of the eyelid margin. The morbidity of the condition is a result of ocular surface
irritation and damage.

181.Best way to reduce gonococcal conjunctivitis.


The best way to avoid it is to treat the mother and if necessary to prevent a vaginal delivery in a gonorrhea infected mother.
Erythromycin and tetracyclin drops are effective as prophylactic agents.

182.A scenario of patient with Herpetic dendrites and a picture also.


157
They asked what is associated with this condition. [Exactly the same picture]

Answer: Typically, patients with HSV keratitis present with blurry vision, extreme photophobia, pain, redness, tearing, foreign body
sensation, and conjunctival hyperemia. Sometimes vesicular blepharitis (blisters on the eyelid) follows, symptoms worsen, vision
blurs, and blisters break down and ulcerate, then resolve without scarring in about a week. Toronto note 2015

183.Eye pain & itching after 4 month regimes of TB drug.... Which drug cause that?
Answer: Ethambutol

184.COPD developed DM and acute closure glaucoma>>> which treatment:


I think they want you to avoid anticholinergic.

185.Proliferative diabetic retinopathy.

186.Q about chlamydia conjunctivitis, about prevention: (all options about water sanitization and vectors) (not related to delivery
)
by water cholorization
st
Answer: I tried to search something not related to delivery but couldn’t find except for these below. The 1 is delivery related.

Educate parents or care providers to wash their hands frequently to prevent transmission of neonatal conjunctivitis. Educate
pregnant women on the importance of regular examinations to detect and treat sexually transmitted infections such as herpes
simplex, gonorrhea, and chlamydia in order to decrease the incidence of neonatal conjunctivitis.
158
 Prophylaxis against conjunctivitis — In most of the United States, neonatal prophylaxis against gonococcal conjunctivitis is
routinely performed at birth, and required in the majority of states. However, neonatal ocular prophylaxis is not effective in
preventing neonatal chlamydial conjunctivitis [31,40-44]. The diagnosis and treatment of chlamydial infections in pregnant
women is the best method for preventing chlamydial disease

 Eye care:
Ocular infections in newborns caused by Chlamydia trachomatis are common in the United States [5]. The agents also used for
gonococcal prophylaxis are not effective in preventing neonatal chlamydial conjunctivitis. Povidone-iodine appears to be
significantly more effective against C. trachomatis than silver nitrate or erythromycin.
Technique — After wiping each eyelid with sterile cotton gauze, the prophylactic agent is placed in each of the lower conjunctival
sacs [5]. The agent should be spread by gentle massage of the eyelids, and excess solution or ointment can be wiped away after one
minute. The eyes should not be irrigated after the application because doing so may reduce efficacy.
Reference: http://cursoenarm.net/UPTODATE/contents/mobipreview.htm?6/19/6449?source=see_link&anchor=H6#H6
http://cursoenarm.net/UPTODATE/contents/mobipreview.htm?7/35/7728

187.Pic of eye (there is proptosis and redness of upper eyelid on one eye) what is diagnosis;
A. Orbital cellulitis
B. Chalazion
Answer: A
In orbital cellutis, the patient would have from fever Conjunctival chemosis, Decreased vision, Elevated intraocular pressure and
Pain on eye movement. A chalazion is usually a painless swelling on the eyelid that has been present for weeks to months.

188.A case about pathological myopia.

159
Orthopaedics

160
1. Patient is referred to Orthopedics because of abnormally high bone density scan. She has no history of fractures. She takes
analgesics and is waiting for hip replacement surgery. Her dietary calcium and serum biochemistry are normal. What is the
cause of her high density result? (no numbers were provided).
A- Osteoarthritis.
B- Paget’s disease.
C- Osteoporosis.
D- Osteopenia.
Answer: B
Paget disease is a localized disorder of bone remodeling that typically begins with excessive bone
Resorption followed by an increase in bone formation. This osteoclastic overactivity followed by
Compensatory osteoblastic activity leads to a structurally disorganized mosaic of bone (woven bone),
Which is mechanically weaker, larger, less compact, more vascular, and more susceptible to fracture
Than normal adult lamellar bone.
Reference: http://emedicine.medscape.com/article/334607-overview

2. High density bone in DEXA with scheduled total hip replacement:


A - osteoporosis
B - osteomalacia
C - osteoarthritis
D - paget disease

Answer: C (osteoarthritis)
Evidence: Bone density is actually HIGHER rather than LOWER in osteoarthritis.
Low bone density is the telltale sign of osteoporosis, a skeletal disorder
characterized by weakened bones due to excessive loss of bone mass.
Osteoarthritis, on the other hand, is characterized by increased bone density
and bony growths (osteophytes) in conjunction with articular cartilage degeneration
Reference: http://www.orthop.washington.edu/?Q=patient-care/articles/arthritis/osteoarthritis.html

Symptoms of OA: joint pain or tenderness, Stiffness, loss of flexibility, Grating Sensation, & bone Spurs
Tests & Diagnosis: X-ray will show narrowing of the space between the bones in the joint,
Reference: http://www.mayoclinic.org/diseases-conditions/osteoarthritis/basics/tests-diagnosis/con-20014749
Http://emedicine.medscape.com/article/330487-differential
Artefactual causes of raised BMD—no true increase in bone mass:
OA
DISH
AS
Vertebral fractures
Vascular calcification
Thalassaemia major
Abdominal abscesses
Gallstones
Renal calculi
Gluteal silicone implants
Gaucher’s disease
Intestinal barium
Surgical metalwork
Laminectomy
Vertebroplasty and kyphoplasty
http://www.ncbi.nlm.nih.gov/pmc/articles/PMC3651616/

161
3. 37 years old presented with back pain. On examination: there was tenderness when palpating para-spinal muscles,
neurovascular exam was normal. What is the treatment? – (MRI showed mild spinal stenosis)
A. Physiotherapy
B. Bio feedback
C. Cortisone
D. Surgery
Answer: A

4. Patient with fracture humerus what the nerve injury?


A. Axillary
B. Median
C. Ulnar
D. Radial
Answer: A & D
Mid-shaft fractures may damage the radial nerve
Surgical neck fracture may damage axillary nerve

Evidence:
Important tips to remember: Fracture of humerus at:
- Surgical neck: axillary nerve and posterior humeral circumflex artery.
- Mid-shaft: radial nerve (loss of wrist extension) and profunda brachii artery.
- Supracondylar region: median Nerve and brachial artery.
- Medial epicondyle: ulnar nerve.

5. Humerus fracture, how will he present?


A. Waiter’s tip hand
B. Wrist drop
C. Claw hand
Answer:
Mid shaft humerus > radial nerve injury > wrist drop
Surgical neck > axillary nerve
Medial epicondyle > ulnar nerve > claw hand
Supra condylar > median n > ape hand

6. Injury to the surgical neck of humerus cause injury to:


A - Radial A
B - Median
C - Ulnar
D - Brachial plexus (axillary)

Answer: D
Evidence:

162
- This type is proximal humerus injury
- A, B and C go with distal and humeral shaft injury. Assessment: Perform a neurological examination, particularly examining the
axillary nerve Assess and brachial plexus injury through distal neurological examination. Reference: Toronto Notes 2015

The surgical neck of the humerus is a constriction below the tubercles of the greater tubercle and lesser tubercle. It is much more
frequently fractured than the anatomical neck of the humerus. A fracture in this area is most likely to cause damage to the axillary
nerve and posterior circumflex humeral artery. Reference: Wikipedia: http://www.orthobullets.com/trauma/1015/proximal-
humerus-fractures

7. Man was hit by a car, cleared by trauma team, vitally stable with 2 pictures: one showing open fracture of tibia and fibula and
in the other there is bleeding in the shin. How to manage?
A. External fixation
B. Internal fixation
C. Drainage with fixation (didn't mention the type of fixation in this choice)
D. IV antibiotic
Answer: B

8. Football player came to the doctor with pain in the knee after the other player hit him from the side of the knee, valgus test
is positive, what ligament is injured?
A. Fibular (lateral) collateral ligament
B. Tibial (medial) collateral ligament
C. ACL
D. PCL

Answer: B
Medscape: Evocative testing of collateral ligaments:
Valgus stress testing of the MCL: The patient is in the supine position with the knee flexed 25-30 º. The examiner places one hand on
the lateral knee and grasps the medial ankle with the other hand. Then the knee is abducted. Pain and excessive laxity indicate
stretching or tearing of the MCL.

- Mechanism of injury of lateral collateral ligament contact injury, such as a direct blow to the medial side of the knee, or a
noncontact injury, such as a hyperextension stress, may result in a varus force across the knee injuring the LCL
- LATERAL COLLATERAL LIGAMINT TESTED BY VARUS STRESS TEST
- MEDIAL COLLATERAL LIGAMENT TESTED BY VLULGAS STRESS TEST

9. Case of tibial collateral ligament injury: (no stem of Q, missing options, answer not known)
A - Valgus stress testing of the MCL
B - Varus stress testing of the LCL

Recommendations
Injury severity
Medial collateral ligament (MCL)* Lateral collateral ligament (LCL)**
Grade I [< 5 cm - Compression, elevation, & cryotherapy
Treatment similar to that for MCL injuries + hinged
laxity (partial - Short-term use of crutches with weight-bearing–as–
brace for 4-6 weeks
tear)] tolerated (WBAT) ambulation & early ambulation
- short-hinged brace blocking 20% of extension but allows
Grade II (5 - 10 full flexion Treatment similar to that for MCL injuries + hinged
cm laxity) - WBAT with closed-chain exercises to allow for brace for 4-6 weeks
strengthening of knee musculature without putting stress

163
on the ligaments
- Initially non–weight-bearing (NWB) on the affected
- Surgical treatment due to rotational instability
Grade III [> 10 cm lower extremity
(involvement of posterolateral corner of the knee)
laxity (complete - A hinged braced should be used, with gradual
- Bracing & physical therapy for ~ 3 months to
tear)] progression to full weight-bearing (FWB) over 4 weeks
prevent instability
(may require 8-12 weeks to heal)
Answer: ?
Evidence:
* All MCL injuries should be treated with early range of motion (ROM) and strengthening of musculature that stabilizes the knee
joint. Conservative measures usually are adequate, but, if the patient fails to progress with treatment, a meniscal or cruciate
ligament tear is suggested
** heal more slowly  difference in collagen density
Reference: Medscape: http://emedicine.medscape.com/article/307959-treatment#d9

10. Pt with long scenario had trauma to knee. The knee displaced to anterior. (+ anterior drawer test) Which structure is injured?
A - anterior cruciate ligaments
B - posterior cruciate ligaments
C - medial meniscus
D - lateral meniscus

Answer: A
Evidence:
The pivot shift test is performed by extending an ACL-deficient knee, which results in a small amount of anterior translation of the
tibia in relation to the femur. During flexion, the translation reduces, resulting in the "shifting or pivoting" of the tibia into its proper
alignment on the femur. It is performed with the leg extended & the foot in internal rotation, & a valgus stress is applied to the tibia.
Reference: http://emedicine.medscape.com/article/89442-overview

11. Description of anterior cruciate ligament tests?

Answer:
 The knee is flexed at 20–30 degrees with the patient supine. The examiner should place one hand behind the tibia
and the other grasping the patient's thigh. It is important that the examiner's thumb be on the tibial tuberosity. The
tibia is pulled forward to assess the amount of anterior motion of the tibia in comparison to the femur. An intact
ACL should prevent forward translational movement ("firm endpoint")
 Primary function Resists anterolateral displacement of the tibia on the femur
 Secondary function Resists varus displacement at 0 degrees of flexion
Reference: orthobullets

Anterior drawer test: The patient should be supine with the hips flexed to 45 degrees, the knees flexed to 90 degrees and the feet
flat on table. The examiner grasps the proximal lower leg, just below the tibiofemoral joint line, and attempts to translate the lower
leg anteriorly. The test is considered positive if there is a lack of end feel or excessive anterior translation relative to the
contralateral side.

Lachman test: with the knee flexed 20–30°, the tibia is displaced anteriorly relative to the femur; a soft endpoint or greater than 4
mm of displacement is positive (abnormal).

12. Lachman test, if positive it indicates what?


A. ACL
164
B. PCL
C. Medial meniscus
D. Lateral meniscus
Answer: A
The Lachman Test is a commonly used in orthopedic examinations to test for anterior cruciate ligament (ACL) integrity.
Http://physicaltherapyweb.com/lachman-test-orthopedic-examination-knee/

13. Young boy was playing sports with his friends, then presented to hospital with left shoulder pain, the arm is adduction with
internal rotation. This boy has dislocation in which area?
A. Inferior
B. Subglenoid Anterior
C. Subacromial Anterior
D. Supracondylar Posterior

Answer: D
The type of sholder dislocation to present with adduction and internal rotation is posterior, which is less common than anterior
dislocation.
Http://www.wheelessonline.com/ortho/posterior_shoulder_dislocation
14. Military soldier with flat foot. Which of the following will be sustained?
A - Flexor retinaculum
B - Extensor retinaculum
C - Spring ligament
D - Achilles tendon

Answer: C
Evidence:
Medial arch (navicular) injury: Injuries to the navicular bone and related structures jeopardize the structural integrity of the medial
foot arch and thus can lead to severe disability and chronic pain. Therefore, navicular injury is considered high risk, particularly for
runners. Such injuries include navicular stress fractures, tendinopathy of the posterior tibialis tendon insertion onto the navicular,
traumatic separation of an accessory navicular, and partial or complete tears of the attachment of the plantar calcaneonavicular
(spring) ligament.
Reference: http://www.uptodate.com/contents/evaluation-and-diagnosis-of-common-causes-of-foot-pain-in-adults#H13378296
Clinical conditions:
- posterior tibial tendon dysfunction: calcaneonavicular ligament is the most likely ligament to be attenuated in a patient with a
type II flatfoot deformity secondary to posterior tibial tendon dysfunction
- acute spring ligament tear:
= acute trauma can occur with forceful landing on flat foot
= the tibialis posterior tendon is usually normal
Reference: http://www.orthobullets.com/foot-and-ankle/7005/ankle-ligaments

15. Adult with mid-clavicular fracture. X-ray showed overlapped bone fragments. How will you treat?
A - internal fixation
B - external fixation
C - splint
D - figure 8

Answer: A
Evidence:
Nondisplaced midshaft clavicle fractures: arm sling or figure-of-eight dressing (arm sling is better)

165
Displaced mid-shaft fractures: Although non-operative treatment is a viable option to treat displaced mid-shaft fractures, operative
repair should be considered in patients with multiple risk factors for non-union, especially significant fracture displacement or
clavicle shortening. Reference: http://www.aafp.org/afp/2008/0101/p65.html

16. A patient was diagnosed with scoliosis, based on the cobb angle, when to do an orthopaedic referral?
A - > 5 degrees
B - > 10 degrees
C - > 15 degrees
D - > 20 degrees

Answer: B
References: Toronto notes. Http://www.medscape.com/viewarticle/449233_5
Evidence:
The term “Cobb Angle” is used worldwide to measure and quantify the magnitude of spinal
deformities, especially in the case of scoliosis. The Cobb angle measurement is the “gold standard” of
scoliosis evaluation endorsed by Scoliosis Research Society. It is used as the standard measurement to
quantify and track the progression of scoliosis.
Treatment:
• based on Cobb angle
Ƒ < 25°: observe for changes with serial radiographs
Ƒ > 25° or progressive: bracing (many types) that halt/slow curve progression but do NOT reverse deformity
Ƒ > 45°, cosmetically unacceptable or respiratory problems: surgical correction (spinal fusion)
Referral: Any child should be referred immediately when there is suspicion of scoliosis. Radiographs should be left to the specialist to
order. Plain anterior/posterior and lateral views are inadequate and may lead to a missed diagnosis of scoliosis

17. Patient with septic arthritis. The labs showing resistance to the antibiotic. What will you do? (answer not known)
A - stop antibiotic
B - start vancomycin
C - add vancomycin
D - gentamicin

Answer: ? (THIS QUESTION IS NOT COMPLETE, IF THERE IS SURGERY OPTION IT WILL BE THE ANSWER. IF NOT WE SHOULD KNOW
THE CULTURE AND THE RESULTS OF INVESTIGATION) | Reference: http://emedicine.medscape.com/article/236299-medication#2
18. What is the type of elbow joint? Or (Which of these joint is hinge joint?)
A - Ball and socket joint.
B - Saddle joint.
C - Pivot joint.
D - Hinge joint.

Answer: D (elbow is a hinge joint) | Reference: http://www.teachpe.com/anatomy/joints.php

19. What type of joint is the hip joint?


A - Ball and socket joint.
B - Saddle joint.
C - Pivot joint.
D - Hinge joint.

Answer: A
20. Type of intervertebral disc joint?
Answer: secondary cartilaginous, gliding joint.
References:
166
Https://en.wikipedia.org/wiki/Cartilaginous_joint
Http://www.innerbody.com/image_skel07/skel32.html#full-description

21. Osteoporosis, how to prevent spinal compression fx?


A - Daily vit D
B - Aerobic exercise
C - Weight bearing
D - Walking
Answer: A
Evidence: Vitamin D is used for prevention and treatment, and prevents total number of fractures
Reference: http://www.uptodate.com/contents/osteoporotic-thoracolumbar-vertebral-compression-fractures-clinical-
manifestations-and-
treatment?Source=search_result&search=osteoporotic+vertebral+compression+fracture&selectedtitle=1~150#H1565673

22. Occipital headache and neck pain can't do lateral flexion weakness of deltoid bilaterally on x-ray
A. osteophyte
B. neck strain
C. cervical spondylitis
D. brachial plexus neuropathy

23. Renal or small cell cancer stage III with bone pain what is the immediate action?
A. MRI only
B. Radiotherapy
C. IV steroid and MRI
D. No immediate action

Answer: A?
Not sure about the answer (whether immediate or not) but mostly it is

24. Man has a RTA. He was conscious, oriented, alert, had pain in all of his 4 extremities, what’s the type of shock?
A - irreversible
B - neurogenic
C - cardiogenic
D - haemorrhagic

Answer: B

25. Pt. Presented with deformity in the hand, xray showed fracture in the radius, they put picture of
xray and the hand. How will you fix it?
A - Close reduction ... Below elbow
B - Closed reduction. ... Above elbow
C - Open reduction ... Below elbow
D - Open reduction above elbow

Answer: A (Q not clear; it says fracture in the radius but didn’t specify radial head, shaft or distal)
Evidence:
For the distal radius fracture the treatment depend:
- if extra-articular and/or <5mm shortening we will do closed reduction and cast
- if its intra-articular we can do either fixation (ORIF) and cast
- Most distal radial fractures are casted with below elbow

167
References:
https://www2.aofoundation.org/wps/portal/surgery?Showpage=redfix&bone=Radius&segment=Distal&classification=23-
A1.1&treatment=&method=Cast&implantstype=&approach=&redfix_url=1285238880882&Language=en
- http://www.orthobullets.com/trauma/1027/distal-radius-fractures#4604
- http://www.orthobullets.com/trauma/1027/distal-radius-fractures

26. Woman complaining of left hand tingling mainly at thumb and index on exam there was mild atrophy of thenar muscle
Tinel’s test was positive which nerve may be affected?
A - Radial nerve.
B - Median nerve.
C - Musculocutaneous nerve.
D - Ulnar nerve.

Answer: B
27. What is the most accurate test for carpal tunnel syndrome:
A - Tinel
B - Compression test
C - Durkan's carpal test
D - Phallens test

Answer: B (Sensitivity 89%, specificity 96%)


References: uptodate & Medscape: http://www.uptodate.com/contents/carpal-tunnel-syndrome-clinical-manifestations-and-
diagnosis?Source=outline_link&view=text&anchor=H5#H5

Answer: B | Reference: http://www.medicinenet.com/script/main/mobileart.asp?Articlekey=16687

28. Patient with numbness of index finger when he uses scissors, what is the diagnosis:
A - Osteoarthritis.
B - Ducyptus.
C - Carpal Tunnel Syndrome.
D - Cubital Tunnel Syndrome.

Answer: C

29. Carpal tunnel syndrome. Pt can't work and write which muscles affected:
A. Thenar eminence
B. Interossei palmar muscle
C. Interossei dorsal muscle

Answer: A Compression of the median nerve as it runs deep to the transverse carpal ligament (TCL) causes atrophy of the thenar
eminence, weakness of the flexor pollicis brevis, opponens pollicis, abductor pollicis brevis

30. A patient is complaining of pain and numbness in his left thumb and index with loss of sensation over the dorsum 1/3 of left
hand. What will you do to confirm the diagnosis?
A. Nerve conduction studies
B. CT
C. MRI
Answer: A
The patient’s symptoms and signs match the distribution of median nerve, so carpal tunnel syndrome (CTS) is the main differential
diagnosis. Electromyography (EMG) and nerve conduction studies are the first line investigation in suspected CTS. An MRI is useful
preoperatively if there is any space occupying lesion. But before doing an MRI, ultrasound should be done.
168
Reference http://emedicine.medscape.com/article/327330-workup#c7

31. (Picture) 4 y/o boy with distal ulnar & redial bone fracture the skin was slightly injured by the bone what is the management?
A - closed reduction with cast below elbow
B - closed reduction with cast above the elbow
C - open reduction with plaid
D - Aggressive debridement and irrigation and Fracture stabilization

Answer: D| Reference: http://www.orthobullets.com/trauma/1004/open-fractures-management


32. Falls onto an outstretched hand leads to:
A - Clavicle fracture.
B - Colles fracture.
C - Shouder Dislocation.
D - Mith fracture.

Answer: B
 Colle’s: Extra-articular transverse distal radius fracture (~2 cm proximal to the radiocarpal joint) with dorsal displacement ±
ulnar styloid fracture.
 Smith’s: Volar displacement of the distal radius (i.e. Reverse Colle’s fracture)

33. A girl fell on outstretched hand. X-ray shows open fracture of radius and ulna. What is the treatment?
A. Internal fixation
B. External fixation
C. Drainage and fixation (didn’t mention type of fixation in this choice)
D. Antibiotic
Answer: A

34. Pt fall on outstretched hand, & he can't flex the distal part of his second finger? Which muscle is injured?
A - Flexor pollicis longus.
B - Flexor digitorum superficialis.
C - Flexor digitorum profundus.
D - Extensor carpi radialis longus.

Answer: C (flexor digitorum profundus for the proximal & distal joints, & flexor digitorum superficialis for the proximal joints)
Reference: Wikipedia

35. A 13 years old boy with right hip pain for 3 months, x-ray shows degenerative
collapse of femoral neck, WBC = 17,
ESR is normal, ALP is normal:
A. Perthe diseases
B. Hip arthritis
C. Tumor
D. Developmental Dysplasia of the HIP (DDH)

Answer: A (Also known as Legg–Calvé–Perthes disease (LCPD) or Coxa Plana and it is


a paediatric disease causing an idiopathic avascular necrosis of the femoral head.
Features: Antalgic gait ± pain, hip pain. Diagnostic features on X-Ray is a collapsed
femoral head) | Reference: Kaplan surgery

169
36. Pt after RTA, no abduction and lateral rotation of the arm. What is the origin of the affected nerve?
A - Medial plexus
B - Lateral plexus
C - Lower plexus
D - Root

Answer: D (Axillary nerve palsy, but the answers are not clear)
Evidence: Erb’s Palsy or Upper plexus or C5 root injury: Shoulder abduction, extension and external rotation are lost.
Reference: http://www.physio-pedia.com/Brachial_plexus_injury#Function

37. Old Patient with back pain. Spine DEXA results is provided below:
(Cervical = 0.05 ; Thoracic = < - 1.5 ; Lumbar = < - 2.8 ; Sacral = < - 1.3). What is the diagnosis?
A - Osteopenia
B - Severe osteopenia
C - Osteoporosis
D - Established osteoporosis

Answer: C (> - 1 is normal, - 1 to - 2.5 is osteopenia, < or equal to - 2.5 is osteoporosis)


Evidence: Bone mineral density T-score criteria for osteoporosis and low bone mass
Diagnosis T-score
Normal ≥-1.0
Low bone mass (osteopenia) Between -1.0 and -2.5
Osteoporosis ≤-2.5
Severe (established) osteoporosis ≤-2.5 and fragility fracture
Reference: uptodate:
http://www.uptodate.com/contents/image?Imagekey=ENDO%2F53999&topickey=ENDO%2F2056&source=see_link

38. Which of the following is most common non traumatic fracture in osteoporosis?
A - Vertebral fracture.
B - Statin induced myopathy in old lady.
C - Colles fracture.
D - Ulnar fracture.

Answer: A (Vertebral fracture is the most common clinical manifestation of osteoporosis) | Reference: uptodate

39. Old male not known to have any medical illness presented with right knee swelling, no fever or tenderness, what
investigation will you order?
A - Arthrocentesis.
B - Knee MRI.
C - X-Ray.
D - CT.

Answer: C | Reference: approach to non-traumatic knee effusion: http://www.aafp.org/afp/2000/0415/p2391.html

40. Which part of body bone is forming the ankle?


A - Calcanues.
B - Cuboid.
C - Talus.
D - Cuneiforms.

170
Answer: C (Ankle joint: the fibula, the tibia and the talus)

The true ankle joint is composed of three bones, anterior view: the tibia which forms the inside, or medial portion of the ankle; the
fibula which forms the lateral or outside portion of the ankle and the talus underneath.
The true ankle joint is responsible for the up-and-down motion of the foot.

41. Examiner asked a patient to stand on his toes, what is the nerve he examined?
A - Femoral nerve.
B - Deep peroneal nerve.
C - Tibial nerve.
D - Superficial peroneal nerve.

Answer: C (Toe Dorsifelxion by Deep peroneal nerve ; Toe Plantar flexion by Tibial nerve)
Reference: http://www.orthobullets.com/spine/2002/lower-extremity-spine-and-neuro-exam

42. A patient can't do dorsiflexion & eversion: (Foot drop) + loss of sensory of first web space between 1st and second toes
A - Femoral nerve.
B - Deep peroneal nerve.
C - Tibial nerve.
D - Superficial peroneal nerve.

Answer: B

Explanation: deep peroneal nerve is responsible for dorsiflexion of the foot and sensation of the first web space. Injury can cause
foot drop.
Reference: http://teachmeanatomy.info/lower-limb/nerves/deep-fibular/

- common peroneal causes foot drop and its course is close to the tibia plateau posteriorly at the lateral side (tibia plateau = head)
- if the question stated that there is numbness in the dorsum of the foot, then it is the deep peroneal

43. Patient with 1st metatarsal joint pain, red, tender, hot joint? What is the cause?
A - Staphylococcus aureus.
B - Monosodium urate crystal.
C - Ca pyrophosphate crystal.
D - Haemarthrosis.

Answer: B
- Gout is caused by monosodium urate monohydrate crystals.
- Pseudogout is caused by calcium pyrophosphate crystals and is more accurately termed calcium pyrophosphate disease.

44. Bilateral shoulder and hip stiffness and pain. What is the diagnosis:
A - Polymyalgia rheumatica.
B - OA.
C - Fibromyalgia.
D - Frozen shoulder.

Answer: A
45. Loss of adduction of fingers caused by injury to:
A - Ulnar nerve.
B - Median nerve.
C - Radial nerve.

171
D - Axillary nerve.

Answer: A

46. Patient with Osteoarthritis which type of exercise is the best:


A - High repetition and high resistance.
B - Low repetition and high resistance.
C - High repetition and low resistance.
D - Low repetition and low resistance.

Answer: D

47. Limping child, painful left hip (x-ray given)?


A - Slipped Capital Femoral Epiphysis.
B - Perthe's disease.
C - Developmental Dysplasia of the Hip (DDH).
D - Osgood-Schaltter Disease.

Answer: A
Evidence:
Klein's Line On AP view, line drawn along supero-lateral border of femoral neck
should cross at least a portion of the femoral epiphysis. If it does not, suspect SCFE

48. What is the artery supplying the gluteal region?


A - Internal pudendal artery.
B - Internal iliac artery.
C - External iliac artery.
D - Femoral artery.

Answer: B

49. An elderly patient came with bilateral decreased range of motion of knee joint. There was surrounding muscle wasting.
Examination shows crepitus. What is the Diagnosis:
A - Osteoarthritis
B - Rheumatoid arthritis.
C - Septic Arthritis.
D - Patella dislocation.

Answer: A
50. Mother and father were holding their son from his hand and they elevate them over the ground, then the child complain of
pain in his elbow he was flexing his arm with pronation he didn't let anybody touch it. What was the injured ligament?
A - Quadrant ligament.
B - Annular ligament.

172
C - Sheet between radius and ulnar bone.
D - Medial collateral ligament.

Answer: B | Reference: Pulled elbow: http://orthoinfo.aaos.org/topic.cfm?Topic=A00717

51. Patient complaining of hip pain after long periods of using the hip it keeps him awake at night and have prolonged hours of
stiffness in the morning:
A - Osteoporosis.
B - Osteoarthritis.
C - Hip fracture.
D - Muscle strain.
Answer: B

52. Knee aspiration in young boy showing labs result: WBC > 75, >25 neutrophils. Typical scenario. What is the treatment?
A - Oral antibiotic.
B - IV antibiotic
C - Surgical drainage with IV antibiotic.
D - Rest & pain killer if needed.

Answer: C
Evidence:
Medical management of infective arthritis focuses on adequate and timely drainage of the infected synovial fluid, administration of
appropriate antimicrobial therapy, and immobilization of the joint to control pain.

53. Shoulder dislocation, was put into place then loss of sensation over lateral arm occurred. What’s the nerve affected?
A - Axillary nerve.
B - Radial nerve.
C - Median nerve.
D - Ulnar nerve.

Answer: A

54. Pain in the snuff box would most likely to be due to fracture of (other version: What is snuff box fracture?)?
A - Scaphoid.
B - Head of hamate fracture.
C - Colle’s fracture.
D - 1st metacarpal fracture.
Answer: A

55. What is the most commonly injured Carpal bone? (missing options)
A - Scaphoid.

Answer: A
Evidence:
Scaphoid (palpated in anatomic snuff box) is the most commonly fractured carpal bone and is prone to avascular necrosis owing to
retrograde blood supply.
Dislocation of lunate may cause acute carpal tunnel syndrome. A fall on an outstretched hand that damages the hook of the hamate
can cause ulnar nerve injury.

173
Reference: FA USMLE step 1

56. Post-menopausal with fractures and osteoporosis, what is the best drug?
A - Calcitonin.
B - Calcitriol.
C - Alendronate.
D - Estrogen.

Answer: C (Alendronate is the first line for postmenopausal osteoporosis)

57. Old man on steroids and osteoporosis, what to give:


A - Calcitonin.
B - Calcitriol.
C - Alendronate.
D - Raloxifene.

Answer: C

58. Old lady postmenopausal with osteoarthritis and risk for osteoporosis, what will you do:
A - Calcium, TSH, dihydroxy vit D.
B - Bisohosphonate, vit D, calcium.
C - DEXA scan.
D - Estrogen level.

Answer: C
Thyroid-stimulating hormone level: Thyroid dysfunction has been associated with osteoporosis.
Http://emedicine.medscape.com/article/330598-overview

59. Child with radial and ulnar fracture and 1cm laceration in the volar aspect of the hand, what is most likely damaged nerve?
A - Radial nerve.
B - Median nerve.
C - Ulnar nerve.
D - Axillary nerve.

Answer: C

60. An elderly woman who has chronic back pain which increases while walking uphill and decreases while walking down hill,
what is the most likely diagnosis?
A - Claudication.
B - Lumbar Strain.
C - Lumbar Spine Stenosis.
D - Pott Disease.

Answer: B (lumbar spine stenosis)

174
Reference: https://www.laserspineinstitute.com/back_problems/neurogenic/symptoms/

61. In which condition bone age is greater than chronological age?


A - Constitutional growth delay.
B - Growth hormone deficiency.
C - Precocious puberty.
D - Hypothyroidism.

Answer: C
Evidence:
An advanced bone age is common when a child has had prolonged elevation of sex steroid levels, as in precocious puberty or
congenital adrenal hyperplasia

62. Female with leg pain when she walks 300 m. Relief by rest.
A - Claudication.
B - DVT.
C - Venous insufficiency.
D - Ligament rupture.

Answer: A
Read more about Vascular Claudication Versus Spinal Stenosis.

63. 10 years old child complaining of pain in tibial tubercle he has just had growth spurt. What is the cause of the pain:
A - Osgood schlatter disease.
B - Legg-perthes-canal disease.
C - Stress fracture.
D - Tibial splint.

Answer: A (partial avulsions or microavulsions of chondrofibro-osseous tibial tubercle)

64. Pt with tibiofibular fracture, plaster of paris applied, came back with extreme pain, what’s the sign to your dx:
(What is the first sign of compartment syndrome of the leg?)
A. Loss of pulse
B. Loss of sensation

175
C. Loss of movement
D. Pain out of proportion
Answer: D
Patients with compartment syndrome typically present with pain whose severity appears out of proportion to the injury. The pain is
often described as burning. The pain is also deep and aching in nature and is worsened by passive stretching of the involved muscles.
The patient may describe a tense feeling in the extremity.
(http://emedicine.medscape.com/article/307668-clinical)

CLINICAL FEATURES — The signs and symptoms of acute compartment syndrome (ACS) generally appear in a stepwise fashion,
although the timing of the appearance of specific findings varies. Important clues to the development of ACS include rapid
progression of symptoms and signs over a few hours and the presence of multiple findings consistent with the diagnosis in a patient
at risk. Therefore, serial evaluation is of great importance in patients at risk for ACS. However, the limitations of the physical
examination for identifying ACS must be emphasized; any tense painful muscle compartment represents a possible ACS.

Symptoms of ACS can include the following:

 Pain out of proportion to apparent injury (early and common finding)


 Persistent deep ache or burning pain
 Paresthesias (onset within approximately 30 minutes to two hours of ACS; suggests ischemic nerve dysfunction)

Examination findings suggestive of ACS include the following:

 Pain with passive stretch of muscles in the affected compartment (early finding)
 Tense compartment with a firm "wood-like" feeling
 Pallor from vascular insufficiency (uncommon)
 Diminished sensation
 Muscle weakness (onset within approximately two to four hours of ACS)
 Paralysis (late finding)

65. Non athlete heard a click at the left leg calf when he Forcefully plantar flex his left foot while the leg is fully extended. On
examination, he can't plantar flex his foot against resistance & can't walk by toe-foot walking of the affected site. What is the
injured structure?

A. Calcaneal ligament
B. Quadriceps ligament
C. Plantar ligament
D. Femoris rectus
E. Achilles tendon rupture

Answer: E?
Http://www.orthobullets.com/foot-and-ankle/7021/achilles-tendon-rupture
66. Superficial palmar arch artery of:
A. Ulnar
B. Radial "deep arch"
C. Anterior
D. Posterior
Amswer: A
Https://en.wikipedia.org/wiki/Superficial_palmar_arch

176
67. Pt present with pain of inferior heal which getting worse at morning & improve at the end of the day.
A. Plantar fasciitis
B. Calcaneal spur
C. Calcaneal fracture
D. Achilles tendinitis

Answer: A
Plantar fasciitis is the most common cause of pain on the bottom of the heel.
Although many people with plantar fasciitis have heel spurs, spurs are not the cause of plantar fasciitis pain. One out of 10 people
has heel spurs, but only 1 out of 20 people (5%) with heel spurs has foot pain. Because the spur is not the cause of plantar
fasciitis, the pain can be treated without removing the spur
Http://orthoinfo.aaos.org/topic.cfm?Topic=a00149

68. Pt with open fracture in left arm. After open reduction & internal fixation pt suddenly cant extend his forearm, wrist or hand.
Which of the following nerve affected?
A. Radial nerve in spiral groove
B. Median nerve in cubital fossa
C. Ulnar nerve on medial epicondyle
D. Median nerve in lateral epicondyle
Answer: A
- Ulnar N passed post to medial epicondyle if injured >> claw hand and sensory lost over medial aspect of the hand.
- Radial N passed anteriorly to the lateral epicondyle within the cubital fossa of the forearm if injured >> depend on the site (
if in the anterior spiral groove >> wrist drop )
Http://www.orthobullets.com/pediatrics/4008/medial-epicondylar-fractures--pediatric

The radial nerve provides motor innervation to the extensors of the forearm, wrist and fingers.
Reference: http://emedicine.medscape.com/article/1244110-clinical
Kaplan USMLE step 1 – Anatomy

Radial nerve innervates all muscles of the posterior compartments of arm and forearm (extensors). Primary action: extend MP joints,
wrist and elbow.
Radial n. Injury: mid-shaft of humerus at radial groove or lateral elbow:
 Loss of forearm extensors of the wrist and MP joints (wrist drop)
 Weakened supination
 Sensory loss on posterior forearm and dorsum of thumb

69. What is the first thing in management of depressed fracture?


A. Discharge the patient
B. Antibiotics
C. IV fluid
D. Lateral CT scan
Answer: A
Treatment of skull fractures is primarily conservative , consists of observation to rule out any ongoing complications such as CSF
leak, seizure, or infection. Medical interventions such as anticonvulsant and antibiotic prophylaxis are not routinely given to
patients with isolated skull fractures.
Http://bestpractice.bmj.com/best-practice/monograph/398/treatment/details.html

177
70. Patient is a golf player came medial epicondylitis what to do?
A. Refer to ortho.
B. Stop playing golf.
C. Recommend surgery for the patient
D. X-ray
Answer: B
Toronto:
 Rest, ice, nsaids
 Use brace/strap.
 Physiotherapy, stretching, and strengthening.
 Corticosteroid injection
 Surgery: percutaneous or open release of common tendon from epicondyle (only after 6-12 mo of conservative therapy)

71. Patient sustain a trauma to the jugular foramen & hypoglossal canal, which function will be preserved ;

A. Raising the shoulders


B. Thoracic sensation
C. Vocal cords movement
D. Taste sensation of the tongue (i forget which part ; anterior or posterior )
Answer:
•Hypoglossal canal: Hypoglossal (CN XII) Inspect tongue for signs of lateral deviation, atrophy, fasciculations, asymmetry of
movement and strength
•Jugular foramen: IX(glossopharyngeal),X(vagus)(these two control autonomic nervous system+ vocal cords) ,XI(accessory)(control
some shoulder muscles).

72. Women wearing high-heels one of them was broken and her leg goes outward what is the ligament affected
A. Sprain
B. Cacinofibuler
C. Deltoid
D. Anterior Talofibular ligament
Answer: C
Medial ligament Complex (deltoid ligament) is associated with eversion injuries and the lateral ligament complex (Anterior
Talofibular, Calcaneo fibular, Posterior Talofibular) are associated with inversion injuries.

Reference: Toronto notes

[The medial, or deltoid, ligament is stretched in an Eversion (medial) ankle sprain.]


Evidence: "deltoid ligament injury occurs with pronation (eversion) trauma leading to forced external rotation & abduction of ankle"
Reference: http://www.orthobullets.com/foot-and-ankle/7005/ankle-ligaments

73. Girl with high heels, RT heel breaks with inversion of RT foot, what’s the tendon injured (other versions: A lady wearing high
heels fell & had ankle sprain, which tendon is affected? ; Twisted ankle what is the most common ligament?)? (missing
options)
A - calculotibialis
B - anterior tibialis

Answer: Anterior talofibular ligament is stretched in an Inversion (lateral) ankle sprain


Evidence:

178
- The most common injury is ankle sprain rather in supination most common or in pronation less common. There is no tendon
rupture or tear in this injury. There is ligament injury and are lateral ligaments complex...
ATFL = Anterior talofibular lig (most common), PTFL=posterior talofibular lig, CFL=calcanofibular lig
- The most common type of ankle sprain occurs when the foot is inverted too much, affecting the lateral side of the foot. When this
type of ankle sprain happens, the outer, or lateral, ligaments are stretched too much. The anterior talofibular ligament is one of the
most commonly involved ligaments in this type of sprain. Approximately 70-85% of ankle sprains are inversion injuries.
Reference: http://www.orthobullets.com/foot-and-ankle/7005/ankle-ligaments ;
http://www.iaaf.org/download/download?Filename=5b2291a7-258c-4635-bbc0-
8657eec73740.pdf&urlslug=Chapter%2010%3A%20Specific%20injuries%20by%20anatomic%20site

74. Flat shoulder 1/3 middle arm sensation lost:


A. Ape palsy
B. Complete claw
C. Partial claw
D. Wait until 12 w
Answer: most likely C
The question has missing details, but in general, cervical cerclage is done at 12 to 14 weeks for women with a history-based
diagnosis of cervical insufficiency.
Reference: http://www.uptodate.com/contents/cervical-insufficiency

75. If you take blood sample from thigh how to prevent sciatica nerve?
A. Upper medaila
B. Lower medial
C. Upper lateral
D. Lower lateral

Http://www.medscape.com/viewarticle/551320_3

76. Unilateal knee pain , swelling , middle age i think Joint apsirate
Wbc : nl Neut : 80% Did not mention crystals :
A. RA
B. Gout
C. Septic arthritis
D. Psudogout ??

77. Patient have parotitis, pain with eating that radiate to the ear, with nerve transmit pain with eating?
A. 8
B. 9
C. 10
179
D. 7

Answer: D, http://teachmeanatomy.info/head/organs/salivary-glands/parotid/

78. Lesion in forehead, what LN will be enlarged?


A. Mastoid
B. Auriculomandibular (should be pre-auricular or submandibular)
C. Superficial cervical
D. Parotid

Answer: should be submandibular


In general, the face, scalp, and ear –> drains into occipital, retroauricular, parotid, buccal, submandibular, submental, superficial
cervical l.n.
Http://images.slideplayer.com/16/5126607/slides/slide_11.jpg
http://images.slideplayer.com/12/3440402/slides/slide_9.jpg
http://teachmeanatomy.info/neck/vessels/lymphatics/

79. Patient come with back pain when awake from sleep he had stiffness for almost 30 minutes and it's resolved he take i think
paracetamol or other thing in examination there is a para spinal muscle stiffness When you did a spine CT you found mild
lumbar stenosis what your management:
A. epidural steroid injection
B. physiotherapy
C. ibuprofen
D. bed rest

Answer:

80. Hyperextension injury developed pain in distal phalanx, tender palms, cant flex distal phalanx:
A. Rupture superfascialis tendon
B. Rupture flexor digitorum profundus tendon
C. Extra articular fracture of distal phalanx
D. Intra articular fracture of middle phalanx
Answer: b. Rupture flexor digitorum profundus tendon. American Family Physician
Http://www.aafp.org/afp/2006/0301/p810.html & http://www.aafp.org/afp/2001/0515/p1961.html

81. A female patient injured her left index. Now she is unable to flex the distal part of the finger with an associated swelling.
What is the structure most likely injured?
A. Fracture to the distal phalanx bone
B. Fracture to the proximal phalanx bone
C. Torn flexor digitorum profundus (FDP)
D. Torn flexor digitorum superfisialis (FDS)
Answer: C
FDP is responsible for flexing the distal part of the finger while the FDS is responsible for flexing the fingers at the middle.
The Hand Examination and Diagnosis by American Society for Surgery of the Hand

82. Someone had a dislocation of mandible and the doctor tried to retract the mandible inferiorly. Which muscle retract the
muscle to its normal position
180
A. Medial ptyrgoid
B. Lateral ptyrgoid
C. Masters
D. Temporlais
Answer: D
Reference: http://teachmeanatomy.info/head/muscles/mastication/

83. Picture of tibial fracture proximal peripheral perfusion normal and normal sensation the best next to investigate?
A - Angiography
B - MRI
C - Doppler US
D - X-Ray

Answer: D
Evidence:
The standard protocol is to obtain anteroposterior and lateral radiographs of the injured leg. CT scans and MRI are reserved for
more complex injuries. MRI has particular value in higher tibial fractures that may extend into the knee joint or are suspected of
involving the tibial plateau. MRI also helps delineate associated meniscal or ligamentous injuries.

84. ‫ صورة ساق مكسوره من تحت الركبه النبض موجود‬duplex is .83


What to do?
A. Angiograph
B. Venography
C. Duplex US
D. MRI
Answer: A
Angiography remains the gold-standard investigation for the further investigation and delineation of vascular injury. In most
traumatic injury settings, angiography is best performed in the operating room, with the surgeon exposing the vessel proximal to the
injury for control and expediency. Transfer to the radiology suite should be restricted to haemodynamically stable patients with
proximal or torso injuries. Angiography may be used to treat certain selected injuries, and where expertise and technical facilities
are available. Proximal control may be possible with an angioplasty catheter prior to transfer to the operating room.

85. Pt with trauma, femoral/hip fracture. Lt leg short and laterally rotated. What muscle is responsible for lateral rotation:
A- gluteus maximus
B- rectus femorus
C- gracilus
D- adductor Magnus
Answer: A

86. Pt fall on his elbow what will you see in lateral x ray (no other info)
A) Radial line anterior to acetabulum (
B) Some line bisect with acetabulum
C) Anterior fat
D) Posterior fat
Answer: D
Supracondylar Fracture: extension type most common (95-98%), occur most commonly in children aged 5 to 7, mechanism of injury:
fall on outstretched hand, posterior fat pad sign:
Lucency along the posterior distal humerus and olecranon fossa is highly suggestive of occult fracture around the elbow
Http://www.orthobullets.com/pediatrics/4007/supracondylar-fracture--pediatric

181
87. An osteochondroma patients developed a mass (or pain) in his pelvis. Histology shows cartilaginous tumor…..what is the
most likely diagnosis?
A. Ewing’s sarcoma
B. Chondroblastoma disease
C. Chondrosarcoma
Answer: C
Explanation: osteochondroma (exostosis) is the most common benign tumor. It can –rarely- transform to chondrosarcoma which is a
malignant cartilaginous tumor that commonly occurs in the spine or pelvis. Ewing’s sarcoma histology show anaplastic blue cells.
Reference First Aid USMLE step 1

Chondrosarcoma is a cancer of cartilage cells. It is the second most common primary bone cancer. This cancer is rare in people
younger than 20. After age 20, the risk of getting a chondrosarcoma goes up until about age 75. Women get this cancer as often as
men. Chondrosarcomas can develop anywhere there is cartilage such as the pelvis, leg bone or arm bone. Occasionally,
chondrosarcoma will develop in the trachea, larynx, and chest wall. Other sites are the scapula (shoulder blade), ribs, or skull.
Osteosarcoma: is the most common primary bone cancer. This cancer starts in the bone cells. It most often occurs in young people
between the ages of 10 and 30, but about 10% of osteosarcoma cases develop in people in their 60s and 70s. It is rare in middle-
aged people, and is more common in males than females. These tumors develop most often in bones of the arms, legs, or pelvis.
Ewing tumor: is the third most common primary bone cancer, and the second most common in children, adolescents, and young
adults. The most common sites for this cancer are the pelvis, the chest wall (such as the ribs or shoulder blades), and the long bones
of the legs or arms.
Link: http://www.cancer.org/cancer/bonecancer/detailedguide/bone-cancer-what-is-bone-cancer

88. Patient with history of trauma to the shoulder. He is unable to raise his hand above his head. (There’s an accompanying MRI
picture of the shoulder with an arrow pointing at a small white spot within the joint on the top part)

A. Acromioclavicular junction
B. Shoulder dislocation
C. Rotator cuff muscle tear

Answer: C

 Acute trauma to the shoulder suggests joint dislocation, fracture, or tendon tear.
 Rotator cuff tendonitis and other inflammatory/degenerative joint disease are the results of chronic repetitive overhead
activity.
 Conventional shoulder MRI is a T2-weighted image most of the time. Hyper-intensity on T2-weighted shoulder MRI can
mean: tumors, tendinitis, tendon tears, post-traumatic edema and inflammation.
 The MRI image described in this question is probably a partial thickness rotator cuff tendon tear.
Source: http://emedicine.medscape.com/article/401714-overview

89. Muscle responsible of unlocking the knee?


A. Gastrocnemius
B. Fibularis
C. Popliteus muscle
Answer: C

Reference:
Https://books.google.com.sa/books?Id=0h7ftcb02euc&pg=PA412&lpg=PA412&dq=popliteus+muscle+unlock&source=bl&ots=m0sw
f6rnme&sig=-g9Jv-
182
cwfhslndy9vjmrc8micow&hl=ar&sa=X&ved=0ahukewj08_2Py8zJAhUD_hikhyq2dei4kbdoaqgcmae#v=onepage&q=popliteus%20mus
cle%20unlock&f=false

90. Muscle which make the knee flexion from upright position is?
Https://en.wikipedia.org/wiki/Knee

- The popliteus muscle at the back of the leg unlocks the knee by rotating the femur on the tibia, allowing flexion of the knee.
- The quadriceps femorus muscle group (rectus femoris, vastus lateralis, vastus medius, and vastus intermedius) crosses the knee via
the patella and acts to extend the leg.
- The hamstrings (semitendinosus, semimembranosus, and biceps femoris) flex the knee and extend the hip (except for the short
head of the biceps femoris).

Source: Boundless. “Muscles that Cause Movement at the Knee Joint.” Boundless Anatomy and Physiology. Boundless, 26 May.
2016. Retrieved 30 May. 2016 from https://www.boundless.com/physiology/textbooks/boundless-anatomy-and-physiology-
textbook/muscular-system-10/muscles-of-the-lower-limb-107/muscles-that-cause-movement-at-the-knee-joint-579-9335/

91. What is the type of fracture that has several segments in the diaphysis?
A - compound
B - impacted
C - comminuted

Answer: C | Reference: The pathophysiology of fractures Textbook + picture 

92. A patient with Tibial fracture with abnormal ABI. What will you do next?
A - MRI
B - CT
C - Angiography

Answer: C
Evidence:
- Tibial fracture with ABI/PVR abnormal, Needs vascular team involvement and urgent vascular imaging (CT angiography)
Reference: Tibial Plateau Fractures – Initial Management Guidelines:
http://www.medicine.virginia.edu/clinical/departments/orthopaedics/orthopaedic-education/residency%20programs/treatment-
protocols/Tibial_Plateau_Fractures_2013_Initial_Management_Guidelines.pdf

93. Old lady complaining of right femur pain when she goes upstairs and downstairs. Examination of the right hip: flexion,
extension and rotation are normal but abduction caused pain. What is the best investigation to do?
A - Radiological assessment.
B - Blood testing.
C - Rheumatoid factor.

Answer: A
Evidence:
(Pain while abduction of Rt hip) Most probably to be posterior hip dislocation because all movements are intact (flexion internal
rotation...) Except abduction so we need radiological views to confirm the diagnosis
Reference: Netter’s concise orthopaedic anatomy

183
94. Football player he can't raise his big toe. What muscle affected:
A - Deep muscle under foot
B - Superficial muscle under foot
C - Tibia

Answer: A
Evidence:
(Pain while raising up the big toe) Deep muscles under foot responsible for dorsiflexion of big toe most probably the diagnosis will be
turf toe, which is common in football players (it sprains to the ligaments around the big toe)
Extensor hallucis longus muscle is responsible for Dorsiflexion of the great toe
Reference: Miller’s review of orthopaedic

95. Child with fracture of forearm (x-ray shows open fracture of radius & ulna), what is the treatment? (answer not known)
A - Closed reduction
B - Open reduction
C - Surgical intervention

Answer: A
O.R.I.F with plates and screws; closed reduction with immobilization usually yields poor results for displaced forearm fractures
(EXCEPT in children) | Reference: Toronto Notes 2015
open reduction internal fixation is treatment of choice in fracture radius and ulna together
References: Prof. Mohammed Jalaat Alfaisal & http://emedicine.medscape.com/article/1239187-treatment#d10

96. Fracture of distal ulna and radius with a pic what's the immediate management
A. Debridement, irrigation and fixation
B. Close reduction and cast above the elbow
C. Close reduction and cast below the elbow
Answer: B.
Close reduction and cast above the elbow. The majority of distal radial fractures is a below elbow cast. If the ulna is involved, a sugar
tong or an above elbow cast may be considered, in order to control forearm rotation. To limit forearm pronation and supination and
provide stabilization of the fracture and allow ossification. Http://www.aafp.org/afp/2009/0901/p491.html &aofoundation

97. Blood supply of post. Leg compartment:


A - tibial
B - Common fibula
C - Superfacial fibula

Answer: A (posterior tibial artery)


Evidence:
- The posterior tibial artery is the larger of the terminal branches of the popliteal artery. It descends deep to soleus, where it can be
exposed by splitting gastrocnemius and soleus in the midline, then becomes superficial in the lower third of the leg and passes
behind the medial malleolus between the tendons of flexor digitorum longus & flexor hallucis longus.
- posterior compartment of leg has two parts (superficial and deep):
= Superficial contains: Gastrocnemius, soleus, plantaris
= Deep contains: popliteus, flexor hallucis longus, flexor digitorum longus and tibialis posterior
Reference: Review of orthopedics by Miller + Gray's anatomy

98. What is the most specific sign for osteoporosis?


A - ↑ bone density

184
B - subchondral cyst
C - compression fracture

Answer: C (compression fracture)


Evidence:
Fractures are the most dangerous aspect of osteoporosis. Debilitating acute and chronic pain in the elderly is often attributed to
fractures from osteoporosis and can lead to further disability and early mortality. These fractures may also be asymptomatic. The
most common osteoporotic fractures are of the wrist, spine, shoulder and hip. The symptoms of a vertebral collapse ("compression
fracture") are sudden back pain. Reference: https://en.wikipedia.org/wiki/Osteoporosis

99. Fracture of the distal one-third of the humeral shaft commonly associated with which type of radial nerve injury?
A - neuropraxia
B - neurotemesis
C – axontemesis

Answer: A (Neuropraxia)
Evidence: Neuropraxia is the answer if closed fracture, while neurotemesis is the answer if open fracture + radial nerve palsy
Reference: http://www.orthobullets.com/trauma/1016/humeral-shaft-fractures

100.Patient complained of progressive wrist pain since 2 months and increased in the past 1 month, give history of caesarean
delivery. On exam there is numbness and normal range of motion. What is the Rx?
A - Thumb cast
B - Whole are cast
C - Surgical decompression

Answer: A (Thumb cast)


Evidence:
- looks like pregnancy induced carpal tunnel. If that’s the case, it should resolve after delivery. Since there is no muscle wasting and
weakness, it’s not that severe
- start with conservative management in carpal tunnel syndrome, no improvement/severe  consider surgery

101.Which nerve is affected in case of fibular fracture?


A - common peroneal n
B - deep peroneal
C - superficial peroneal

Answer: common peroneal nerve


References:
- https://www.nlm.nih.gov/medlineplus/ency/article/000791.htm
- http://teachmeanatomy.info/lower-limb/nerves/common-fibular-nerve/

102.Trendelenburg gait features:


A - internal rotation with adduction
B - internal rotation with ab
C - external rotation with abduction
Answer: A
Evidence: Since the Trendelenburg gait develops as a result of weakness in Abductor muscle, the affected side tend to be in adduced
position with internal rotation, based on the pictures below:

185
103.Cancer patient c/o bone pain if you suspect mets what is the best radiological test?
A. Bone scan
B. MRI
C. Positron immersion tomography.
Answer: A. (Bone scan . step up to medicine)

104.Most common site of metastasis:


A. Bone
B. Kidney
C. Breast

Answer: A
The most common sites of cancer metastasis are, in alphabetical order, the bone, liver, and lung.
Reference: http://www.cancer.gov/about-cancer/what-is-cancer/metastatic-fact-sheet#q3

105.Young man close the door on his nail, the colour of which became blue under the nail. What will you do?
A - evacuate hematoma
B - remove nail
C - reassure it well resolved by itself

Answer: C (if painful, decompression is indicated)


References: http://www.webmd.com/skin-problems-and-treatments/bleeding-under-nail?Page=2 ;
Http://emedicine.medscape.com/article/82926-overview#a3

106.Young adult playing golf, came with Hx of atrophy of thenar muscles and tenderness at medial epicondyle, first evaluate:
A - X-ray
B - CT scan
C - reassure

Answer: A or C? (Golfer’s elbow; no need for imaging initially) | Reference: http://emedicine.medscape.com/article/97217-workup


Evidence: Plain x-rays are necessary to rule out a bony supracondylar process in the distal humerus or other post traumatic or
degenerative pathology at the elbow as the cause for compression.
Reference: http://www.orthoontheweb.com/nerve_compression.asp +

186
107.Physician in the clinic tells the child to bend forward and hang his both hands freely. This test is used in detection of?
A - Rectal Prolapse
B - Sexual Abuse
C - Scoliosis

Answer: C
Reference: http://www.physio-pedia.com/Adam's_forward_bend_test

It is called ADAM test which is a screening test for scoliosis.


Http://www.orthobullets.com/spine/2053/adolescent-idiopathic-scoliosis

108.Scoliosis what radiological method will you order:


A - Xray
B - CT scan
C - MRI

Answer: A
Evidence: PA spine radiographs — Radiographs are required to confirm the diagnosis of scoliosis, evaluate the etiology (congenital,
neuromuscular, idiopathic), determine the curve pattern and measure the magnitude (Cobb angle), and to evaluate skeletal maturity
(to determine the risk for progression).
Reference: uptodate: http://www.uptodate.com/contents/adolescent-idiopathic-scoliosis-clinical-features-evaluation-and-
diagnosis?Source=machinelearning&search=scoliosis&selectedtitle=1~150&sectionrank=2&anchor=H15#H15

109.Which part of bone is firstly affected in haematogenous osteomyelitis?


A - epiphysis.
B - metaphysis.
C - diaphysis.

Answer: B | References: http://patient.info/doctor/osteomyelitis-pro +


Http://www.uptodate.com/contents/overview-of-osteomyelitis-in-adults?Source=outline_link&view=text&anchor=H1#H1

110. Pt can't do abduction at hip joint, site of injury?


A - Gluteal maximus.
B - Inferior gluteal artery.
C - Head of femur.

Answer: A

111.Fracture of the tibia, displaced transverse, mx?


A - Close redaction.
B - Open with nail.
C - Antibiotic.

Answer: B
112.Elderly having knee pain for years and he is on NSAID when needed. Now he developed Left foot oedema. What is the best
investigation?
A - Echo
B - CBC
C - Doppler

Answer: A

187
113.Patient with active osteoarthritis with knee swelling on ex there is effusion but no pain not hot with crepetus during
movement. What is the best next? (answer not known)
A - Bilateral knee x Ray and ibuprofen
B - Aspirations of fluid
C - Bone…

Answer: ?

114.Child with FOOSH. He has pain on affected arm and shoulder. X-ray shows midclavicular fracture with segments overriding.
What's the mang:
A-sling
B-surgery
C-closed reduction
Answer B
Http://www.uptodate.com/contents/clavicle-
fractures?Source=see_link&sectionname=FRACTURES+OF+THE+MIDDLE+THIRD+%28MIDSHAFT%29+OF+THE+CLAVICLE&anchor=H6
#H6

115.A thyroid-disease patient developed pain while moving her shoulder and decreased range of motion. What is the diagnosis?
A. Impingement syndrome
B. Rotator cuff tear
C. Adhesive capsulitis
Answer : C

116.Best x ray to show fracture rib


A-AP
B-PA
C- Oblique
Answer: c
45° oblique view on expiration is recommended for radiographic imaging of patients with clinical signs of fracture, e.g. Evaluation of
lower rib fractures, while 45° oblique view during fast breathing is recommended for suspected upper rib fractures.
Http://www.ncbi.nlm.nih.gov/pmc/articles/PMC3529706/

117.Trauma to the knee ABI was low next step?


A. Angio Doppler
B. Ct
C. MRI
Answer: A
If ABI in knee injury less than 0.9 the patient should undergo arterial duplex ultrasound
http://www.orthobullets.com/trauma/1043/knee-dislocation

118.X ray Pic of fibula fracture which was multiple, + other pic for the leg of the pt
And gave hx and asked what u will do also,,
A. -Discharge home
B. -Debriment internal fixation
C. -External fixation

119.(photo & x-ray of open fracture). Pt had MVA and presented to ER with this legion. After stabilizing the pt what you will do? :
A. Give oral antibiotic & discharge

188
B. Discharge home no need for further management
C. Immediate surgery for debridement & ORIF

Answer: C
Http://orthoinfo.aaos.org/topic.cfm?Topic=A00582

120.Median nerve injury cause:


A. Ape hand
B. Clow hand
C. Waiter's tip deformity
Answer: A

121.Acute knee swelling with balloted patella, what to do next


A. ESR
B. CBC
C. Arthocentesis
Answer: missing data
Http://www.aafp.org/afp/2000/0415/p2391.html

122.What is the priority in pt management :


A. Airway
B. Breathing
C. Circulation
Answer: A

123.8 year-old male with left hip externally rotated with decrease in range of motion and decrease abduction of the hip. X-ray
identical to this one. What is your diagnosis :
A. Developmental dysplasia of the hip (DDH)
B. Legg–Calvé–Perthes disease
C. Slipped capital femoral epiphysis

Answer: A
Explanation:
 DDH is initially a clinical diagnosis that’s suspected when there’s a limited abduction or asymmetry between gluteal
folds. Physical signs include Barlow and Ortolani’s test. In x-ray, patients show broken shenton’s line & femoral neck
above Hilgenreiner’s line.
 Legg-Calvé-Perthes disease: idiopathic avascular necrosis of femoral head that is commonly found in boys 4-10 years
old. Initially patients present with painless limp.
 Slipped capital femoral epiphysis: separation of proximal femoral epiphysis through growth plate that presents with
painful limp
Reference: First aid step 2CK
124.X-ray for displaced tibial fracture with another pic of skin sinus with discharge, received iv ab and tetanus toxiod , what is ur
next management :
A. discharge on oral antibiotic
B. external fixation
C. surgical debridement with internal nail fixation

Answer: C

189
Explanation: the picture is probably is of the open fracture and since it is described as sinus then size most likely <1 cm or size 1-10
cm, respectively type 1 or type 2 in the Gustilo classification. After primary debridement, Intramedullary nailing is the best option for
Gustilo-Anderson types I, II, and III fractures.
Reference: http://emedicine.medscape.com/article/1249761-treatment#d10

125.Patient presented with distal thigh pain erythema and warmth seems inflamed but the knee is completely normal there’s
limited range of motion with tenderness which of the following would be helpful in establishing the diagnosis:

A.C-reactive protein (or ESR)


B.X ray
C.Blood culture

Answer: b
Http://emedicine.medscape.com/article/336054-overview#a6

126.Epileptic patient had an episode of seizure presented to ER with arm in adduction and internal rotation what do you expect
him to have: Hx of epi
A. Anterior dislocation
B. Posterior dislocation
C. Inferior dislocation

Answer: B
Posterior shoulder dislocation usually associated with seizures.

Http://lifeinthefastlane.com/posterior-shoulder-dislocation/

Stress fracture – READ ABOUT


Https://www.merckmanuals.com/professional/injuries-poisoning/sports-injury/stress-fractures

1. Mechanism of injury – A blow to the anterior portion of the shoulder, axial loading of an adducted and internally rotated
arm, or violent muscle contractions following a seizure or electrocution represent the most common causes of posterior
shoulder dislocation [25-27].
2. Examination – Examination reveals prominence of the posterior shoulder with flattening anteriorly. The coracoid process
appears prominent. The patient holds the arm in adduction and internal rotation and is unable to externally rotate (picture
3 and movie 1) [28].
3. Imaging studies – Radiographic evidence of a posterior shoulder dislocation on a standard anteroposterior (AP) view is
subtle and may go undetected in up to 50 percent of cases [29]. Clues to the diagnosis include the "light bulb" sign, rim sign,
and trough line sign

127.16 years old boy known case of sickle cell anemia presented to with painful right hip pain for several weeks (this was the
scenario and it was for several weeks ). What is the most likely diagnosis:
A. Avascular necrosis
B. Stills disease
C. Tumor
** osteomyelitis was not in the choices

Answer: A
A condition caused by interruption of blood supply. Remember ASEPTIC mneumonic:
Alcohol, AIDS
190
Steroids (most common), Sickle, SLE
Erlenmeyer flask (Gaucher’s)
Pancreatitis
Trauma
Idiopathic/ Infection
Caisson’s (the bends)
Http://www.orthobullets.com/recon/5006/hip-osteonecrosis

Explanation: femoral head avascular necrosis pain most commonly localized to the groin area, but it may also manifest in the
ipsilateral buttock or greater trochanteric region. Approximately 30% of all patients with SCD have hip pathology by age 30 years.
The other choice, Adult Still's disease is a rare illness in which almost all patients have high fevers, rash, and joint pain. It may lead to
long-term (chronic) arthritis.
Reference: http://emedicine.medscape.com/article/205926-clinical
Https://www.nlm.nih.gov/medlineplus/ency/article/000450.htm

128.Pt with bilateral deltoid muscle weakness and other symptom?


A. Brachial plexus neuropathy
B. Neck spasm
C. Cervical spondylopthy
Answer: The main cause could be due to cervical radiculopathy or Brachial injury (Erb’s Palsy) depending on the case given.
However, Cervical spondylosis represents the natural degenerative process of the cervical motion segment (intervertebral disc and
facets) often leads to the clinical conditions of cervical radiculopathy, cervical myelopathy, disco genic neck pain.
Cervical Radiculopathy
A clinical symptom caused by nerve root compression in the cervical spine C5 radiculopathy
 Deltoid and biceps weakness
 Diminished biceps reflex
Or Part of a brachial plexsus injury
Http://www.orthobullets.com/spine/2030/cervica
l-radiculopathy
Http://www.orthobullets.com/trauma/1008/brac
hial-plexus-injuries
Http://www.orthobullets.com/spine/2029/cervica
l-spondylosis

129.Acute knee swelling with balloted patella,


what to do next (ortho)
A. ESR
B. CBC
C. Arthocentesis
Answer: According to aafp, the approach of any swollen knee algorithm by order is
1- Hx
2- PE
3- Radiograph
4- Arthocentesis

130.Girl with Right knee swelling Aspirations results show (?) Wbcs and what best management:

Answer: ? (missing details of Q, missing options, answer not known)

191
Evidence:
- if the WBC 15,000 to 200,000 the diagnosis is (Septic Arthritis)
- Empiric intravenous antibiotic treatment of septic arthritis should be based on the organism found in the Gram stain of the synovial
fluid, or on the suspicion of a pathogen from the patient's clinical presentation.

References:
- all about the knee: http://www.aafp.org/afp/2000/0415/p2391.html#abstract
- management of septic arthritis according the gram stain
- http://www.aafp.org/afp/2011/0915/p653.html#sec-3

131.Case with pic: old male complaining of cervical pain abnormal gait and his wife (‫الحظت انو االشياء بتطيح من يدو كتيير‬
A. Cervical stenosis ( my answer)
B. Spinal tumor
C. Stroke

132.Peroneal nerve is branch of ?


A. Femoral
B. Posterior tibial
C. Anterior tibial
Answer:?

The common peroneal nerve is a branch of the sciatic nerve. It has 2 branched the superficial and deep peroneal nerves.

Reference: Orthobullet

133.Child with history of comminuted clavicular # due to MVA, ttt :


192
A. Arm sling
B. Close reduction
C. ORIF
Answer: C
Reference: http://www.orthobullets.com/trauma/1011/clavicle-fractures
134.Stiffness with pain in shoulders and osteopenia:
A. bursitis
B. arthritis
C. adhesive capsulitis

Answer: question and choices incomplete, but the writer put adhesive capsulitis as an answer.
Explanation: The loss of passive range of motion (ROM) is a critical element in establishing the diagnosis of a true frozen shoulder
(adhesive capsulitis). Although conditions such as subacromial bursitis, calciifc tendinitis, and partial rotator cuff tears can be
associated with significant pain and loss of active ROM, passive ROM is preserved. Therefore, patients with those conditions should
not be classified as having a frozen shoulder. Osteopenia of the humeral head may be noted as a result of disuse.
Reference: http://emedicine.medscape.com/article/1261598-overview#a5
Http://www.aafp.org/afp/2000/0601/p3291.html

Explanation: Bone scans (Nuclear scintigraphy) are useful in a wide range of diseases. A common reason to obtain a bone scan is in
the evaluation of pain, in which a bone scan can help determine whether the source of the pain is from bone pathology or from the
soft tissues. For example, a long-distance runner may have foot pain due to a fracture or a sprain. The bone scan can help determine
if a bone injury or a tendon sprain is the cause of the pain. Bone scans can also be useful in the evaluation of systemic diseases such
as cancer or nonspecific widespread bone pain.
Reference: http://emedicine.medscape.com/article/2109077-overview#a1

135.Case scenario about child fall on his toys and twisted his ankle, they mention that he could not walk and want to be held by his
parents:
A. Soft tissue swelling of the ankle
B. Spiral fracture of tibia
C. Dislocation of fibula

ANSWER: A
- Soft-tissue injuries fall into two basic categories: acute injuries and overuse injuries.
1- Acute injuries are caused by a sudden trauma, such as a fall, twist, or blow to the body. Examples of an acute injury
include sprains, strains, and contusions.
2- Overuse injuries occur gradually over time, when an athletic or other activity is repeated so often, areas of the body do
not have enough time to heal between occurrences. Tendinitis and bursitis are common soft-tissue overuse injuries.
- Http://orthoinfo.aaos.org/topic.cfm?Topic=A00111

136.Case scenario about child falling on his toys and twisting his ankle, they mentioned that he couldn't walk and want to be held
by his parents:

A. Soft tissue swelling of the ankle


B. Spiral fracture of tibia
C. Dislocation of fibula

Answer is B
 Fibular dislocation is pretty rare.
 Ankle sprains & strains don’t cause inability to bare weight, only pain on walking
Source: http://www.orthobullets.com/trauma/1045/tibia-shaft-fractures
193
Http://www.orthobullets.com/foot-and-ankle/7028/low-ankle-sprain

137.Temporal arthritis pt "clear 194cenario" this pt is in higher risk of:


A. CAD
B. Blindness
C. Brain tumor
Answer: B
Around 50% of patients with GCA experience visual symptoms!
Http://emedicine.medscape.com/article/332483-clinical#showall
http://www.merckmanuals.com/professional/musculoskeletal-and-connective-tissue-disorders/vasculitis/giant-cell-arteritis

138.Non athletic patient after 15 of football playing he her a click sound when he do a plantar flexion while the knee extended
after that he can not do plantar flexion against resistance or stand on the tiptoe what is the tendon rupture :
A. olecranon
B. quadriceps
C. rectus femoris

139.Patient had injury to the soles of left foot what is the first structure u can see>>
A. Tibias posterior
B. Extensor halluces lounges tendon
C. Extensor digiturum longus
Answer: a. Tibias posterior.

140.Loss of sensation of the 1st lumbrical in the dorsal side of hand


A. Radial
B. Ulnar
C. Median.
Answer: c. Median. Medscape
Http://emedicine.medscape.com/article/1285060-overview
141.Injury in 1st layer of plantar foot, which is the structure may be affected?
A- Musculocutaneous nerve
B- Abductor hallucis
C- Flexor digitorum longus
Answer: B
First Plantar Layer: Most superficial of all the layers: Muscles
- Abductor hallucis
- Flexor digitorum brevis (FDB)
- Abductor digiti minimi

Reference: http://www.orthobullets.com/foot-and-ankle/7003/layers-of-the-plantar-foot

142.Carpenter 80 yrs. Can’t raise shoulder. X-ray showing osteopenia. Diagnosis


A. Glionhumoral arthritis
B. Acromial arthritis
C. Bursitis
194
Answer: C
The patient is carpenter means doing repetitive movements.
Reference: http://emedicine.medscape.com/article/2145588-clinical

143.Child was playing with his toy, as he was walking around he tripped and twist his leg, since then he does not want to walk
and want to be carried around. What is wrong with him?
A. Fracture of the tibia
B. Fracture of the femur
C. Soft tissue swelling of the ankle
Answer: Most likely C. Question is not clear but ankle strain is more likely in this case.

144.Pt with leg pain for a while. X-ray shows periosteal elevation and onion skin. Wts the mng?
A- IV antibiotic
B- Warm comp, rest and elevation
C- Steroids
Multilayered periosteal reaction, also known as a lamellated or onion skin periosteal reaction, demonstrates multiple concentric
parallel layers of new bone adjacent to the cortex, reminiscent of the layers on an onion. The layers are thought to be the result of
periods of variable growth.
It has been associated with:
osteosarcoma
acute osteomyelitis
Ewing sarcoma
Langerhans cell histiocytosis (LCH)

145.Hyperextension of DIP will cause?


2 choice Rupture tendon muscle,
A. Flexor digotorum profondus
B. Intra articular fracture of PIJ join
C. Extra articular fracture of DIJ

146. Fracture of head and neck of fibula which artery is affected?


A. Deep something
B. Superficial something

- Superior segment of Circumflex fibular artery which comes from posterior tibial A.
- Also anterior tibial A passes medial to fibula neck.
* If the question is asking about the nerve -_-, the nerve is common peroneal nerve.

147.A patient present with painful limping, fixation of femur at 30 degrees with limited internal rotation and maybe shortening
not sure. What is the most likely diagnosis?
A. Legg–Calvé–Perthes disease
B. Slipped capital femoral epiphysis
Answer: B
Explanation Both diseases have limited abduction and internal rotation, the difference is in the type of limp and age group
 Legg-Calvé-Perthes disease: idiopathic avascular necrosis of femoral head that is commonly found in boys 4-10 years old.
Initially patients present with painless limp.
195
 Slipped capital femoral epiphysis: separation of proximal femoral epiphysis through growth plate that presents with painful limp
Reference First aid step 2ck

Type I Salter-Harris epiphyseal injury at proximal hip most common adolescent hip disorder. The risk factors are: male, obese (#1
factor), hypothyroid (risk of bilateral involvement). The clinical features of acute slipped capital femoral epiphysis : sudden, severe
pain with limp. The clinical features of chronic slipped capital femoral epiphysis are : groin and anterior thigh pain ,positive
Trendelenburg sign on a affected side, tender over joint capsule and restricted internal rotation, abduction, flexion (Whitman’s sign:
obligatory external rotation during passive flexion of hip)

Reference: Toronto notes

148.Femoral tumors contain tumors and calcification


A. Osteosarcoma
B. Ewing’s

Answer: ?
Merck Manual
Osteosarcoma (osteogenic sarcoma): is the 2nd most common primary bone tumor and is highly malignant. It is most common
among people aged 10 to 25, although it can occur at any age. Osteosarcoma produces malignant osteoid (immature bone) from
tumor bone cells. Osteosarcoma usually develops around the knee (distal femur more often than proximal tibia) or in other long
bones, particularly the metaphyseal-diaphyseal area, and may metastasize, usually to lung or other bone. Pain and swelling are the
usual symptoms.
Findings on imaging studies vary and may include sclerotic or lytic features. Diagnosis requires biopsy. Patients need a chest x-ray
and CT to detect lung metastases and a bone scan to detect bone metastases
Ewing’s sarcoma: of bone is a round-cell bone tumor with a peak incidence between 10 yr and 25 yr. Most tumors develop in the
extremities, but any bone may be involved. Ewing’s sarcoma tends to be extensive, sometimes involving the entire bone shaft, most
often the diaphyseal region. About 15 to 20% occur around the metaphyseal region. Pain and swelling are the most common
symptoms.
Lytic destruction, particularly a permeative infiltrating pattern without clear borders, is the most common finding on imaging, but
multiple layers of subperiosteal reactive new bone formation may give an onion-skin appearance. X-rays do not usually reveal the
full extent of bone involvement, and a large soft-tissue mass usually surrounds the affected bone. MRI better defines disease
extent, which can help guide treatment. Many other benign and malignant tumors can appear very similarly, so diagnosis is made
by biopsy. At times this type of tumor may be confused with an infection. Accurate histologic diagnosis can be accomplished with
molecular markers, including evaluation for a typical clonal chromosomal abnormality
149.A young person with pain in forearm worsening day by day, xray shows "onion peel" best investigation?
A. MRI
B. CT
Answer: A
It's Ewing sarcoma; MRI for all bone tumor or US except Osteoid osteoma with CT

150.Child sitting always in w position what the change in bone: (missing options)
A - metatarsus adductus.
B - femoral anteversion (femoral torsion)

Answer: B [femoral anteversion (femoral torsion)]


Evidence: In Toeing: metatarsus Adducts, internal tibial torsion, femoral anteversion; Out
toeing: femoral retroversion. References: VMA course +
http://www.wheelessonline.com/ortho/femoral_anteversion +
http://www.childrenshospital.org/conditions-and-treatments/conditions/femoral-anteversion/symptoms-and-causes
196
151.Old age with R hip pain when palpate the joint with normal adduction flexion and abduction best next management is
A. NASID
B. Physiotherapy

152.A child with avascular necrosis of the head of femur. What should be done?
A - surgical correction
B - keep immobile for 6 weeks
C - keep externally rotated

Answer: ? (it depends on further details in the question)


Evidence:
AVN (Avascular Necrosis) or Osteonecrosis:
Commonest Presentation: Pain (inguinal area then radiated to the buttocks and thigh).
Management:
- Nonoperative management → essentially, with analgesics and pharmacological agents,
- Restricted patient weight bearing with the use of a cane or crutches has not been shown to affect the natural history of the disease
and is useful only in controlling symptoms, poor outcome (only 15% resolved with nonoperative intervention). Although nonsurgical
treatment options like medications or using crutches can relieve pain and slow the progression of the disease, the most successful
treatment options are surgical. .
- Joint-preserving procedures → In early stage 0 to II lesions, in young active patients, core decompression is the most conservative
surgical procedure that offers the best chance at preserving the femoral head.
- Joint replacement
Reference: http://orthoinfo.aaos.org/topic.cfm?Topic=a00216

Evidence:
Until recently, most children with Perthes’ disease were treated with a plaster cast or brace, or surgery. However, it is now known
that at least half of cases heal well without any treatment, particularly children aged five and under, and milder cases
The aim of treatment is to promote the healing process and to ensure that the femoral head remains well seated in the hip socket as
it heals and remodels. Treatments advised can depend on
- child's age. Younger children (age 6 and below) have a greater potential for developing new, healthy bone.
- degree of damage to the femoral head. If more than 50% of the femoral head has been affected by necrosis, the potential for
regrowth without deformity is lower.
- stage of disease at the time your child is diagnosed. How far along the child is in the disease process.
Treatments may include ‘observation’, bed rest and crutches, a plaster cast or special leg brace, or surgery
Reference: http://www.perthes.org.uk/what-is-perthes-disease/

Explanation: patient has Legg-Calvé-Perthes disease: avascular necrosis of the proximal femoral head. Medscape: Initial therapy
includes minimal weight bearing and protection of the joint. Best practice: in patients <5 years, mobilization with monitoring. In 5-7
years, with <50% necrosis, mobilization with monitoring; if necrosis >50%, surgery is the treatment.
References: http://emedicine.medscape.com/article/1248267-treatment#d10
Http://bestpractice.bmj.com/best-practice/monograph/751/treatment.html

References: http://www.ncbi.nlm.nih.gov/pubmed/3047257 &


http://emedicine.medscape.com/article/333364-treatment

197
153.Pt had posterior hip dislocation? (no stem of Q, missing options)
A - internal rotation  adduction
B - external rotation  abduction

Answer: A | Reference: http://slideplayer.com/slide/4311100/

154.Best exercise for osteoporosis? (missing options)


A - Low resistance exercise and conditioning
B - Low resistance and highly repetitive weight bearing

Answer: B
Evidence: Low-impact (resistance) and weight-bearing exercises is recommended e.g. Walking/jogging
Reference: Medscape: http://emedicine.medscape.com/article/330598-overview#showall

155.Boy was playing football with barefoot and was injured in his sole, and Dr found that it is only superficial plantar injury which
structure may be affected: (missing options)
A - posterior tibialis tendon
B - adductor hallucis longus

Answer: abductor hallucis longus, there is no adductor hallucis longus | Reference: consultant

156.To Prevent fracture caused by osteoporosis in old: (missing options)


B - Vit D
C - *other medication*

Answer: (Bisphosphonate is the first-line agent) | References: http://emedicine.medscape.com/article/330598-treatment#d13 &


http://www.uptodate.com/contents/prevention-of-
osteoporosis?Source=machinelearning&search=prevention+of+osteoporosis&selectedtitle=1~150&sectionrank=1&anchor=H28#H2
8
157.Old female with lytic bone lesion, high ALP, what’s the tx? (missing options)
A - Bisphosphonate
B - selective oestrogen

Answer: A | Reference: http://emedicine.medscape.com/article/1253331-treatment#d10


158.Elderly patient with lower back pain, what is the drug cause relaxation of muscle? (missing options)
A - Diazepam
B - Clorcarpine

Answer: A (diazepam) | Reference: http://www.webmd.com/back-pain/muscle-relaxants-for-low-back-pain

159.12 y/o Obese can’t bear weight with left hip external rotation, x-ray provided? (missing options)
A - Slipped capital femoral epiphysis
B - Fracture of femoral neck

Answer: A (12~ y/o, obese, limping & thigh is externally rotated  slipped capital femoral epiphysis) | Reference: Kaplan surgery

160. Which one of these is a feature of osteoporosis? (missing options)


A - Decreased bone mass.

198
B - Decreased mineralization of bone.

Answer: A

161. Femoral neck # his leg was rotated laterally which muscle responsible: (missing options)
A - Rectus femoris.
B - Gluteas maximus.

Answer: Iliopsoas muscle | Reference: Rockwood and Green's Fractures in Adults page1570

162. Boy was playing football with barefoot and was injured in his sole, and Dr found that it is only superficial plantar injury
which structure may be affected: (missing options)
A - Posterior tibialis tendon.
B - Adductor halluces longus.

Answer: flexor digitorum longus

163.Pt c/o mild neck pain then sudden he c/o electrical pain on left arm and weakness and loss of tendon reflexes ? Diagnosis ?

A- cervical disc prolapse


B- polymyalgia rhumatica

Answer: A
Explanation: spontaneous onset neck pain is More likely to be associated with cervical spondylosis. Weakness and Decreased
reflexes are a sign of radiculopathy and cervical spondolysis. Polymyalgia rheumatica doesn't cause neurological symptoms.

Reference: http://bestpractice.bmj.com/best-practice/monograph/577/diagnosis/history-and-examination.html

164.Young male, complains of pain in elbow and shoulder since (1 w or 1 month , not sure ) he can't full extend his elbow , also
mention he squash player what is diagnosis :
A. Olecranon bursitis
B. Fracture
Squash is usually associated with tennis elbow, the question has missing details. If there's a direct trauma to the elbow it might be
olecranon bursitis.
165.Function of ACL
A. Medial rotation of tibia in relation to femur
B. Prevents anterior (forward) movement of the tibia off of the femur

ANSWER: B
- ACL attaches to anterior aspect of the tibia and courses superiorly, posteriorly, and laterally to attach to the lateral condyle
of the femur
- ACL prevents Anterior Displacement of the tibia under the femur
- Tension of the ACL is greater when the knee is extended and resists hyperextension
- It is weaker than the Posterior Cruciate Ligament
- KAPLAN USMLE ANATOMY

199
166.After fracture which one will be elevated?
A. CK
B. ALP
CK is found in the mitochondria and cytoplasm of skeletal muscle (predominantly), cardiac muscle, brain, and other visceral tissues.
The 2 subunits can form 3 isozymes: CK-MM, CK-MB, and CK-BB. Skeletal muscle, myocardium, and neuronal tissue are the main
sources of CK-MM, CK-MB, and CK-BB, respectively.
Increased CK is predominantly used to diagnose neuromuscular diseases and acute myocardial infarction. Neuromuscular disorders
include myopathies, muscular dystrophy, rhabdomyolysis, drug-induced myopathies, neuroleptic malignant syndrome, malignant
hyperthermia, and periodic paralyses.
Http://emedicine.medscape.com/article/2074023-overview#a2
Alkaline phosphatase is abnormal in Cholestasis, hepatocellular enzyme induction, canalicular injury, children during bone growth,
bone disease, pregnancy (placenta origin)
Http://www.medscape.com/viewarticle/710045_3

Explanation: Orthopedic trauma such as fractures and compartment syndrome can cause rhabdomyolysis by direct injury and
disruption of muscle tissue. This causes leakage of cellular contents such as CK and potassium.
Reference http://emedicine.medscape.com/article/1007814-overview#a1

167.After fracture which u must do for evaluate bone density?


A. X ray
B. Dexa
Http://www.uptodate.com/contents/bone-density-testing-beyond-the-basics

168.What the first management of risk factor?


A. Dexa
B. Biphosphate.
Bone mineral density (BMD) measurement is recommended in the following patients:
 Women age 65 years and older and men age 70 years and older, regardless of clinical risk factors
 Postmenopausal women and men above age 50–69, based on risk factor profile
 Postmenopausal women and men age 50 and older who have had an adult-age fracture, to diagnose and determine the degree of
osteoporosis
The NOF recommends that pharmacologic therapy should be reserved for postmenopausal women and men aged 50 years or older
who present with the following:
 A hip or vertebral fracture (vertebral fractures may be clinical or morphometric [eg, identified on a radiograph alone])

200
 T-score of –2.5 or less at the femoral neck or spine after appropriate evaluation to exclude secondary causes
 Low bone mass (T-score between –1.0 and –2.5 at the femoral neck or spine) and a 10-year probability of a hip fracture of 3% or
greater or a 10-year probability of a major osteoporosis-related fracture of 20% or greater, based on the US-adapted WHO
algorithm for calculating fracture risk ( FRAX)
Http://emedicine.medscape.com/article/330598-overview

169.Strongest ligament of hip prevent injury? (Prevent overextension of the hip)


A) Iliofemoral
B) Ischiofemoral
Answer: A
The iliofemoral ligament is not only stronger than the two other ligaments of the hip joint, the ischiofemoral and the pubofemoral,
but also the strongest ligament in the human body
Https://en.wikipedia.org/wiki/Iliofemoral_ligament

Evidence: Iliofemoral ligament: Located anteriorly. It originates from the ilium, immediately inferior to the anterior inferior iliac
spine. The ligament attaches to the intertrochanteric line in two places, giving the ligament a Y shaped appearance. It prevents
hyperextension of the hip joint during standing by screwing the femoral head into the acetabulum.
Reference: http://teachmeanatomy.info/lower-limb/joints/the-hip-joint/

170.Young male was stabbed in his back can't (abduct or adduct) his right thigh what is the affected muscle?
A. Magnus
B. Longus

Http://teachmeanatomy.info/lower-limb/muscles/thigh/medial-compartment/

171.Surgeon did suprarenal (procedure?), What he will hit anteriorly to it?


A. Kidney
B. Tail of pancreas

172.Muscle causes ankle dorsiflexion?


A) Tibialis anterior muscle
B) Gastrocnemius muscle
Answer: A
The tibialis anterior muscle is located alongside the lateral surface of the tibia. It is the strongest dorsiflexor of the foot. To test
the power of the tibialis anterior, the patient can be asked to stand on their heels (innervated by deep peroneal nerve).
Gastrocnemius muscle Actions: It plantar flexes at the ankle joint, and because it crosses the knee, it is a flexor there (Innervated by
Tibial nerve).
Http://teachmeanatomy.info/lower-limb/muscles/leg/anterior-compartment/

173.Patient complain of lower back pain after lifting heavy objects !


- The vast majority of people with low back pain improve within 4 to 6 weeks without treatment or with simple measures that
can be performed at home.
- Patients who are treated for acute back pain with bed rest have more pain and slower recovery than ambulatory patients.
-Therapy should focus on temporary symptomatic relief, to maximize patient comfort. We suggest a trial of short-term
treatment with either an NSAID or acetaminophen .

201
Reference: http://www.uptodate.com/contents/treatment-of-acute-low-back-
pain?Source=outline_link&view=text&anchor=H25#H25

174.Patient MVA and come with fracture of femur , tibia and fibula what is your action
A. Refer to orthopedic
Answer: supportive then refer
http://www.ncbi.nlm.nih.gov/pmc/articles/PMC3524792/

175.Patient has winging of the scapula. Which part of the trunk is affected?
Answer: long thoracic nerve
Explanation: it innervates serratus anterior which connects scapula to thoracic cage. Can be injured in mastectomy leading to
winging of scapula and lymphedema.
Reference first aid usmle step 1

176.Neck injury and cannot abduct arm > axillary


A. Shoulder dislocation
http://patient.info/doctor/shoulder-dislocation

177.A 3 year old girl with a typical history of elbow dislocation, which of the fallowing ligaments is affected?!
(No choices listed)

178.Child with painless limping. (missing options, answer not known)


A. Perthes disease

Answer: ?
Evidence:
Differential diagnosis of painless limping in children:
Developmental dysplasia of the hip
Neuromuscular disease (Cerebral palsy and muscular dystrophy)
Lower limb length discrepancy
Perthes disease: Typically present with painless limp, but may be associated with groin or anterior thigh pain (It become painful as it
progresses)
References: http://www.medscape.com/viewarticle/490135_4 ; http://www.aafp.org/afp/2000/0215/p1011.html

179.A male patient running 20 km complaining of upper leg pain. (missing options)
A - stress fracture

Answer: A | Reference: http://orthoinfo.aaos.org/topic.cfm?Topic=a00112

180.Athlete presented with severe painful plantar flexion which prevents him from raising the foot. Which ligament is affected?
A - Plantar fascia (Plantar fasciitis) (missing options)

Answer: A
Evidence:
The ligament affected depends on the mechanism of the injury:
Anterior Talofibular Ligament (ATFL)  Primary restraint to inversion in plantar flexion

202
Calcaneal Fibular Ligament (CFL)  Primary restrain to inversion in neutral or dorsiflexed position
Deltoid Ligament  injury occurs with pronation (eversion) trauma leading to forced external rotation and abduction of ankle
Reference: http://www.orthobullets.com/foot-and-ankle/7005/ankle-ligaments

181.Septic arthritis organism? (no stem of Q, missing options)


A - Staph aureus

Answer: A
Evidence:
Staphylococcus aureus is the cause of the vast majority of cases of acute bacterial arthritis in adults and in children ≥ 2 years.
Neisseria gonorrhoeae is the most common pathogen (75% of cases) among younger sexually active individuals.
Reference: http://emedicine.medscape.com/article/236299-overview

182.Constrictor with repeated hand use: (missing options)


A - Lateral epicondylitis

Answer: A
Evidence:
Lateral epicondylitis (tennis elbow): Repetitive extension (backhand shots) or idiopathic- pain near lateral epicondyle.
Medial epicondylitis (golfer’s elbow): Repetitive flexion (forehand shots) or idiopathic - pain near medial epicondyle.
Reference: http://orthoinfo.aaos.org/topic.cfm?Topic=a00068

183.A man arrived to ER after MVA the neck of the femur was fractured. Which of the following may happen in sequence?

Answer: ? (missing options, answer not known)


Possible answer  Injury of sciatic nerve; blood supply to femoral head—avascular necrosis (AVN)
Reference: Surgical recall 6th edition
184.Female patient with vertebral compression fracture. Which one of the following will prevent osteoporosis? (missing options)
A - Vit D supplement

Answer: A
Evidence:
Some of the most important treatments for preventing osteoporosis include diet, exercise, and not smoking, Calcium intake,
Vitamin D intake
Who needs treatment with a medication? — People with the highest risk of fracture are the ones most likely to benefit from drug
therapy. In the United States, the National Osteoporosis Foundation (NOF) recommends use of a medication to treat
postmenopausal women (and men ≥50 years) with a history of hip or vertebral fracture or with osteoporosis (T-score ≤-2.5).
Reference: http://www.uptodate.com/contents/osteoporosis-prevention-and-treatment-beyond-the-basics

185.A patient presented with Trendelenburg gait. Which muscle is responsible? (After MVA the patient walk toward the
unaffected side)
a. Gluteus medius (you need to know which site)

Explanation: A Trendelenburg gait, in which there is weakness of the hip abductors (gluteus medius muscle), is characterized by
trunk shift over the affected hip. Trendelenburg gait is caused by a neuromuscular weakness caused by a damage or a neuronal
injury of the superior gluteal nerve.
Reference: http://www.physio-pedia.com/Trendelenburg_Gait

203
186.Function of anterior forearm muscle: (missing options, answer not known)

Answer: ?
Evidence:
- Superficial group: 5 muscles: flexor carpiradialis, Flexor carpiulnaris, flexor digitorum superficialis, palmaris longus & pronator teres
- Deep group: 3 muscles; flexor digitorum profundus, flexor pollicis longus and pronator quadratus.
- All responsible for flexion and pronation
Reference: Zuckerman handbook of fractures

187.Type of bone hunger. (missing options, answer not known)

Answer: ? There are no types!


Evidence: Hungry bone refers to severe and prolonged hypocalcaemia which occurs after parathyroidectomy or less commonly
thyroidectomy despite normal or even elevated level of parathyroid hormone (PTH). The fall in serum calcium is primarily due to
functional or relative hypoparathyroidism leading to increase calcium influx of Ca+ into bone in patient without end-stage renal
disease Reference: uptodate and Toronto Notes

188.(Long scenario) Patient with low back pain, loss of sensation, weakness and loss of reflexes, what will do next?

Answer: ? (missing options, answer not known) Urgent MRI  possible answer
Evidence:
The most common indication for the use of imaging procedures (MRI or CT) is the clinical setting of Low back pain complicated by
radiating pain (radiculopathy, sciatica), as well as in cauda equina syndrome (bilateral leg weakness, urinary retention, saddle
anesthesia), neurogenic claudication and/or spinal stenosis. MRI of the lumbar spine has become the initial imaging modality of
choice in complicated LBP, displacing myelography and CT in recent years.
Reference: http://www.guideline.gov/content.aspx?Id=35145
Evidence:
- first-line: NSAID, physical therapy, steroid injection
- persists for 3-6 mo or severe weakness/autonimc dysfunction surgery
Reference: Orthobullet: http://www.orthobullets.com/spine/2037/lumbar-spinal-stenosis

189.Boutonnière deformity (BD)? (no stem of Q, missing options, answer not known)
Answer: ?
Evidence: can manifest itself acutely after trauma, but most bds are found weeks
following the injury or as the result of progressive arthritis. The proximal interphalangeal
(PIP) joint of the finger is flexed, and the distal interphalangeal (DIP) joint is
hyperextended. Reference: Medscape

190.Osteoporosis t score: (missing options)


A - > - 2.5 osteoprosis

Answer: > - 2.5 | Reference: 3rd edition UQU last touch Q 61

191.Q about old female with recurrent fracture: (no stem of Q, missing options)
A. Oestrogen Deficiency

204
Answer: oestrogen improves calcium absorption and reduces the amount of calcium lost in urine, so deficiency of oestrogen will
lead to osteoporosis. An osteoporosis-related fracture is at high risk of recurrent fractures.

192.Tx of medial epicondylitis

Answer: conservative physiotherapy | Reference: miller

193.Pt doesn’t complain of anything & has sudden knee swelling. What’s the best
thing to do? (missing options)

Answer: X-ray
Evidence:
- acute knee swelling warrants x-ray
- if traumatic/sport injury  Ottawa knee rule to decide if x-ray is needed
- Ottawa knee rule for trauma x-ray: > 55 yrs old, inability to flex 90°, inability to bear weight, tenderness
Reference: http://www.medscape.org/viewarticle/714758

194.Mechanism of Osgood Schlatter disease: (missing options)

Answer: osgood–schlatter disease or syndrome (tibial tubercle apophyseal traction injury and epiphysitis of the tibular tubercle) is
an irritation of the patellar ligament at the tibial tuberosity.
- It is characterized by painful lumps just below the knee and is most often seen in young adolescents.
- Risk factors include excess weight and overzealous conditioning (running and jumping).
- Diagnosis is made clinically
- Treatment is conservative with RICE (rest, ice, compression, and elevation) and, if required, acetaminophen.
Reference: http://www.orthobullets.com/sports/3029/osgood-schlatters-disease-tibial-tubercle-apophysitis

o Osgood-Schlatter disease:
o MOI: Overuse apophysitis of the tibial tubercle. Causes localized pain, especially with quadriceps contraction, in active
young boys.
o Treatment: Decrease activity for 2–3 months or until asymptomatic. A neoprene brace may provide symptomatic relief.

195.Muscle that extends the knee? (missing options)

Answer: The main muscle for extension is the quadriceps femoris, which is the most important muscle in stabilizing the knee joint.
The quadriceps is made up of the vastus medialis and lateralis, rectus femoris, and vastus intermedius.
Reference: http://emedicine.medscape.com/article/1898986-overview#a2

196.Chronic Gout? (no stem


of Q, missing options)

Answer: allopurinol
Evidence: picture 
Reference: Toronto note

205
197.Patient presented with knee swelling and pain they did x ray and aspiration and found negative birefringence needle like.
What are you going to discharge the patient with? (missing options, answer not known)
A - Allopurinol

Answer: A? (NSAID/steroid is used for acute flares, allopurinol is not used in the acute setting)
Reference: picture 

198.Pseudogout type of crystals? (no stem of


Q, missing options)

Answer: calcium pyrophysphate crystal


Evidence: picture 
Reference: Toronto note

199.Signs of osteoporosis on X-ray? (missing


options)

Answer: joint space narrowing, osteophytes,


subchondral sclerosis, and subchondral cysts
Reference:
http://emedicine.medscape.com/article/330598-
workup#c9

200.Carpal tunnel syndrome vs. Thoracic outlet obstruction? (no stem of Q, missing options)

Answer: ?
Evidence:
o Carpal tunnel syndrome: Reference: Step 2 CK page 221
Entrapment of the median nerve at wrist caused by decrease space of the carpal tunnel leading to parasthesia, pain, paralysis.
Causes: overuse of wrist flexors, associated with DM, thyroid dysfunction, pregnant, middle aged women.

o Thoracic outlet syndromes (TOS):


a constellation of signs and symptoms arising from compression of the upper extremity neurovascular bundle by various
structures in the area just above the first rib and behind the clavicle, typically within the confined space of the thoracic outlet.
Causes: rib anomalies, muscular anomalies, or a result of injury. Cervical ribs predispose the patient to TOS after
hyperextension-flexion (whiplash) injury. The absence of a rib anomaly makes the diagnosis of arterial TOS less likely. Many
patients with nerve TOS have a prior history of neck trauma or repetitive occupational physical stress. Similarly, venous TOS is
highly associated with repetitive movements, particularly with repetitive overhead upper extremity movements.
Reference: uptodate: http://www.uptodate.com/contents/overview-of-thoracic-outlet-
syndromes?Source=search_result&search=thoracic+outlet+syndrome&selectedtitle=1%7E36

206
201.Retired farmer recently he develops pain in the left arm, which with time progressed until he can't sleep on his left side, by
examination found to have severe decrease in motion, what does he have? (missing options, answer not known)
A - osteoporosis of the bone
B - spondylitis (arthritis that affects the spine only)

Answer: ? (can be rotator cuff injury, bursitis, …)


202.Male pt complains of weakness in flextion of both Rt knee and Rt hip which muscle is affected? (missing options)
A - Sartorius

Answer: A | Reference: http://www.orthobullets.com/anatomy/10055/sartorius

203.Active osteoarthritis in knee pain best exercise: (missing options)


A - Quadriceps Ms strengthen.
Answer: A (Quadriceps and Hamstring Ms lengthening + strengthening)
References: http://emedicine.medscape.com/article/330487-treatment#d10 & http://www.medscape.com/viewarticle/510997_2

& http://emedicine.medscape.com/article/327330-clinical#b4

204.Tx of de Queverian syndrome? is a tenosynovitis of the sheath or tunnel that surrounds two tendons that control movement
of the thumb))
Answer:
Evidence:
Usually to start with conservative interventions, which include a forearm-based thumb spica splint with the interphalangeal joint
free along with short-term nonsteroidal anti-inflammatory drugs (nsaids). We suggest a local glucocorticoid injection for patients
whose symptoms have not resolved with conservative management. Most patients recover with this intervention.
Patients who present with severe symptoms may benefit from a glucocorticoid injection at the initial presentation. Surgical release is
generally reserved for patients who have not improved with conservative therapy and one or two glucocorticoid injections.
Reference: http://www.uptodate.com/contents/de-quervain-
tendinopathy?Source=outline_link&view=text&anchor=H660498#H660498

Forearm-based thumb spica splint with the interphalangeal joint free as well as a concurrent trial of nonsteroidal antiinflammatory
drugs (NSAIDS) for pain relief
If persistante local glucocorticoid injection
For patients with persistent symptoms despite splinting and one or two glucocorticoid injections, surgical therapy may help relieve
symptoms

205.The most common cause of olecranon bursitis is? (missing options)


A - Repeated injury

Answer: A (it is called student’s elbow; repeated injury like leaning on hard surfaces e.g. Tables)
Reference: http://www.nhs.uk/Conditions/Bursitis/Pages/Causes.aspx

206.Pt with +ve sign of Finkelstein test what is your management? (missing options, answer not known)

Answer: Most likely is thumb splint +NSAID (conservative tx) (+ve Finkelstein sign in case of De Quervain's tenosynovitis)
Reference: http://orthoinfo.aaos.org/topic.cfm?Topic=a00007

207.Patient with loss of shoulder passive and active movement: (no stem of Q, missing options)

207
Answer: adhesive capsulitis

208. Soldier walks 1000 miles developed pain on foot: (no stem of Q, missing options)

Answer: Spring ligament


209.Male with lumbar and femoral fractures. Osteoporosis on xray. Alkaline phosphates level abnormal but the rest is fine.
What’s the most likely cause? (missing options, answer not known)

Answer: ?

210.Pt complaining of lower back pain after lifting heavy objects


Answer:
Muscle strain

211.Knee reflex?
Answer: L 3-4 but mainly L4

212.Patient complaining of pain on distal palmar aspect of finger and ask about blood supply?
A. Deep and superficial Palmar arch

213.Shingles at umbilicus level what is the dermatome;


A. 10

214.Patient was complaining of leg pain after running and relax after stretch of the muscle, the problem in which muscle ?
A. Soleus
Answer: Gastrocnemius

208
215.Case of cervical disc present with numbness, shoulder pain and stiffness.........

216.Patellofemoral pain syndrome


Http://emedicine.medscape.com/article/308471-overview

217.Patient with neck pain and occipital headache, no history of trauma, there is a limitation in neck movement. On examination,
there is weakness in the upper shoulder, what is the diagnosis?
A. Cervical spondylosis

ANSWER: A
- Sign and Symptoms: Neck Pain is often accompanied by stiffness, with radiation into the shoulders or occiput "Headache"
which may be chronic or episodic
- In cervical spondylotic myelopathy, the most typical examination findings are suggestive of upper motor dysfunction,
including hyperactive deep tendon reflexes, ankle and/or patellar clonus, spasticity (especially of the lower extremities), the
Babinski sign, and the Hoffman sign.
- Http://emedicine.medscape.com/article/1144952-clinical#b4

218.Case scenario of young adult with avascular necrosis of the head of humerus, what's the best treatment?

Answer: best treatment option depends on radiograph staging.


 Stage I & II: core decompression
 Stage III, IV and V: hemiarthroplasty vs. Total shoulder replacement
Source: http://www.healio.com/orthopedics/curbside-consultation/%7b267d23fd-dc67-4212-a421-
6fd1a8c48b62%7D/what-are-the-indications

219.What’s the nerve responsible for elevation of arms?

Answer: axillary nerve


 Axillary nerve supplies the deltoid muscle which is the main abductor of the arm (abducts arm beyond the initial 15° done
by supraspinatus muscle; supraspinatus is supplied by suprascapular nerve)

220.Obturator nerve injured which muscle affected?


A. Adductor muscle (longus Magnus brevis gracilis

The obturator nerve is responsible for the sensory innervation of the skin of the medial aspect of the thigh.

209
[2]
It is also responsible for the motor innervation of the adductor muscles of the lower extremity (external obturator. adductor
longus, adductor brevis, adductor magnus, gracilis) and the pectineus (inconstant). It is, notably, not responsible for the innervation
of the obturator internus, despite the similarity in name.
+
Http://teachmeanatomy.info/lower-limb/nerves/obturator-nerve/

221.Flat foot I think want the type of ligament??


A. Posterior Tibial Tendon
Acquired Flatfoot: There are numerous causes of acquired adult flatfoot, including fracture or dislocation, tendon laceration, tarsal
coalition, arthritis, neuroarthropathy, neurologic weakness, and iatrogenic causes. The most common cause of acquired adult
flatfoot is posterior tibial tendon dysfunction
Http://emedicine.medscape.com/article/1235600-overview#a8

222.Patient present with Hip and shoulder pain ESR high (polymyalgia rheumatic case ) in addition to symptoms what else can be
?? (proximal muscle tenseness)

Polymyalgia rheumatica (PMR) is a relatively common chronic inflammatory condition of unknown etiology that affects elderly
individuals.
Several diagnostic criteria for PMR exist. One set of diagnostic criteria is as follows:

 Age of onset 50 years or older


 Erythrocyte sedimentation rate ≥40 mm/h
 Pain persisting for ≥1 month and involving 2 of the following areas: neck, shoulders, and pelvic girdle
 Absence of other diseases capable of causing the same musculoskeletal symptoms
 Morning stiffness lasting ≥1 hour
 Rapid response to prednisone (≤20 mg)
Approximately 15% of patients with PMR develop giant cell arteritis (GCA), and 40-50% of patients with GCA have associated PMR.
Http://emedicine.medscape.com/article/330815-clinical

223.Osteoporotic Patient on medication that work by atp-analouge.

Answer: (Etidronate).
There are two classes of bisphosphonate: the N-containing and non-N-containing bisphosphonates. The two types of
bisphosphonates work differently in killing osteoclast cells.
Non-N-containing bisphosphonates: Etidronate, Clodronate, Tiludronate
The non-nitrogenous bisphosphonates (disphosphonates) are metabolised in the cell to compounds that replace the terminal
pyrophosphate moiety of ATP, forming a non-functional molecule that competes with adenosine triphosphate (ATP) in the cellular
energy metabolism.
Https://en.m.wikipedia.org/wiki/Bisphosphonate

224.2pic for hips limbs one painful another painless with progress painful (perth,hip dysplasia ,osteomyelitis) all same choice
A. Metaphyseal displaced.
B. Avascular necrosis of head of femur.
C. Fracture head of femur.

Answer: ?
- Legg-Calvé-Perthes disease (LCPD) is avascular necrosis of the proximal femoral head resulting from compromise of the tenuous
blood supply to this area. LCPD usually occurs in children aged 4-10 years. The disease has an insidious onset and may occur after an

210
injury to the hip. In the vast majority of instances, the disorder is unilateral. The earliest sign of LCPD is an intermittent limp
(abductor lurch), especially after exertion, with mild or intermittent pain in the anterior part of the thigh.
Http://emedicine.medscape.com/article/1248267-overview#a8

225.Internal iliac artery injury

Anatomy: The internal iliac artery supplies the walls and viscera of the pelvis, the buttock, the reproductive organs, and the medial
compartment of the thigh. The vesicular branches of the internal iliac arteries supply the bladder. It is a short, thick vessel, smaller
than the external iliac artery, and about 3 to 4 cm in length.
Https://en.m.wikipedia.org/wiki/Internal_iliac_artery

Most major iliac artery injuries are due to penetrating trauma. Intraoperatively, patients with iliac artery injuries present with active
pelvic bleeding or with a retro- peritoneal hematoma in the lateral aspect of the pelvis. Pelvic fractures associated with severe blunt
trauma may cause disruption of multiple branches of the internal iliac arteries and veins. This may result in fatal hemorrhage and
must always be kept in mind as a source of occult major bleeding.
Http://www.atcs.jp/pdf/2003_9_5/337.pdf

226.Lytic femur lesion & osteoporosis skull?


Answer: Paget disease
It is a chronic disorder of the adult skeleton in which bone turnover is accelerated in localized areas. Normal matrix is replaced with
softened and enlarged bone.
Sx: There are usually no symptoms for a prolonged period. If symptoms occur, they develop insidiously, with pain, stiffness, fatigue,
and bone deformity.
Signs: skull enlargement bitemporally and frontally (frontal bossing); dilated scalp veins; nerve deafness in one or both ears or
vertigo; headaches; angioid streaks in the fundus of the eye; a short kyphotic trunk with simian appearance; hobbling gait; and
anterolateral angulation (bowing) of the thigh, leg, or humerus, often with warmth and tenderness. Deformities may develop from
bowing of the long bones or osteoarthritis. Pathologic fractures may be the presenting manifestation.
Characteristic x-ray finding:
- Increased bone sclerosis
- Abnormal architecture with coarse cortical trabeculation or cortical thickening
- Bowing
- Bone enlargement

227.Lower back pain radiant to thighs what the investigation


In acute sciatica the diagnosis is based on history taking and physical examination and treatment is conservative (non-surgical).
Imaging may be indicated at this stage only if there are indications or “red flags” that the sciatica may be caused by underlying
disease (infections, malignancies) rather than disc herniation.
Although some authors favor magnetic resonance imaging above other imaging techniques because computed tomography has a
9 10
higher radiation dose or because soft tissues are better visualised, evidence shows that both are equally accurate at diagnosing
11
lumbar disc herniation. Radiography for the diagnosis of lumbar disc herniation is not recommended because discs cannot be
visualized by x rays.

228.Most significant risk factor for OP?


Strong Risk factors:
 Prior fragility fracture

211
 Female gender
 White ancestry
 Older age (>50 years for women and >65 years for men)
 Low BMI
 Fhx of maternal hip fracture
 Loss of height
 Postmenopause
 Secondary amenorrhoea
 Primary hypogonadism
 Smoking
 Excessive alcohol use
 Prolonged immobilisation
 Low calcium intake
 Vitamin D deficiency
 Glucocorticoid excess
 Corticosteroid use
BMJ http://bestpractice.bmj.com/best-practice/monograph/85/diagnosis.html

229.Tx of osteoporosis
A. Calcium, bisphosphonates, vitamin D. Step up to medicine

230.Patient was involved in an Accident lost lateral rotation of lower limb Which muscle are affected?
A. Gluteus maximus, Gluteus medius (anterior fibres laterally rotate),Piriformis, Obturator externus, Obturator internus,
Gamellus superior, Gamellus inferior and Quadratus femoris. Anatomyzone
http://anatomyzone.com/3d_atlas/musculoskeletal/lower-limb/hip-lateral-rotators/

231.Patient (long scenario) has low back pain. Investigations revealed fracture. He also has high temperature and night sweats.
What is the most likely diagnosis?
Answer: TB
Reference:
Pott disease (spinal TB): the most common symptom is local pain. In a classic text, the description of the patient with spinal TB
reads: "The muscle spasm, which extends beyond the diseased area, sometimes produces the well-known erect posture and
'aldermanic' gait. Constitutional symptoms, fever, and weight loss are present in less than 40 percent of cases. The most important
complication of spinal tuberculosis is cord compression during the active phase of the infection resulting in paraplegia (Pott's
paraplegia)
Http://cursoenarm.net/UPTODATE/contents/mobipreview.htm?9/62/10208#H6

232.A patient complains of gradual neck pain and loss of side-to-side movement. Neck x-ray showed osteophytes and narrowed
joint space. What is the most likely diagnosis?
Answer: Cervical spondylosis
Http://radiopaedia.org/articles/cervical-degenerative-spondylosis-grading
Http://emedicine.medscape.com/article/306036-clinical

233.Patient complaining of elbow pain that is radiating down and tenderness with dorsiflexion of wrist. Diagnosis?
Answer: Lateral epicondylitis (Overuse injury)

212
234.Pregnant after delivery, with epidural anesthesia she started to complain of Loss of sensation at medial side of the thigh,,
which nerve affected?
A. Obturator Nerve

235.Young patient came with effusion, tenderness in the knee, febrile ,next step?
" there was 5 Qs similar to this q , hip effusion " septic arthritis"
DIAGNOSIS — The definitive diagnostic test is identification of bacteria in the synovial fluid. In the setting of suspected joint
infection, synovial fluid aspiration should be performed (prior to administration of antibiotics); fluid should be sent for Gram stain
and culture, leukocyte count with differential, and assessment for crystals
236.Pain in the midline of the plantar foot with walking?

Pain in the midfoot can occur along the medial arch. The navicular bone serves as a keystone for arch stability, and any pain directly
over the navicular raises the possibility of serious injury. Such injuries include navicular stress fractures, tendinopathy of the
posterior tibialis tendon insertion onto the navicular, traumatic separation of an accessory navicular, and partial or complete tears of
the attachment of the plantar calcaneonavicular (spring) ligament.

Pain over the navicular with an unusual bony prominence suggests a possible accessory navicular, present in greater than 20 percent
of patients. Pain over the plantar surface of the arch can arise from an acute strain or longitudinal arch collapse.

A more common location for midfoot pain is over the dorsum of the articulation of the first tarsometatarsal (Lisfranc) joint, and
sometimes such pain extends to the articulations of the second and third tarsometatarsal joints as well. Injuries to the
tarsometatarsal joints are often referred to as Lisfranc injuries. When pain occurs along the dorsum of the Lisfranc joint, particularly
following acute trauma, a serious injury such as a fracture or fracture dislocation may have occurred. Diagnosis is made by
radiograph. Ganglions can develop at the dorsal midfoot and cause more localized pain.

The lateral midfoot includes the cuboid and the base of the fifth MT. Pain at the base of the fifth metatarsal is common, but pain
over the cuboid less so. Pain in the lateral midfoot caused by instability of the ligaments that stabilize the cuboid may only be
elicited by dynamic assessment of the midfoot articulations. The examiner can grasp the base of the fifth MT with one hand and the
cuboid with the other and see if excessive motion occurs with vertical displacement of the fifth MT on the affected but not
unaffected side. Tenderness at the base of the fifth MT suggests injury and warrants imaging with plain radiographs, as these injuries
can be difficult to diagnose and manage.

237.Which tendon medial to Dorsalis pedis artery?


Answer: extensor hallucis longus tendon
The dorsalis pedis artery pulse can be palpated readily lateral to the extensor hallucis longus tendon (or medially to the extensor
digitorum longus tendon) on the dorsal surface of the foot, distal to the dorsal most prominence of the navicular bone which
serves as a reliable landmark for palpation

213
Incomplete Questions

1. Skull fracture? (missing details of Q, no stem of Q, missing options, answer not known)
- C-spine.

Answer: ?

2. 7 years old presented with back pain. Had a similar attach 1 year ago? What is the diagnosis? (missing options, no answer)
A. Ancklyosing spondulitis.

Answer: ?

3. Numbness of the lateral side of hands and fingers in computer programmer female that is confirmed by phalen test , in
which position would u splint the hand :
Dorsiflexion

Explanation: Wrist splints with the wrist joint in neutral or slight extension (to be worn at nighttime for a minimum of 3-4 wk) have
some evidence for efficacy. Since choices are incomplete. If neutral is one of the choices, then it is the most likely answer. If not,
then dorsiflexion.
Reference: http://emedicine.medscape.com/article/327330-treatment

4. Read about when to do bone sciantography : to determine wether the pain from the bone or from its surrounding tissue .

5. Bisphosphonate which drug make boneclast ??

6. Old women came for check up and said decrease calcium in food and consider she think has risk factor of osteoporosis

7. Five years limping with pain over five months no Hx of trauma or fever?

8. Many Q about bisphosphonate may be more than 4. Read about it spinal osteoporosis in x-ray show Erosion spine

9. Between symphysis pubis and inguinal. What artery inferior?

A. Inferior mesenteric?
Not sure

10. Nerve between heads of pronator teres muscle? Median nerve

214
Anesthesia

215
1. In epidural anesthesia the anesthesiologist hit just lateral to spinal processes which structure he will injure/hit first?
A. ligamentum flavum
B. Posterior longitudinal ligament
C. Anterior longitudinal ligament
D. interspinous ligament.

Answer: A
http://www.frca.co.uk/article.aspx?articleid=100361

2. What is the fasting duration for non-breast milk?


A. 8 hours
B. 6 hours
C. 4 hours
D. 2 hours

Answer: B
8 hours (solid)
6 hours (formula)
4 hours (breast milk)
2 hours (clear liquid)
Reference: Toronto notes.

3. What is the sign that indicate inadequate general anesthesia?


A. Dry skin.
B. Hypotension.
C. Bradycardia.
D. Dilated Pupils.

Answer: D
Reference: Toronto Notes

4. Pregnant on delivery she has hypotension and dyspnea. Which type of anesthesia will be given?
A. Pudendal n
B. Local cervical
C. General anesthesia
D. epidural ( Newly added )

Answer: General anesthesia


Reference: UpToDate

5. During labor, the anesthesiologist injects analgesic drug at L3-L4, after 2 days the mother still have pain in the site of
injection. Which ligament is affected?
A. Anterior longitudinal ligament.
B. Posterior longitudinal ligament.
C. Ligamentum flavum.
D. Interspinous ligament.

Answer: D
In this Q: we don`t know if the doctor used spinal or epidural but in general pain at site of injection = inflammation of supraspinous
lig or interspinous lig. (Answered by an anesthesia consultant at KFMC)
Note: The associations between back pain and epidural analgesia are unclear.
216
- Local tenderness at the site of epidural or spinal placement are relatively common & usually clears within several days to 3 weeks
and may be related to superficial irritation of the skin or periosteal irritation or damage.
- Although short-term back pain is common, it does not appear to be related to the use of regional analgesia. Similarly, no causal
relationship exists between the use of epidural analgesia and the development of long-term postpartum backache.

6. Station +1 80% effacement and 4 cm dilatation and Rupture of membrane with clear fluids, : what anesthesia to give?
A. VS
B. B-Pudendal,
C. Para cervical,
D. GA

Answer: D

7. to increase hyperbaric in intrathecal analgesia put patient in :


A. lateral
B. supine
C. trundlberg
D. antitrendulberg

Answer: C
Hyperbaric solutions goes with gravity
Hypobaric solutions goes against gravity
So to increase hyperbaric solution’s spread the patient is put in antitrendulenberg position

8. Which of these analgesia has x100 the strength of morphine?


A. Fentanyl
B. Tramadol
C. NSAID

Answer: A
Reference: Toronto Notes.
Extra notes:
Common Side Effects of Opioids: Nausea and vomiting, Constipation, Sedation, Pruritus, Abdominal pain, Urinary retention,
Respiratory depression..
When prescribing opioids, consider: Breakthrough dose, Anti-emetics, Laxative.
Fentanyl is a powerful synthetic opiate analgesic similar to but more potent than morphine. It is typically used to treat patients with
severe pain, or to manage pain after surgery.
Reference: https://www.drugabuse.gov/drugs-abuse/fentanyl

9. Mallampati class 3 what you will see?


A. Soft palate and uvula
B. Soft palate and base of uvula
C. Soft palate,uvula and tonsillar pillars

Answer: B

10. Women in labor (long scenario) with low Hgb & platelet >> what is the Type anesthesia?
A. GA
B. Para cervical
C. Pudendal block

217
11. pregnant lady with hypotension , what type of anesthesia you will given :
A. Pudendal
B. Epidural
C. General
Answer: ?

12. When you are going to intubate unconscious male but there was difficulty with ventilation with bag-mask, what to do:
A. Proceed to intubation
B. Apply cricoid pressure
C. Head tilt

Answer: C
- A Patient’s head should be extended to make the airway patent (check the positioning section in the first reference)
- Mask seal – Mask seal requires reasonably normal anatomy, absence of facial hair, lack of interfering substances, such as
excessive vomitus or bleeding, and the ability to apply pressure to the face with the mask.

Reference : http://emedicine.medscape.com/article/80184-overview & https://www.das.uk.com/guidelines/cvci.html

13. Patient came from RTA , in preparing of setting of intubation patient became desatting more and ambo bag could not
maintain saturation, which should be done ?
A. More jaw thrust
B. More head tilt
C. Precede immediate for intubation

Answer: A or B
According to up to date, if the patient's oxygenation cannot be maintained, immediate rescue by cricothyrotomy is necessary.
Reference :
http://www.uptodate.com/contents/the-failed-airway inadults?Source=outline_link&view=text&anchor=H351136572#H351136572

14. Best medication for chronic pain disease?


A. Ibuprofen
B. Acetaminophen
C. There is another choice started with ci >>>.
Answer: b
http://www.webmd.com/pain-management/tc/chronic-pain-medications

15. What’s usually giving with analgesics to reduce side effects?


A. Cimetidine
B. Metoclopramide

Answer: B
Explanation: the question probably means opioid analgesics; with which we usually give antiemetic like metoclopramide (dopamine
antagonist). Cimetidine is a histamine H2 antagonist for GERD and peptic ulcer, and is not an antiemetic therefore, not the answer.
References: http://www.anzca.edu.au/Documents/Acute-Pain-final-version- page 65

16. During delivery something happened C/S was required, what type of anesthesia?
A. Pudendal.
B. General.

Answer: epidural and spinal anesthesia, B/c in General anesthesia the drugs are given to the mother will affect the infant.
Reference: http://www.uptodate.com/contents/c-section-cesarean-delivery-beyond-the-basics

218
17. Best MS relaxant in sever back pain?
A. Diazepam
B. Metaloxone

Answer: B
The goal of muscle relaxants is to normalize muscle excitability, decrease pain, and improve the motor function. They exert their
pharmacologic effect centrally at the level of the spinal cord, the brainstem, or the cerebrum.
http://www.emedexpert.com/classes/skeletal-muscle-relaxers.shtml

18. Elderly, asthmatic, what is the best induction


A. Propofol
B. ketamine

Answer: A
Propofol is considered to be the agent of choice for induction of anesthesia in asthmatics.
http://www.ncbi.nlm.nih.gov/m/pubmed/11050961/

19. How to prevent a high level of anesthesia?


A. Anti-Trendelenburg
B. Trendelenburg

Answer: A?
If the level of spinal anesthesia is not fixed, the Trendelenburg position can alter the level of spinal anesthesia and cause a high
level of spinal anesthesia in patients receiving hyperbaric local anesthetic solutions. This can be minimized by raising the upper
part of the body with a pillow under the shoulders while keeping the lower part of the body elevated above heart level.
Reference: http://www.nysora.com/index.php?news=3424

20. Which anesthetic agent provides sedation with sub-anesthetic dose (in other version: an analgesic in mild doses)
A. Ketamine
B. Pentamin
C. Midzpam

Answer: A

21. Patient with Sever asthmatic attack have O2 and inhaled beta agonist what to give?
A. intubation
Answer: A

22. Direction of IM injection related to sciatic nerve? safest to use the upper outer quadrant.

23. Patient with dilated pupil tachycardia cause? Sympathomimetic

24. Which Ligament when you do epidural anesthesia

25. Earliest sign of local anesthetic toxicity:


A. Tongue and circumoral numbness

Answer: A
Reference: Toronoto Notes.

219
Reference: http://www.ncbi.nlm.nih.gov/pubmed/25197290

26. Same of common about breastfeeding what is indicative of poor anesthesia??

Answer:
"awareness with recall" (AWR) refers to both intraoperative consciousness and explicit recall of intraoperative events. Other terms,
such as "intraoperative awareness during general anesthesia," "anesthesia awareness," or simply "awareness," are used as
synonyms.

Anesthetic underdosing — The most important contributing factor for AWR is underdosing of anesthesia relative to a given patient's
specific requirements.
This can occur for the following reasons [21-23]:
●There is a mistake or failure in the delivery of anesthesia
●The anesthetic technique results in inadequate anesthesia
●It is judged unsafe to administer sufficient anesthesia
●A specific patient's needs are underappreciated

Ref: UpToDate

220
Extra information

221
222
223
224

You might also like